Vous êtes sur la page 1sur 129

Test Series: October 2019

MOCK TEST PAPER 1


FINAL (NEW) COURSE: GROUP – II
PAPER – 6D: ECONOMIC LAWS
Attempt any four out of five case study based questions.
Each Case Study carries 25 Marks.
Time Allowed – 4 Hours Maximum Marks – 100
Case Study 1
Way back in November, 2011, Mr. Hariharan Reddy, a senior professor of Biology in the University of
Hyderabad, booked a 3BHK apartment in Royal Golf Burg - an impressive and integrated housing project
proposed to be developed by a reputed builder popularly known as Raj Group. The project was large enough
to accommodate 1200 fully-furnished apartments of different sizes spread out in 15 towers; each tower having
80 apartments. In addition, a Golf Course and a three storeyed mall was also to be developed. Th is project
was to come up near Shankar pally Road, Hyderabad, a non-polluting and posh area having all the facilities
in close vicinity including ultra-modern cinema halls, markets, schools, colleges, hospitals, etc.
As mentioned above, the Raj Group which undertook to develop Royal-Golf-Burg consisted of Dhanraj and
his younger brother Yuvraj, a well-known figure of Hyderabad. Both the brothers were the directors of Eklavya
Estates Private Limited (EEPL) which had registered office at Gachibowli, Hyderabad. EEPL owned the plot
of land where the proposed housing project including mall was to be developed.
The gated residency with a nice and peaceful environment as provided by Royal Golf Burg was meant for
golf lovers who wanted to live a sleek and sporty lifestyle by making golf playing a routine. The glamour which
attracted Mr. Reddy the most was that every apartment owner after occupation would feel the ownership of
the golf course due to its strategic location vis-à-vis each apartment.
The management team of EEPL comprised seasoned architects and professionals who had, in the past, made
luxurious homes possible for every home-buyer and the team was considered to be a dedicated one having
will and honesty as its strong pillars which could build integrated properties with excellent infrastructure and
services. The builders had completed several giant opulent projects in Guwahati, Mumbai and Bhopal earlier.
In case of Royal Golf Burg, the company was to give delivery of fully furnished apartments by December,
2017. Construction cost including the cost of the land was valued at around Rs. 800 crores.
The current project, however, missed the deadline of December, 2017 and on the date of delivery, it was
noticed that only six towers were completed; but the apartments in those towers were yet to be furnished.
Other three towers had been constructed with a skeletal structure. In other six towers, only the foundation
and a negligible wall work had been completed. In other words, the construction work was just at the initial
stage and nothing more than that. However, the construction of Mall was almost complete. This angered the
home-buyers including Mr. Reddy a lot but their repeated visits to the office of promoters did not evo ke any
positive response. Six months passed without any significant happening. Nothing was done to furnish the
already constructed apartments or to develop the other nine towers. Disappointed, Mr. Reddy and others
approached Telangana State RERA authorities for redressal of their grievances including filing of complaints
regarding non-delivery of apartments.
Telangana State RERA, after detailed inquiry, found that there were several financial irregularities together
with diversion and siphoning of funds. More than two hundred shell companies were floated in the names of
peons and drivers to divert money. Further, unaccounted money worth crores of rupees was invested in
various other housing projects floated by the Raj Group which sold these flats at throw-away prices on paper
but received black money in cash which was laundered through various shell companies operated by the Raj
Group. This attracted the provisions of Prevention of Money Laundering Act, 2002.

© The Institute of Chartered Accountants of India


In case of 250 apartments built in the six towers, double allotment was also detected where the apartments
were allotted to the persons more than their entitlement at a very nominal amount. It included the person
himself, the spouse or dependent children; and almost in all cases such persons were found to be connected
with the promoters. The double allotment deprived the genuine home-buyers who had parted with their hard-
earned money from getting even the deserving allotment of apartments despite paying around 80% of the
cost. The double allotment was considered to be a form of unfair practice in which the promoters were
involved. Enquiry by the RERA Authority also revealed that ten gardeners and drivers of Raj Group who had
no means of paying the price of ten apartments were allotted the flats though consi deration came from the
Raj Group itself.
A show-cause notice was issued by Telangana State RERA authorities to the developers asking them to
provide a satisfactory response within a period of 30 days from the date of the notice as to why the project
should not be de-registered. The response given by the directors, however, was dismal, lacked substance
and was not at all satisfactory.
However, de-registration of a building project is not an ideal choice for the authorities keeping in view the
larger interests of the stakeholders as well as the nation as a whole and it is resorted to only when all the
possible avenues of reaching a comfortable and plausible solution are shut. Therefore, as a last attempt
before going for de-registration, RERA authorities in the interest of the allottees, permitted the developer
EEPL to continue with the project and complete it in the next one year at the most subject, however, to the
payment of a fine equivalent to 10% of Rs.800 crores within next thirty days.
However, the developers, seemed to be not serious at all so far as completion of housing project was
concerned. They did not make use of this golden opportunity; thus, letting the project slip out of their hands.
Citing insufficiency of the funds, they did not cough out the required fine of Rs. 80 crores within next thirty
days and therefore, the Telangana State RERA authorities were forced to de-register the project and an order
to this effect was passed. Thereafter, finally the project was taken over by RERA Authorities.
After take-over, RERA authorities with the concurrence of the State Government imposed various restrictions
and controls on the project and the developer. These included:
• Freezing of various bank accounts due to which the developer was not allowed to make any payment or
withdraw from these accounts without the authority’s approval.
• Debarring the developer EEPL from accessing the web-site of RERA in relation to the project.
• RERA offices in other States and Union Territories were given information about such a revocation.
• As a part of name and shame, the name of the EEPL was mentioned in the list of the defaulters along
with the photographs of Dhanraj and Yuvraj and also relevant information about the case was displayed.
The officials of Telangana State RERA opined that after de-registration, there were several options before
them to solve the issue in favour of home-buyers. The authority could give the first right of completion of the
project to the home-buyers. If case the buyers were not in a position to do so by pooling their resources
together and required RERA to supervise the development work which could be undertaken by another trust -
worthy developer, then the RERA Authority could take steps to develop mechanism to supervise the project.
In case there was not enough money left in the project fund, the Authority could also start proceedings to
recover the diverted funds from the EEPL and it could also explore other possibilities to complete the project
if the home-buyers so wished.
The response from the Home-buyers’ Association of Royal Golf Burg was positive and therefore, a conciliatory
committee was formed. President and Secretary of the Home-buyers’ Association were nominated to the
committee and RERA then appointed Mr. Yudhister Pal, a retired IAS Officer as a conciliator to supervise the
operations of the committee. Another developer Uttam Constructions Private Limited was given the charge
to complete the project within one year under the supervision of RERA Authority represented by
Mr. Yudhishtra Pal.

© The Institute of Chartered Accountants of India


RERA ordered that all money realised from the sale of Mall as well as remaining dues to be given by the
home-buyers would flow into an ‘Escrow Account’ opened solely for the construction of the project and Mr.
Yudhister would release the funds only with the consent of the President and Secretary of Home-buyers’
Association. Proceedings to recover the diverted funds from the EEPL were also started. It was hoped that
the project would be completed as per the new schedule.
Multiple Choice Questions (MCQs) [2 Marks each]
1. RERA Authority passed the order of de-registration of Royal Golf Burg Project against its promoter
EEPL. Within how many days of receipt of order of de- registration, EEPL, as aggrieved party, can file
an appeal with the concerned Real Estate Appellate Tribunal?
(a) 30 days
(b) 45 days
(c) 60 days
(d) 120 days
2. The promoter EEPL did not complete the Royal Golf Burg Project within the projected time -frame as
shared through declaration with RERA Authorities while seeking registration under Section 4. For such
contravention, how much penalty the EEPL is liable to pay?
(a) Penalty may extend up to 2% of the project cost
(b) Penalty may extend up to 5% of the project cost
(c) Penalty may extend up to 10% of the project cost
(d) None of the above
3. In case of Royal Golf Burg project, it is seen that it was de-registered by the RERA Authorities due to
various irregularities. Choose from the given options as to who shall have the first right of refusal for
carrying out the remaining development works in case of such revocation of registration.
(a) Home-buyers’ Association of Royal Burg Golf
(b) EEPL
(c) Mr. Yudhister Pal, Head of Conciliatory Committee
(d) RERA Authority
4. In the above case study relating to Royal Golf Burg, who is Benamidar?
(a) Gardeners and drivers employed by Dhanraj and Yuvraj
(b) Dhanraj and Yuvraj themselves
(c) Hariharan Reddy
(d) None of the above
5. In case the Real Estate Appellate Tribunal admits the appeal of EEPL against de-registration of the
Project, then within how much maximum time such appeal must be disposed of?
(a) Within 30 days from the receipt of appeal
(b) Within 45 days from the receipt of appeal
(c) Within 60 days from the receipt of appeal
(d) Within 120 days from the receipt of appeal
Descriptive Questions
1. According to the above case study, the Royal Golf Burg promoted by the Raj Group was de -registered
by the Telangana State RERA authorities because the promoters were found to be involved in certain
3

© The Institute of Chartered Accountants of India


unfair and fraudulent practices. You are required to state the various reasons due to which registration
granted to a project under RERA can be revoked and the project stands de-registered. (5 Marks)
2. In the given case study, the Telangana State RERA authorities resorted to de-registration of the Royal
Golf Burg Project due to unfair and fraudulent practices and irregularities followed by its promoters while
developing the project and the development was carried out at such a slow pace that ultimately the
home-buyers could not get the possession of fully furnished apartments well within the promised time.
What are the obligations of the RERA Authority and other matters associated with it if it recourses to
de-registration of a project? (5 Marks)
3. The circumstances stated in the above case study require the RERA Authority to revoke the registration
of Royal Golf Burg Project instead of its extension. State the provisions under which RERA Authority
may be required to extend the registration instead of revoking it. (5 Marks)
Case study 2
In northern and western part of the country production of sugarcane is reasonably good. But the large amount
of pendency of the payments by sugar mills to sugarcane producers is cause of worry. Common platform is
essential requirement to provide solution to this problem. Northwest Agro Produce Cooperative Society
was formed to ensure the timely collection of sale proceeds from sugar mills. Northwest Agro Produce
Cooperative Society developed a charter, in form of memorandum for its members, to regulate and control
supply, price, term of sales of sugar canes, collection of sale proceed and recovery if required. This
memorandum is binding on all the members of society.
Northwest Agro Produce Cooperative Society extend the support to cane grower, by given them offer; to sell
their entire farm produce of canes to society at mutually agreed price; which society will further sale to sugar
mills. But farmer who avail this facility have to sell his entire farm produce to Northwest Agro Produce
Cooperative Society, means farmer can’t sale any portion of his farm produce in open market. In order to
trade with the sugar mills, and deals with regulatory authorities, financial institution etc; Northwest Agro
Produce Cooperative Society decided to promote Limited Liability Company named North West Agro
Limited. The extracts from latest audited financial statements of North West Agro Limited are as follows;
Sr. No. Particular Amount (In Lakhs INRs)
1 Proceed (Net of taxes) from sale of sugar canes 320000
2 Operating assets 72800
3 Paid –up share capital 48900
4 Net Profit 9600
With Passage of time North West Agro Limited became the big hit, for role it play as intermediary; in incredible
transformation in process of sale of sugarcane by cane farmers.
Mr. Vijendra Narang, who is CEO of North West Agro limited, heard about forward integration as method of
expansion and growth strategy. Mr. Narang prepared a proposal, which is duly approved by board of directors
and then by members of company to takeover Sun Sugar Limited, by acquiring controlling stake from open
market. Sun Sugar Limited is running sugar mills, with global presence.
Around 60% of sales by Sun Sugar Limited constitute exports of raw sugar, majorly to IRAN. One year back
Sun Sugar Limited opened one branch office in IRAN, as IRAN starts buying sugar from India, in order to
settle trade balance; because IRAN is blocked from the global financial system; including using U.S. dollars
to transact its oil sales. On such branch office, during last financial year, annual recurring expenditure in
foreign currency out of EEFC accounts; was equivalent to INRs 14000 lakhs.
For last financial year, turnover of Sun Sugar Limited was recorded at INRs 120000 lakhs which was
INRs 10000 lakhs more than year earlier to last financial year; whereas operating assets as on reporting date
were INRs 27000 lakhs. Paid–up share capital was INR 12600 lakhs. After acquisition both the entities were
not merged, both kept respective separate identity.
4

© The Institute of Chartered Accountants of India


Sun Sugar Limited has strong domestic network or tie-up with retail shops and stores through which they sale
their sugar, under brand name 'Meetha', which constitute around 40% of sale. Such retail shops and stores
are provided with instruction not to charge the price more then what is suggested by Sun Sugar Limited
although lower prices can be charged and specific jurisdiction is given to each retailer for resale.
Mr. Nair who is head of marketing at North West Agro Limited, also look after marketing at Sun Sugar Limited,
according to him; in order to acquire substantial market share (in term of new customers), Sun Sugar Limited
has to sell sugar at the prices lower than cost. Ignoring the resistance from the governing body of Sun Sugar
Limited, new pricing policy implemented. Resultantly price decreased from INRs 40 per kilogram to INRs 35
per kilogram. But in order to restrict loss, on account of selling sugar at price lower than cost; Sun Sugar
Limited ask to all the shopkeeper and stores, through whose counter they are sale their su gar produce, not
to bill more than 2 kg of ‘Meetha’ sugar per purchase
Northwest Agro Produce Cooperative Society promotes another company named South West Agro Limited,
whose object clause includes; provide weather research and forecast reports, other nec essary technical
knowledge or guidance to members of parents society apart from conducting market research for North West
Agro Limited.
Out market research conducted by South West Agro Limited, it was found that Moon Sugar Limited, hold
major stake in retail of packed sugar, around 30% across the nation under brand name ‘Aur’ (Price of which
is INRs 40 per Kilogram); which cause stiff competition among the players who sell packed sugar. Since
acquisition of Sun Sugar Limited by North West Agro Limited, remains largely successful; hence showing
trust in un-organic growth, a bear-hug letter sent to senior management of Moon Sugar Limited. For latest
financial year, turnover of Moon Sugar Limited is recorded at INRs 280000 lakhs where as operating assets
are of INRs 56800 lakhs. Paid–up share capital is INR 36400 lakhs.
Since Moon Sugar Limited is already undisputed market leader hence refuse the bear hug offer. North West
Agro Limited with help of South West Agro Limited performs hostile acquisition and both th e companies
acquires around 25.5% stake in voting rights each; by tender notice over the stock exchange. Governing body
of Moon Sugar Limited restructured completely. Post acquisitions of Moon Sugar Limited, North West Agro
Limited got the dominance over the market. Since new pricing policy introduced principle buyer of North West
Agro Limited is multifold. Hence company decided to re-price their product, which is renamed also ‘Aur
Meetha’. New price is INRs 42 per Kilogram. To support the price rise, North West Agro Limited starts
restricting supply.
North West Agro Limited also entered in memorandum of understanding with Star Ethanol Limited, which is
$ 20 million (assets base) company for transfer of technology.
Multiple Choice Questions (MCQs) [2 Marks each]
1. Takeover (acquisition) of Sun Sugar limited by North West Sugar limited, will be considered as
combination if
(a) Assets of enterprise created after merger is equal than INRs 1000 crores
(b) Turnover of enterprise created after merger is more than INRs 1000 crores
(c) Turnover of enterprise created after merger is more than INRs 3000 crores
(d) Assets of enterprise created after merger is more than INRs 3000 crores
2. South West Agro Limited and North West Agro Limited will be considered as group because, these are
in capacity of
(a) Exercise 26% or more of the voting right of Moon Sugar Limited
(b) Appoint more than 50% of members of board of directors in the sun limited
(c) Control the management or affairs of the sun Limited
(d) All of above
5

© The Institute of Chartered Accountants of India


3. Exchange Earners' Foreign Currency Account can be open by foreign exchange earner;
(a) Who are not resident in India
(b) Who are resident in India
(c) Which are situated in SEZ
(d) Person of Indian Origin, but residing outside India.
4. Decision of North West Agro Limited, on part of Sun Sugar Limited; not to sell more than 2 kg of sugar
per purchase can be categorized as;
(a) Exclusive supply agreement
(b) Exclusive distribution agreement
(c) Refusal to deal
(d) None of the above
5. Exchange Earners' Foreign Currency Account can be open with
(a) Authorised Dealer - Category I
(b) Authorised Dealer - Category II
(c) Authorised Dealer - Category III
(d) Full Fledged Money Changers
Descriptive Questions
1. Is Northwest Agro Produce Cooperative Society can be considered as ‘Cartel’. (3 Marks)
2. Does North West Agro Limited hold dominance over market, if yes at what instances you feel it abuse
its dominant position? (4 Marks)
3. Explain briefly regulatory aspects of combination in respect in light of case for North West Agro Limited
and regulation thereof under Competition Act, 2002. (5 Marks)
4. Mention provisions regarding expenditure on ‘maintaining office abroad’ under FEMA, 1999 & related
rules. Is expenditure done by Sun Sugar Limited is in violation thereto. (3 Marks)
Case study 3
APPRAY is the Karta of a Hindu Undivided family (HUF) also consisting of his wife LAXMI DEVI, 3 sons,
SUBHASH, GIRISH and RAJESH. The eldest son SUBHASH runs a Sugar Mill taken over from his father
APPRAY.
RAJESH, the third son of APPRAY, always feels ignored by his family, looking for some fast easy money,
joins hands with MOHANLAL, who is a Real Estate Agent, who promises to pay RAJESH, a commission in
cash if he helps MOHANLAL to buy 25 Acres of Land and hold the land in his name on behalf of one of his
customers MANORANJAN in good trust and in good faith. RAJESH agrees and a Purchase Agreement for
25 Acres of Land was registered in the name of RAJESH and one MADHAV RAO . Subsequently, Rajesh
entered into several similar agreements in his name on behalf of others.
In due course of time, RAJESH also formed a Company XYZ Pvt Ltd, primarily for a Hotel business, but the
source of funding was secret drug dealings. The Company accepted illegal monies in cash as legitimate
business transactions with fake income and receipts. The monies were then deposited into the Company’s
Bank accounts as clean money. He kept fraudulent records, which did not demonstrate the current state of
his businesses. Monies in the Bank Accounts of XYZ Pvt Ltd were also often transferred as legitimate
business transactions, to the Bank Accounts of RDX Pvt Ltd, which is also in the similar businesses like XYZ
Pvt Ltd. Original source of money is thus disguised.

© The Institute of Chartered Accountants of India


The Company XYZ Pvt Ltd also mobilized funds from various investors, but were never utilized for which they
were collected. The Funds were transferred to bank accounts of some group companies, which were mainly
paper companies, from where they were systematically siphoned off and were used for purchase of various
properties in India.
RAJESH has also held some properties purchased in the name of his wife SUGUNA from his known income
from legal sources.
MAHESH is a friend of GIRISH, the Second son of APPRAY is a Company Secretary of a listed Public Limited
Company ABC Ltd. MAHESH gives Rs. 5 lacs loan to GIRISH, who in his turn gives loan of Rs. 5 Lacs to his
friend RAGHU for investment in the shares of ABC Ltd. RAGHU trades in shares of ABC Ltd on behalf of
MAHESH.
MAHESH also ensures that some money is passed on to various legitimate Companies to buy the shares of
ABC Ltd so that it results in increase in the price of shares. Intention is to show higher valuation of shares
before proposing to the investors or to discourage the shareholders from applying to the buyback scheme.
RAGHAV is the brother in law of SUBHASH, employed in UAE and a non resident Indian. RAGHAV purchased
some properties in Mumbai for Rs. 75 Lacs. He paid RS. 40 Lacs through his NRE Account, Rs. 10 Lacs
through direct transfer from his salaries account in UAE to the sellers account as advance through normal
banking channels, complying with all the procedural requirements, but balance Rs. 25 Lacs payment was
made though some unknown sources.
RAGHAV also invested in Equity shares of various Listed Companies in India in the name of his wife DIVYA,
who is a Resident in India and himself as joint holders from an account not disclosed to tax authorities in
India. RAGHAV also purchased a Flat in Mumbai in the name of DIVYA and himself as joint holders from his
NRE Account.
SUBHASH has a married daughter MANGALA, who is a UK resident. SUBHASH invested Rs. 1.50 Crores in
a Bank Fixed deposit in the name of MANGALA without her knowledge. Later during the course of enquiries
by Tax officials MANAGALA denies ownership of Bank Fixed Deposit.
Since all of his children are well settled, due to the old age and deteriorating health conditions of APPRAY
and LAXMI DEVI, the family decided to sell off the loss making Sugar Mill. Later after much negotiations, the
Sugar Mill was sold to a person well known to the real estate agent MOHANLAL, but unknown to the APPRAY
Family, at a reasonable price.
Multiple Choice questions (MCQs) [2 Marks each]
1. Purchase of properties in Mumbai by RAGHAV for Rs. 75 Lacs:
(a) Is a Fully valid transaction
(b) Is valid to the extent of Rs.40 lacs
(c) Fully invalid transaction and to be considered as “Benami”
(d) May be Benami to the extent of Rs. 25 lacs, since through some unknown sources.
2. Which one of the following transaction is NOT Benami done by RAJESH?
(a) Transaction in respect of a property, where the person providing the consideration to Rajesh is not
traceable.
(b) An arrangement by Rajesh in respect of a property made in a fictitious name.
(c) Property held by Rajesh in the name of his spouse and consideration paid out of known legal
sources.
(d) A transaction by Rajesh in respect of a property where the owner is unaware of or denies
knowledge of the ownership.

© The Institute of Chartered Accountants of India


3. Share Trading by Raghu on behalf of Mahesh is:
(a) Valid transaction since he is not at all connected with ABC Ltd.
(b) Can be proved as Benami trading in stock markets by Mahesh, the Company Secretary, who has
insider price sensitive information.
(c) The transaction is not at all to be considered as Benami.
(d) Valid transaction if Girish does the share trading on behalf of Mahesh, out of the loan of Rs. 5 Lacs
given by Mahesh
4. Which one among the following statement is correct as per PBPT Act?
(a) Resale of the benami property from Rajesh to one of the real owners is a valid transaction.
(b) Resale of the Benami property from Rajesh to a person acting on behalf of real owner is a valid
transaction
(c) The Benami Act prohibits sale of a benami property by Rajesh to a third person
(d) PBPT Act prohibits resale of the Benami property from the Benamidar to the real owner or to any
person acting on his behalf. Such transactions would be considered as null and void.
5. In respect of transactions done by XYZ Company above, crime money injec ted into the formal financial
system is layered, moved or spread over various transactions in different accounts. This step in money
laundering is referred to as:
(a) Smurfing
(b) Integration
(c) Layering
(d) Placement
Descriptive questions
1. In the context of various property dealings in the above case study, critically analyze the statement “the
provisions of the Prohibition of Benami Property Transactions Act, 1988 (PBPT Act) need not
necessarily applicable only to persons, who try to hide their properties, but may also sometimes apply
to genuine properties acquired out of disclosed funds” (5 Marks)
2. In case of confiscation of properties of Rajesh in the above example, how “Fair market value” in relation
to a property is defined under Sec 2(16) of the PBPT Act? Also interpret the Rule 3 of Prohibition of
Benami Transactions Rules, 2016 when the price is not ascertainable? (5 Marks)
3. With reference to various transactions by XYZ Co Private Ltd in the above example, what are the
provisions relating to attachment of property involved in money laundering under Section 5 of the
Prevention of Money laundering Act 2002? (5 Marks)
Case study 4
Alpha Promoter is the promoter of real estate based at New Delhi. The Alpha Promoter proposed to establish
the residential project named “AASHIYANA” The project plan constituted 50 apartment mix of 3BHK and
2BHK Apartments. It includes 20 3BHK Apartments and 30 2BHK Apartments. The Residential project needs
to be registered under RERA. The residential project is based at East Delhi. The Alpha’s son a Qualified
Chartered Accountant and a RERA Consultant. Mr. Alpha discuss the proposed projects with his son usually
once in the month. Mr. Alpha discussed with his son CA Surendar about the newly established project
“AASHIYANA” based at East Delhi. The Father Mr. Alpha disc uss the applicability of various provisions on
the new project “AASHIYANA” based at East Delhi with his son CA Surendar under the Delhi Real Estate
(Regulation & Development) Act, 2016 and rules made thereto. The father inquired about the prior approval
before establishment of new project “AASHIYANA”, if any required by Delhi Real Estate (Regulation &

© The Institute of Chartered Accountants of India


Development) Act, 2016. Further Mr. Alpha inquired about the creation of webpage of the Alpha Promoter on
the website of DELHI RERA Estate (Regulation & Development) Act, 2016, after getting Login ID and
Password.
Also, Mr. Alpha want to publish the advertisement for the residential project “AASHIYANA” for which he
wants to know the content. The advertisement specifies the condition of advance payment first wi thout
entering into agreement for sale. The advance payment (deposit) shall not be less than 15% of the cost of
apartment. Only after payment of such advance of 15% of the cost of the apartment, a promoter will enter
into agreement for sale with allottee of the particular apartment. The promoter made the policy for non-
disclosure of any stage-wise time schedule of the completion of the project, including the provisions for civic
infrastructure like water sanitation & electricity at the time of booking and issue of allotment letter to allottee.
The promoter forgot to include any terms for cancellation of allotment in the agreement of sale. The
construction started and afterwards the promoter make some major alterations in the sanctioned plans &
layout plans as well as nature of fittings without the previous approval from allottees. One of the allottee Mr.
Abhay ask for minor changes in his allotted apartment which was further also approved by authorised architect
and proper declaration and intimation is also made regarding such minor change to the allottee (Mr. Abhay).
Meanwhile, presently the Alpha Promoter intended to transfer the project “AASHIYANA” to Beta Promoter
(third party) without obtaining any approval.
Issue Raised
• Can Alpha Promoter can transfer the project to the Beta Promoter suo moto?
• Is Alpha Promoter mandatorily required to maintain webpage on the website of DELHI RERA
AUTHORITY?
• What are requirements for registration of new residential project “AASHIYANA”?
• Can advertisement published with or without mentioning website address?
• Is any possibility of holding right as promoter of not providing any details on stage-wise time schedule
of completion of the project at the time of booking to the allottee?
• Is promoter having right to demand more than 15% of the cost of apartment before entering into
agreement for sale with the allottee.
Few apartments are purchased in the name of Foreign National (Non-Residents) and few are purchased in
joint name of both Indian National (Resident) and Foreign National (non-residents) in light of the provisions
of the Prohibition of Benami Property Transactions Act, 1988, the following transactions took place for
purchase of Apartment under project “AASHIYANA”. The description of such transaction are as follows:
• Mr. X, a non resident in India purchased residential apartment of “AASHIYANA” residential project
under RERA for INR 50,00,000. The apartment is purchased in the joint name of Mr. X and his sister
but the payment is made from unknown source.
• Mr. Y, a non resident in India purchased residential apartment of “AASHIYANA” residential project
under RERA for INR 60,00,000 in the name of his wife from his NRE Account.
• Mr. Z, a resident in India sold a apartment in India (second hand apartm ent) of some other RERA
registered residential project for INR 50,00,000 which was not in his name and was in unknown person
name. But the sale proceed is deposited in his (Mr. Z) personal bank account.
• Mr. P, a non resident in India purchased residential apartment of “AASHIYANA” residential project
under RERA for INR 50,00,000. The payment is made in parts INR 20,00,000 by P’s NRE account,
INR 29,50,000 from unknown sources and INR 50,000 in cash. The registry was done at value of
INR 45,00,000 which was paid by own cheque (INR 20,00,000) and from unknown sources jointly
(INR 25,00,000).

© The Institute of Chartered Accountants of India


Based on the above mentioned facts of the case and issues raised you are requested to give professional
advice as Corporate Consultant to the Alpha Promoter and allottees of the residential apartment on the below-
mentioned questions:
Multiple Choice Questions (MCQs) [2 Marks each]
1. The Alpha Promoters shall not transfer or assign his majority rights & liabilities in respect of a real estate
project to a third party without obtaining:
(a) Prior written consent from two-third allottees;
(b) Prior written consent from two-third allottees; except the promoters
(c) Prior written consent from two-third allottees; except the promoters and prior written approval of
the Authority.
(d) Prior written consent from two-third allottees; except the promoters and prior written approval of
the Authority. However, such transfer or assignment shall not affect the allotment or sale of the
apartments.
2. The Alpha Promoter needs to get registration of the new project “AASHIYANA” through the
operationalise web based online system for submitting application for registration of the project within a
period of ____________________.
(a) One year from the date of its establishment.
(b) One year from the date of its commencement.
(c) One year from the date of its initiation.
(d) One year from the date of its starting.
3. The promoter Alpha at the time of the booking and issue of allotment letter to the allottee shall be
responsible to make available to the allottee the _________.
(a) Sanctioned Plans
(b) Layout Plans
(c) the stage-wise time schedule of completion of the project.
(d) All of the above.
4. The Alpha Promoter shall be responsible for providing and maintaining the essential services, on
reasonable charges, till the taking over of the maintenance of the project “AASHIYANA” by
_____________________.
(a) The allottees;
(b) The third party;
(c) The association of the allottees;
(d) None of the above.
5. Mr. Z, a resident in India sold a apartment in India (second hand apartment) of some other RERA
registered residential project for INR 50,00,000 which was not in his name and was in unknown person
name. But the sale proceed is deposited in Mr. Z’s personal bank account. The given transaction is
___________________ as per the provisions of Prohibition of Benami Property Transactions Act, 1988.
(a) Benami Transaction.
(b) Not a Benami Transaction
(c) May be a Benami Transaction.

10

© The Institute of Chartered Accountants of India


(d) May not be a Benami Transaction
Descriptive Questions
1. In Light of the provisions of the Section 11 of the Real Estate (Regulation & Development) Act, 2016,
the Alpha Promoter upon receiving Login ID and password from RERA Authority create webpage on the
Website of the authority including all details of the proposed project for public viewing.
Describe in detail what information are required to be disclosed for public view on the webpage of
promoter on the Website of the RERA Authority? (5 Marks)
2. Under the light of the provisions of “Prohibition of Benami Property Transactions Act, 1988”. Examine
whether the below mentioned transaction is considered as the Benami Transactions or not?
Mr. P, a non resident in India purchased residential apartment of “AASHIYANA” residential project under
RERA for INR 50,00,000. The payment is made in parts INR 20,00,000 by P’s NRE account,
INR 29,50,000 from unknown sources and INR 50,000 in cash. The registry was done at value of
INR 45,00,000 which was paid by own cheque (INR 20,00,000) and from unknown sources jointly
(INR 25,00,000). (5 Marks)
3. Examine the following:
(i) Alpha Promoter’s responsibility when the Real estate Project “AASHIYANA” is developed on a
leasehold land. (2 Marks)
(ii) Alpha Promoter can cancel the allotment whether or not such terms of cancellation are included in
the executed agreement of sale. (3 Marks)
Case study 5
Mr Suresh Agarwal, is a Jaipur based banker who joined the Samradhi National bank in the year 2008 as a
clerk. He got his first posting at Jodhpur branch. After five years in job, he was transferred and promoted as
assistant manager, Bikaner branch of Samardhi National bank. One day he met Archana in the bank, who
visited bank to discuss some loan issues of her company with the bank. Slowly they started meeting each
other. Both of them like each other and decided to marry after sometime. Archana is a director in a Newtech
Software company. They decided to get married after one year. With the blessing of their parents they got
married in 2013.
After marriage Archana need to visit America for some business meeting of the company with foreign client.
So, they both planned a tour parallelly with the business trip. Company issued Prepaid Forex Travel Card to
Archana for all his expenses abroad. The card is loaded with amount of USD 2,50,000. After two days of
prolonged business meetings, Archana finally got free to enjoy trip with her husband. They went to Los Vegas
and played some games in casino. They paid USD 2000 with the Forex card. They hardly won any amount
out of it. They enjoyed shopping and had all fun. After 10 days they return back to India. Archana returned
the business card to the company with remaining USD 100000.
Back to daily routine they both got busy with their respective work schedule. As an assistant manager,
Mr Suresh got gifts in form of cash and kind all the year round. He came in contact with many rich and
successful business owners from different walks of life. They all were happy with him as mostly they got their
work done on a single phone call. In the year 2016 post Demonetization, Mr Suresh helped many of his bank
customers to launder their money through their bank accounts. He helped one of the bullion trader, Mr Naval
Kishore, account holder in his bank. He suggested him to take money from his customers and issue cheque
back for the same amount, less 5 per cent. The bullion dealer gave them a purchase bill to show that he had
purchased his gold jewellery or silver utensils from him. Than On the amount of the cheque when they will file
their return, the customer will have to pay no capital gains tax as jewellery and silver utensils are of personal
use, and capital gains do not arise on sale of personal things.

11

© The Institute of Chartered Accountants of India


Mr. Naval Kishore with some agents, in past use to trade illegal gold through illicit shell or front companies
using false or incomplete documents. The gold is often then smuggled through third -party countries and
ultimately sold to refineries in the United States.
Mr Suresh offers to exchange demonetized banknotes for a charge of 10 per cent. Much of this was managed
by using scores of individual PAN and Aadhar details already available with his bank. Mr Suresh helped Mr
A.K. Bajaj, to exchange demonetized cash, through his workers ID's at the bank. Mr. Bajaj employed nearly
200 workers of his manufacturing units to get the demonetized notes exchanged. The Demonetization period
became a blessing in disguise for Mr. Suresh as he was able to amass huge profits out of it.
Chetan singh is a director of XYZ Company Ltd. He use to share a good relation with Mr. Agrawal and he
also started giving him attention when he visited bank for day today business. Chetan one day came to Mr .
Agarwal to sanction him INR 0.50 crores. The documents required to process the loan was incomplete. The
branch manager refused to sanction the loan without completing all the formalities. Chetan had a talk with
Mr. Agarwal and promised him to pay 5% of total sanctioned loan amount. So, both of them arranged some
forged documents to complete the file with help of one person called Piyush Sharma, an accountant friend of
Mr. Agarwal. After the completion of all the formalities Mr. Agarwal gave his clearance regarding completion
of the documents, after which his branch manager sanctioned the loan. The deal bought all of them close to
each other. After that Mr Agarwal, Chetan and Piyush became good friends.
As Piyush was accountant and an employee in financial advising company he used to meet many people. He
got some couple of loans sanctioned with the help of Mr Suresh Agarwal. As a result, on performance basis
evaluation Mr Agarwal got the promotion as Bank manager and transferred to the other branch of Bikaner.
Archana in 2016, was appointed as a director, in Zippy International Company Ltd, a foreign based Company.
For Couple of months she stayed in Italy for board meeting and giving financial advice regarding the business
transactions. In lieu of his service and remuneration she received shares of the company worth USD 3,00,000.
During his period of service in bank, Mr Agarwal was able to accumulate INR 20 lakhs from his unauthorized
sources. With this money he bought a plot in his home town Jaipur. The cost of the plot is INR 50 lakhs. To
cover up his source of income he took a loan of INR 20 lakh from his bank. Likewise, he invested his income
got in his mother's name and got the plot registered.
In 2017, Archana again went to London for company related business. She stayed there for a duration of six
months. With a view to invest abroad, she bought a flat their through outwards remittance. She remitted USD
2,49,000 via LRS route.
Mr Agarwal and his wife Archana went on a tour to Qatar and Dubai in 2018.They booked the flight tickets
online through their credit card, rest all the expense of logging and boarding was borne by Oyster tour and
travel agency. The company recently opened his new branch office in Dubai. The company's AD b ank is the
bank in which Mr. Suresh is branch manager. He made all the transactions of Oyster tour and travels easy.
Whenever Exchange is required by the branch office in Dubai he easily released them. The profit earned by
Dubai branch, of Oyster tour and travel was invested, in one company of Dubai engaged into real estate
business and its profits is deposited in foreign bank.
Mr Chetan has a planned to earn money through a new export and import business. He submitted a fake
factory proposal of garment manufacturing to the bank whose branch manager is Mr Agarwal. Mr Chetan had
a plot in the outskirts of the city, where he made some construction to give it a factory look. As per the mutual
understanding between both of them some loan of INR 2 crores was sanc tioned to ship the machines and
other products from abroad. Some fake invoices were prepared to show the dispatched of garment orders on
record but sent almost nothing in return. Mr Agarwal and Mr Chetan jointly did all the invoice manipulation.
As a result siphoning off of foreign exchange against fictitious and bogus imports. The money was invested
abroad in mutual funds and buying properties.

12

© The Institute of Chartered Accountants of India


Multiple choice Questions (MCQs) [2 Marks each]
1. Archana went to Italy regarding his official visits as a company director. She bought a flat in Italy through
outwards remittance. Can she continue to hold immovable property outside India which was acquired
by her when he was a non-resident?
(a) With the permission of Reserve bank of India she can hold the property abroad.
(b) Without the permission of Reserve Bank of India she can hold the property abroad.
(c) The property needs to be sold and the funds transferred should be repatriate back to India.
(d) The property needs to be sold out and transferred should be repatriate to her FCNR account.
2. Let's assumed that the plot is bought by Mr Suresh from his known sourced income in his mother's
name. Mr. Tarun an NRI wants to purchase that plot. State your answer.
(a) Mr. Tarun can buy the plot only with permission of RBI.
(b) Mr. Tarun can buy the plot through normal banking channel of India without RBI permission.
(c) Mr. Tarun can purchase the plot in foreign currency with permission of RBI.
(d) Mr. Tarun can make part payment in foreign currency and partly through his NRO account in India.
3. Archana was allotted USD 300000 shares in the foreign entity in lieu of the professional services
rendered by her in lieu of Director’s remuneration. Can she acquire such shares?
(a) Archana can acquire USD 300000 shares as it is paid against her remuneration.
(b) Archana can acquire USD 300000 shares as her remuneration with permission of RBI.
(c) Archana cannot acquire USD 3,00000 shares as it is not in permissible limit of LRS.
(d) Archana can acquire USD 3,00000 shares under LRS but needs to repatriate the profits back to
India after sale of it.
4. Has Suresh and Archana contravene any provision of law, while they sent USD 2000 playing at the
casino? Choose the correct answer.
(a) They did not contravene any provision of the law to as it was issued to her in official capacity for
her trip.
(b) They have used the card in violates the provisions of FEMA act and hence liable under it.
(c) They did not contravene any provision of the law as it was within the permissible limit of their
expenses.
(d) They were only liable under FEMA Act if they had won more than USD 2000 and if it is remitted
back to India.
5. In post Demonetization period Mr. Suresh helped many of his customers to exchange demonetized
currency through various methods. Is he liable for punishment under any provision of the law in India?
(a) Mr. Suresh is not liable for any punishment under any provision of law applicable in India.
(b) Mr. Suresh is liable to punishment under Part A of the schedule, of prevention of Money Laundering
Act.
(c) Mr. Suresh is only liable to be punished under Part C of the Schedule, of Money Laundering Act.
(d) Mr. Suresh is liable for punishment under Part A, PARAGRAPH 1 and PARAGRAPH 4 of Money
Laundering Act.
Descriptive Questions.
1. Advise in the above mentioned two situations as per the relevant applicable law in India?

13

© The Institute of Chartered Accountants of India


(i) Assuming Mr. Suresh wanted to further lease the plot, he bought in his mother name, to a foreign
entity who wants to open his liaison office in India.
(ii) Can Mr. Suresh Agarwal mother's gift this plot to his niece, an NRI. (8 Marks)
2. Examine the given situations in the light of the PBPT Act:
(i) Explain why the property brought by Mr Agarwal in his mother name is a 'Benami transaction'?
(3 Marks)
(ii) If in case a fraudulent transfer is done by Mr Suresh Agarwal, pending the process of inquiry by
the Initiating Officer, what will be the consequences? (2 Marks)
(iii) State what will the effect if property (the plot in the name of Mr Agarwal's mother name) is
confiscated by Initiating Officer? (2 Marks)

14

© The Institute of Chartered Accountants of India


Test Series: October, 2019

MOCK TEST PAPER 1


FINAL (NEW) COURSE: GROUP – II
PAPER – 6D: ECONOMIC LAWS
Suggested Answer
Case Study 1
Multiple Choice Questions (MCQs)
1. (c)
2. (b)
3. (a)
4. (a)
5. (c)
Descriptive Questions
Answer 1
De-registration of a building project is not an ideal choice for the authorities keeping in view the larger
interests of the stakeholders as well as the nation as a whole and it is resorted to only when all the possible
avenues of reaching a comfortable and plausible solution are shut.
According to Section 7 of the Real Estate (Regulation and development) Act, 2016 (in short ‘RERA Act’)
various reasons responsible for revocation of registration granted to a project under RERA due to which the
building project stands de-registered are stated as under:
(a) Making of default: The promoter makes default in doing anything required by or under the RERA Act
or the rules or the regulations made thereunder;
(b) Violation of terms or conditions of approval: The promoter violates any of the terms or conditions of
the approval given by the competent authority. From the case study it shall be noticed that broadly the
following terms or conditions of the approval were violated by the promoters of the Royal Golf Burg:
(i) Non-construction of all the apartments in fifteen towers till the due date.
(ii) Non-furnishing of apartments though the deadline to handover the furnished apartments passed.
(c) Involvement in unfair practice or irregularities: The promoter is involved in any kind of unfair practice
or irregularities.
Explanation. — For the purposes of this clause, the term “unfair practice” means a practice which, for
the purpose of promoting the sale or development of any real estate project adopts any unfair method
or unfair or deceptive practice including any of the following practices, namely:
(A) the practice of making any statement, whether in writing or by visible representation which, —
(i) falsely represents that the services are of a particular standard or grade;
(ii) represents that the promoter has approval or affiliation which such promoter does not have;
(iii) makes a false or misleading representation concerning the services;

© The Institute of Chartered Accountants of India


(B) the promoter permits the publication of any advertisement or prospectus whether in any newspaper
or otherwise of services that are not intended to be offered;
(d) Fraudulent practices: The promoter indulges in any fraudulent practices. From the case study it shall
be noticed that the fraudulent practices undertaken by the promoters of the Royal Golf Burg included:
(i) Resorting to double allotment due to which the genuine home-buyers were not allotted the
apartments which they very much deserved.
(ii) Diversion of funds meant for constructing the apartments to shell companies.
(iii) Allotment of apartments in the name of the peons and drivers though such allotments were actually
meant for the use of the promoters since the consideration flowed from them.
Answer 2
When a project is de-registered due to revocation of registration, following are the obligations of the RERA
Authority and other matters associated with it in terms of Section 8 of the RERA Act:
(i) Consultation with the Appropriate Government: The concerned RERA Authority may consult the
appropriate Government to take such action as it may deem fit. This will include:
• the carrying out of the remaining development works by competent authority; or
• the carrying out of the remaining development works by the association of allottees; or
• the carrying out of the remaining development works by in any other manner, as may be determined
by the Authority.
(ii) When the direction, decision or order of the Authority shall take effect: It is provided that a
direction, decision or order of the Authority under section 8 shall take effect only after the expiry of the
period of appeal provided under the provisions of the RERA Act.
(iii) First right of refusal rests with association of allottees: It is provided that in case of revocation of
registration of a project under the RERA Act, the association of allottees shall have the first right of
refusal for carrying out of the remaining development works.
Answer 3
(i) According to Section 6 of the RERA Act, under the circumstances relating to "force majeure” and where
there is no default on the part of the promoter, the registration granted under section 5 of the RERA Act
may be extended by the Authority.
The expression "force majeure" means a case of war, flood, drought, fire, cyclone, earthquake or any
other calamity caused by nature affecting the regular development of the real estate project.
For the purpose of extension, the Promoter of the building project affected by "force majeure" shall make
an application to the Authority in the specified form and shall make payment of the specified fee as
prescribed by the RERA Regulations made by the Authority.
The Authority may in reasonable circumstances, without default on the part of the promoter, based on
the facts of each case, and for reasons to be recorded in writing, extend the registration granted to a
project for such time as it considers necessary. However, the extension of registration shall, in
aggregate, not exceed a period of one year.
The Authority shall not reject any application for extension of registration unless it gives the applicant
an opportunity of being heard in the matter.

© The Institute of Chartered Accountants of India


(ii) According to Section 7 (3) of the RERA Act also, the Authority may, instead of revoking the
registration under sub-section (1) of Section 7, permit the registration to remain in force subject to
such further terms and conditions as it thinks fit to impose in the interest of the allottees, and
any such terms and conditions so imposed shall be binding upon the promoter.
Case study 2
MCQ
1. c
2. d
3. b
4. d
5. a
Descriptive Questions
Answer 1
As per section 2(c) “cartel” includes an association of producers, sellers, distributors, traders or service
providers who, by agreement amongst themselves, limit, control or attempt to control the production,
distribution, sale or price of, or, trade in goods or provision of services.
Although, Northwest Agro Produce Cooperative Society was formed to ensure the timely collection of sale
proceeds from sugar mills. But Northwest Agro Produce Cooperative Society also developed a charter, in
form of memorandum for its members, to regulate and control supply, price, term of sales of sugar canes
(even though on behalf cane-growers), collection of sale proceed and rec overy if required. This
memorandum is binding on all the members of society. Hence Northwest Agro Produce Cooperative Society
is ‘Cartel’ under Competition Act, 2002.
Answer 2
Yes, North West Agro Limited hold dominance, because as per explanation (a) to section 4 “dominant
position” means a position of strength, enjoyed by an enterprise, in the relevant market, in India, which
enables it to (i) operate independently of competitive forces prevailing in the relevant market; or (ii) affect its
competitors or consumers or the relevant market in its favour.
Instances of abuse of dominance
Predatory Pricing after acquisition of Sun Sugar Limited - North West Agro Limited, acquired substantial
network of retailer after takeover of sun sugar Limited, help of which they tried to penetrate in the market
using predatory pricing [Section 4(2)(a)(ii)]. North West Agro Limited reduce the price of its sugar ‘Meetha’
from INRs 40 to 35 per kilogram, where as other player in market like Moon Sugar limited selling sugar at
INRs 40 per kilogram.
As per explanation (b) to section 4 “predatory price” means the sale of goods or provision of services, at a.
price which is below the cost, as may be determined by regulations, of production of the goods or provision
of services, with a view to reduce competition or eliminate the competitors.
Increase the price after acquisition of Moon Sugar Limited – After hostile acquisition of Moon Sugar
Limited, with help of another group company South West Agro Limited; North West Agro Limi ted raise the
prices of its sugar ‘Aur Meetha’ from INRs 35 to 42 per kilogram; even Moon Sugar Limited, originally selling
its sugar ‘Aur’ at INRs 40 per Kilogram. Section 4(2)(b)(i) says there shall be an abuse of dominant position
under sub-section (1) of section 4, if an enterprise or a group limits or restricts production of goods or market
therefore .

© The Institute of Chartered Accountants of India


Answer 3
Provision of related to combination detailed in section 5 of Competition Act ,2002
Sr. Nature of Case facts Criteria Is combination
No Combination
Acquisition by North West Agro Joint Asset overYes, Joint turnover is INRs
single acquirer but Limited Takeover INRs 1000 crores4400 crores (3200+1200)
1 different goods Sun Sugar Limited or Turnover overwhich is more than INRs 3000
(Section 5(a)(i) INRs 3000 crorescrores, whereas joint assets
base is only INRs 998 crores
Acquisition by North West Agro Group Asset over No, Joint Asset base of group
group with similar Limited Acquired INRs 4000 crores is only INRs 1566 crores and
goods (Section Moon Sugar or Turnover over aggregate turnover is also
2 5(b)(ii) Limited, with help of INRs 12000 crores INRs 7200 Crores
another group
company South
West Agro Limited
Not considered MOU between Not eligible to be Not Applicable
as combination North West Agro considered as
3
Limited and Star combination
Ethanol Limited

Regulation of Combinations (Section 6)


No person or enterprise shall enter into a combination which causes or is likely to cause an appreciable
adverse effect on competition within the relevant market in India and such a combination shall be void.
Any person or enterprise, who enter into a combination, give notice to the Commission, disclosing the
details of the proposed combination, within thirty days of -
(a) Approval of the proposal concerned with such merger or amalgamation by the board of directors, or (b)
execution of any agreement acquiring of control
No combination shall come into effect until two hundred and ten days have passed from the day on
which the notice has been given to the Commission or the Commission has passed orders whichever is
earlier.
Answer 4
Vide Foreign Exchange Management (Current Account Transactions) Rules 2000, some restriction on
current account foreign exchange transaction prescribed. A general permission is available for opening of
Bank Account for the purpose of meeting the Branch Expenses abroad.
Authorised Dealer Category – I banks may allow remittance up to ten per cent of the average annual
turnover during the last two financial years. But this restriction shall not be applicable if remittance to
account maintained abroad, made out of the funds held in EEFC account.
In given case turnover for relevant 2 years was INRs 120000 lakhs and INRs 110000 lakhs (i.e. INRs 120000
lakhs - INRs 10000 lakhs). Average of which is INRs 115000 lakhs. Maximum permissible amount of branch
recurring expenditure in case of normal account was 10% of INRs 115000 lakhs i.e INRs 11500 lakhs.
Expenditure incurred by Sun Sugar Limited in given case is INRs 14000 lakhs. But such expenditure will not
be considered in violation of Foreign Exchange Management (Current Account Transactio ns) Rules 2000,
because amount expended out of EEFC Account .

© The Institute of Chartered Accountants of India


Case study 3
Multiple Choice questions (MCQs)
1. (d)
2. (c)
3. (b)
4. (d)
5. (c)
Descriptive questions
Answer1
The general belief is that the provisions of the Prohibition of Benami Property Transactions Act 1988 (PBPT
Act) apply only to persons, trying to hide their properties and not to genuine properties acquired out of
disclosed funds. But that is not true. Even a property acquired using disclosed funds in a genuine transaction
may sometimes be treated as Benami.
“Benami Property” under Sec 2 (8) means any property, which is the subject matter of a Benami transaction
and also includes the proceeds from such property.Benami Property means property without a name. Here
the person, who pays for the property does not buy it under his own name. The person, who finances the
deal is the real owner of the property. The person in whose name the property has been purchased is
Benamidar.
As per the provisions of Section 2 (9) a Benami transaction means-
1. A transaction or arrangement where a property is transferred to or held by one person for direct or
indirect, immediate or future benefit of another person, who has provided or paid the consideration,
except when-
(i) An HUF is purchasing a property in the name of a Karta, or any other member from known sources;
(ii) A person is holding the property in a fiduciary capacity (e.g. trustee, executor, partner of a
partnership firm, director of a company, a depository participant, etc.);
(iii) An individual is purchasing a property in the name of his spouse or any child provided the
consideration is paid out of the known sources;
(iv) Any person is purchasing any property in the name of his brother or sister or lineal ascendant or
descendant, where he is one of the joint-owners, provided the consideration is paid out of the
known sources; or
2. A transaction or arrangement carried out in a fictitious name; or
3. A transaction or arrangement where the owner of the property is not aware of or denies knowledge of
such ownership;
4. A transaction or arrangement, where the person providing the consideration is not traceable or is
fictitious.
Any transaction where possession of any immovable property is taken as a part performance of a contract is
not a Benami transaction if the contract is registered and consideration as well as stamp duty have been paid.
Property would include asset of any kind, whether movable or immovable, tangible or intangible, and includes
rights or interest as well as proceeds from the property.

© The Institute of Chartered Accountants of India


In the above case study, in one of the cases, SUBHASH invested Rs. 1.50 Crores in a Bank Fixed deposit in
the name of his married daughter, MANGALA, who is a UK Resident, without her knowledge. Later during
the course of enquiries by Tax officials MANAGALA denies ownership of Bank Fixed Deposit. H ere, the
transaction is Benami, though the FD is generated using disclosed funds in a genuine transaction.
Answer 2
As per the provisions of Section 2(16) of the Prohibition of Benami Property Transactions Act 1988, "fair
market value", in relation to a property, means—
(i) the price that the property would ordinarily fetch on sale in the open market on the date of the
transaction; and
(ii) where the price referred to in sub-clause (i) is not ascertainable, such price as may be determined in
accordance with such manner as prescribed in Rule 3 of Prohibition of Benami Transactions Rules,
2016
Determination of price in certain cases under Rule 3 of Prohibition of Benami Transactions Rules,
2016
3. (1) For the purposes of sub-clause(ii) of c lause (16) of the section 2 of the Ac t, the price shall be
determined in the following manner, namely:—
(a) the price of unquoted equity shares shall be the higher of,—
(I) its cost of acquisition;
(II) the fair market value of such equity shares determined, on the date of transaction, by a
merchant banker or an accountant as per the Discounted Free Cash Flow method; and
(III) the value, on the date of transaction, of such equity shares as determined in the following
manner, namely:—
The fair market value of unquoted equity shares = (A+B - L) x (PV)/(PE)
where,
A= book value of all the assets (other than bullion, jewellery, precious stone, artistic work,
shares, securities and immovable property) as reduced by,-
(i) any amount of income-tax paid, if any, less the amount of income-tax refund claimed, if
any, and
(ii) any amount shown as asset including the unamortised amount of deferred expenditure
which does not represent the value of any asset;
B= the price that the bullion, jewellery, precious stone, artistic work, shares, securities and
immovable property would ordinarily fetch on sale in the open market on the date of
transaction;
L= book value of liabilities, but not including the following amounts, namely:—
(i) the paid-up capital in respect of equity shares;
(ii) the amount set apart for payment of dividends on preference shares and equity shares;
(iii) reserves and surplus, by whatever name called, even if the resulting figure is negative,
other than those set apart towards depreciation;
(iv) any amount representing provision for taxation, other than amount of income-tax paid, if
any, less the amount of income-tax claimed as refund, if any, to the extent of the excess
over the tax payable with reference to the book profits in accordance with the law
6

© The Institute of Chartered Accountants of India


applicable thereto;
(v) any amount representing provisions made for meeting liabilities, other than ascertained
liabilities;
(vi) any amount representing contingent liabilities other than arrears of dividends payable in
respect of cumulative preference shares;
PE = total amount of paid up equity share capital as shown in the balance-sheet;
PV= the paid up value of such equity shares;
The above provisions of Section 2(16) of the Act Read with Rule 3 is applicable, in case of confiscation of
properties of RAJESH.
Answer 3
Attachment of property involved in money-laundering under Section 5 of the Prevention of Money
laundering Act 2002?.
Section 5 (1) Where the Director or any other officer not below the rank of Deputy Director authorised by
the Director for the purposes of this section, has reason to believe (the reason for such belief to be recorded
in writing), on the basis of material in his possession, that—
(a) any person is in possession of any proceeds of crime; and

(b) such proceeds of crime are likely to be concealed, transferred or dealt with in any manner which
may result in frustrating any proceedings relating to confiscation of such proceeds of crime under
this Chapter,

he may, by order in writing, provisionally attach such property for a period not exceeding one hundred and
eighty days from the date of the order, in such manner as may be prescribed:
Provided that no such order of attachment shall be made unless, in relation to the scheduled offence, a
report has been forwarded to a Magistrate under section 173 of the Code of Criminal Procedure, 1973 (2 of
1974), or a complaint has been filed by a person authorised to investigate the offence mentioned in that
Schedule, before a Magistrate or court for taking cognizance of the scheduled offence, as the case may be,
or a similar report or complaint has been made or filed under the corresponding law of any other country:
Provided further that, notwithstanding anything contained in [first proviso], any property of any person may
be attached under this section if the Director or any other officer not below the rank of Deputy Director
authorised by him for the purposes of this section has reason to believe (the reasons for such belief to be
recorded in writing), on the basis of material in his possession, that if such property involved in money-
laundering is not attached immediately under this Chapter, the non-attachment of the property is likely to
frustrate any proceeding under this Act.
Provided also that for the purposes of computing the period of one hundred and eighty days, the period
during which the proceedings under this section is stayed by the High Court, shall be excluded and a further
period not exceeding thirty days from the date of order of vacation of such stay order shall be counted.
(2) The Director, or any other officer not below the rank of Deputy Director, shall, immediately after
attachment under sub-section (1), forward a copy of the order, along with the material in his possession,
referred to in that sub-section, to the Adjudicating Authority, in a sealed envelope, in the manner as may be
prescribed and such Adjudicating Authority shall keep such order and material for such period as may be
prescribed.

© The Institute of Chartered Accountants of India


(3) Every order of attachment made under sub-section (1) shall cease to have effect after the expiry of the
period specified in that sub-section or on the date of an order made under [sub-section (3)] of section 8,
whichever is earlier.
(4) Nothing in this section shall prevent the person interested in the enjoyment of the immovable property
attached under sub-section (1) from such enjoyment.
Explanation.—For the purposes of this sub-section, "person interested", in relation to any immovable
property, includes all persons claiming or entitled to claim any interest in the property.
(5) The Director or any other officer who provisionally attaches any property under sub-section (1) shall,
within a period of thirty days from such attachment, file a complaint stating the facts of such attachment
before the Adjudicating Authority.
Case study 4
Multiple Choice Questions
1. (d)
2. (a)
3. (d)
4. (c)
5. (a)
Descriptive Questions
Answer 1
As per the provisions of the Section 11 (1) The promoter shall, upon receiving his Login Id and password
under clause (a) of sub-section (1) or under sub-section (2) of section 5, as the case may be, create his web
page on the website of the Authority and enter all details of the proposed project as provided under sub -
section (2) of section 4, in all the fields as provided, for public viewing, including—
(a) details of the registration granted by the Authority;
(b) quarterly up-to-date the list of num ber and types of apartments or plots, as the c ase m ay be,
booked;
(c ) quarterly up-to-date the list of number of garages booked;
(d) quarterly up-to-date the list of approvals taken and the approvals which are pending subsequent to
c ommencement certificate; (e) quarterly up -to-date status of the project; and
(f) suc h other information and doc uments as m ay be spec ified by the regulations m ade by the
Authority.
Thus, the Alpha promoter are required to be disclosed above mentioned information for public view on the
webpage of promoter Alpha on the Website of the DELHI RERA Authority.
Answer 2
The payment for purchase of the flat by Mr. P, a non resident in India is made in parts. Since, the payment
of INR 25,00,000 is made from unknown source for registry of the apartment. So, to the extent of unknown
source of the consideration payment, it may be a Benami Transaction.

© The Institute of Chartered Accountants of India


Answer 3
(i ) As per the provisions of the Section 11(4)(c) of the Real Estate (Regulation & Development) Act,
2016, the promoter shall be responsible to obtain the lease c ertificate, where the real estate project
is developed on a leasehold land, specifying the period of lease, and c ertifying that all dues and
c harges in regard to the leasehold land has been paid, and to make the lease c ertificate available
to the association of allottees.
T hus, Alpha Promoter have to fulfil all responsibilities as discussed above.
(i i ) As per the provisions of the Section 11(5) of the Real Estate (Regulation & Development) Ac t, 2016,
the promoter may c ancel the allotment only in terms of the agreement for sale.
Provided that the allottee may approach the Authority for relief, if he is aggrieved by such cancellation
and such cancellation is not in accordance with the terms of the agreement for sale, unilateral and without
any sufficient cause.
Thus, the Alpha Promoter cannot cancel the allotment without including such terms of cancellation in
the executed agreement of sale.
Case study 5
Multiple choice Questions
1. b
2. b
3. c
4. c
5. b
Descriptive Questions.
Answer 1
(i) A body c orporate incorporated outside India (including a firm or other association of individuals),
desirous of opening a Liaison Office (LO) / Branch Office (BO) in India have to obtain permission
from the Reserve Bank under provisions of FEM A 1999.Establishment of Project Offices/Liaison
Offices in India is regulated in terms of Section 6(6) of Foreign Exchange M anagement Ac t, 1999
read with Notification No. FEMA 22/2000-RB dated M ay 3, 2000.
So if the foreign c ompany wants to establish a Liaison Office in India, it c annot acquire immovable
property. However, the company needs to acquire property by way of lease not exceeding 3 years,
for its Liaison Office.
(ii) A non-resident Indian (NRI) or person of Indian origin (PIO), can inherit any immovable property in
India, whether it is residential or c ommercial. T he FEM A regulations applicable to transfer of
im movable property permit a NRI/PIO to acquire an immovable property in India by way of gift from
a person resident in India provided the property is not agricultural land/ farm house/ plantation
property. All gifts to NRIs will be considered as income accruing in India and would be taxed as per
the norm al slab ra tes applicable to resident Indians. T his m eans that the NRI/PIO needs to
m andatory pay the applicable tax
A liaison office of a foreign c ompany in India should be established only with requisite approvals
wherever necessary, and is eligible to take any immovable property on lease, in India which is

© The Institute of Chartered Accountants of India


nec essary for its activities, provided that all such applicable laws, rules, Regulations or directions
in forc e are duly c omplied with.
But in the aforementioned both the cases the property is bought in c ontravention of the provisions
of benami ac t hence it is a benami transaction. It is liable to attract section 3 of Prohibition of
Benami Property T ransactions Ac t. T he property is liable to attac hment by the Adjudicating
Authority.
A liaison office of a foreign c ompany in India should be established only with requisite approvals
wherever necessary, and is eligible to take any immovable property on lease, in India which is
nec essary for its ac tivities, provided that all such applicable laws, rules, regulations or directions
in force are duly c omplied with. Since the property liable to be leased is in brought in c ontravention
to the provisions of law applicable in India, it c annot be leased. If any c ase it is leased without the
knowledge of the company the lease stand terminated. In c ase of gift to an NRI niece the gift stands
null and void.
Answer 2
i. A Benami transaction is an arrangement, where a property is transferred to, or is held by, in the
nam e of a person c alled Benamidar and the consideration for such property has been provided, or
paid by another person c alled beneficiary; and that property (benami property) is held for the
im mediate or future benefit, direct or indirect, by that person who has provided the consideration.
In the particular case the consideration is paid by M r Suresh Agarwal but the property is purchased
in his m other's name.
So it is c lear that, Mr Suresh Agarwal has done this benami transactions in respect of the holding of
the property, with the object of c oncealing the real owner.
ii. In case, if after the issuance of the initial notice by the Initiating Officer, the property in question is
transferred to a third party secretively, than the said transaction would be null and void and confiscation
would take effect, notwithstanding such a transfer.
However, in c ase where the person proves that, the property is held or ac quired by a person from
the Benamidar for:
a. adequate consideration, prior to the issuance of show-cause notice by the Initiating Officer
b. Without his having knowledge of the benami transaction in such a case the transfer may be valid.
iii. Effect of confiscation of benami property:
Where an order of c onfiscation has been made, all the rights and title in suc h property will vest
absolutely in the Central Government. It will be free of all encumbrances and no c ompensation will
be paid in respect of such confiscation.

10

© The Institute of Chartered Accountants of India


DISCLAIMER

This Suggested Answers do not constitute the basis for evaluation of the

student’s answers in the examination. The answers are prepared by the

Faculty of the Board of Studies with a view to assist the students in their

education. While due care is taken in preparation of the answers, if any

errors or omissions are noticed, the same may be brought to the attention

of the Director of Studies. The Council of the Institute is not in anyway

responsible for the correctness or otherwise of the answers published

herein.

Further, in the Elective Papers which are Case Study based, the solutions

have been worked out on the basis of certain assumptions/views derived

from the facts given in the question or language used in the question. It may

be possible to work out the solution to the case studies in a different manner

based on the assumption made or view taken. Further, there should be no

negative marking for wrong answers in MCQ based questions.

© The Institute of Chartered Accountants of India


2 FINAL (NEW) EXAMINATION: MAY 2019

PAPER-6D – ECONOMIC LAWS


There are three case study questions in the question paper. Candidates are required to
answer all the questions of any two case study questions.

CASE STUDY 1
Mr. Rohit writer is a well known industrialist based in Pune, India and is the founder director of
M/s. Good Phones Private Limited (Good Phones), a fixed line and mobile phone manufacturer.
Good Phones is one of the largest telephone companies in India and its products are much
sought after in India and abroad. Mr. Rohit visits various countries as part of his business travels
and during these visits he spend significant amount of time in Philanthropic activities and social
gatherings and because of this, he is quite well known in the business circles globally. Mr. Rohit
has a penchant for investing his money in buying various real estate property all over India and
has passed this trait on to his son, Mr. Rahul Writer as well. Mr. Rahul completed his MBA from
Stanford University and is assisting Mr. Rohit in his business. Mr. Rohit also has a daughter,
Mr. Sonali Writer, who studies Art in Italy and has opened her own Art Studio in Milan. Mr. Rohit
is very proud of Sonali and supports her financially for her stay in Italy as well as expenses
towards maintaining the studio.
The marketing department of Good Phones introduced various new models in the last couple of
months with new technology such as 2 selfie cameras, faster processor and sleeker look. Good
Phones expect these phones to be a major attraction in the global markets due to the attractive
price range and therefore wanted to promote these phones extensively on a global basis. For
the purpose of advertisements, Good Phones engaged the services of Mr. David Smith, a
prominent baseball player and Ms. Emma Drew, a Miss Universe winner and agreed to pay a
“guaranteed” fee of USD 500,000 each plus 10% bonus based on the sales of the new models
in year 1.
Mr. Rohit sent 5 sample mobile phones and 5 fixed line phones to his dealers abroad (numbering
1000 dealers), clearly marked as not for sale and other promotional material such a brouchers,
3D moulds for display in dealer shops etc. The value of the items were approximately INR 4
crore. He also sent 1 mobile phone to each of his dealers as a token of gift and appreciation
(total value of INR 0.50 Crores). Mr. Srinivas Rajan, the CFO of Good Phones indicated him
that since these products have been sent free of cost and not for sale, these need not be
included in the export declaration to be filed by Good Phones.
On 15 February 2018, Good Phones made a large sale to one of the dealers M/s. Delayed
Ringtone Enterprise, Germany, for USD 5 million and had received USD 2 million by 15 May
2018 and did not receive the balance USD 3 million until 15 August 2018, i.e. 6 months from the
date of sale. After several reminders and threatening calls to stop further shipment, another
USD 1 million was received on 10 October 2018 and the balance remained outstanding as at
31 December 2018.

© The Institute of Chartered Accountants of India


PAPER – 6D: ECONOMIC LAWS 3

Based on the success of Good Phones, Mr. Rohit incorporated a new company, M/s. Stay
Connected Private Limited, (Stay Connected) an Internet service provider and purchased a large
consignment of networking equipment for providing internet operations through dedicated
broadband lines along with a landline facility. This would then provide Mr. Rohit quite a few
synergies with the existing Good Phones business and enable him to become an end to end
Telecom Czar. Mr. Rohit held 60% stake in Stay Connected and the balance 40% was held by
a foreign collaborator. Along with all the networking equipment, Stay Connected hired
transponders from a company in Australia and paid AUD 10 million through its authorized healer.
Stay Connected also entered into an agreement with foreign collaborator (holding 40% stake)
to pay royalty and technical fees for the support provided by them.
During his visit to Milan to meet Ms. Sonali, Mr. Rohit obtained EUR 10,000 from his Italian
dealer for his use during his stay in Italy and instructed the dealer to reduce the sum from the
payments to be made by the dealer for the supplies from Good Phones. Out of such funds,
Mr. Rohit used EUR 5,000 towards purchasing sweepstakes tickets in Mila, Italy, unfortunately,
he did not win any money in the sweepstakes event.
Mr. Rahul, after gaining experience in India, wanted to expand the business in the USA (by
establishing a subsidiary of Good Phones in the USA) and therefore decided to move to the
USA along with his wife. For this purpose, he wanted to dispose off some of the properties
owned by him in India. Accordingly, Mr. Rahul sold an apartment in Mumbai owned by him to
Mr. Stuart Cooper, a citizen of USA, and a fellow student of his at Stanford University. Mr. Stuart
was planning to come to India for the first time in the next couple of months to take up a job and
therefore, wanted to secure a place for his stay. Mr. Rohit also sold a villa and his agricultural
land in Pondicherry to Mr. Rajesh Subramanian, his professor at Stanford, who was a person of
India origin. The payment for the villa and agricultural land was paid by Mr. Rajesh partly (50%)
from his FCNR account and the balance in USD traveller cheques, which will be of use to Mr.
Rahul when he visits USA.
After obtaining his US visa, Mr. Rahul purchased a ranch (farm house) in Texas for USD 2
million, using USD 1.50 million from his RFC account and USD 500,000 sent from his INR
account through normal banking channels.
Mr. William Rutherford, one of Mr. Rohit’s business acquaintances and a citizen of the USA, is
very much interested in Indian culture and practices and therefore stays in India for 8 months
(from April 2018 to November 2018) to attend an art of living course and to learn/practice yoga.
William believes that he has been resident in India for more than the prescribed 182 days and
therefore, is a resident in India under FEMA.
Mr. Rohit, in his penchant for purchasing various properties, zeroed in on an exclusive
apartment complex in Bangalore having state-of-the art facilities. He purchased two 4 bedroom
apartments costing INR 2 crore each, one in the name of Ms. Sonali and one in the name of Mr.
Srinivas Rajan, since Mr. Rohit wanted Mr. Srinivas Rajan to feel happy and trusted. Both the
apartments were given on rent to a large multinational bank and he received a rent of INR 0.20
Crores per year for each of the apartments in the bank accounts of Ms. Sonali and Mr. Srinivas

© The Institute of Chartered Accountants of India


4 FINAL (NEW) EXAMINATION: MAY 2019

Rajan, respectively. After 4 years, Mr. Srinivas Rajan transferred the property back in the name
of Mr. Rohit at zero consideration. Mr. Rohit also purchased a 3 bedroom apartment in the same
complex in his name, jointly with his brother, Mr. Sunil Writer. The property (along with the stamp
duty) was paid for by Mr. Rohit and was being used by Mr. Sunil for his stay though the property
was pending registration due to Rohit’s travel abroad.
Once the property was transferred back by Mr. Srinivas Rajan, Mr. Rohit wanted to sell the same
to Mr. Arjun De Silva, a citizen of Shi Lanka. However, he was advised by Mr. Srinivas Rajan
that Mr. Arjun De Silva cannot acquire property in India and therefore, Mr. Rohit proposed to
lease it to Mr. Arjun De Silva for a period of 20 years for an upfront consideration of INR 1 crore
and an annual rent of INR 8 lakhs payable in advance.
During the review of the bank reconciliation statements of Good Phones, Mr. Srinivasan Rajan
noted that an amount of INR 2 Crore had been received in one of the bank accounts without
any details relating to the same. Mr. Srinivas Rajan informed this to Mr. Rohit and Mr. Srinivas
Rajan suggested to Mr. Rohit to immediately transfer that money out of the bank of Good Phones
to Mr. Rohit’s personal bank account, so that the Company’s bank accounts are cleared and
there are no reconciling items, which Mr. Rohit agreed to. Out of the INR 2 Crore, Mr. Rohit
used INR 1.75 Crores for acquiring further 20% stake in Stay Connected from the foreign
collaborator and the balance INR 0.25 Crores for purchasing a stunning diamond set for his
wife, Ms. Anjali Writer, as a gift for her 50 th birthday.
The extract of the last audited financial statements of Stay Connected was provided by
Mr. Srinivas Rajan to Mr. Rohit to evaluate his acquisition.
Particulars Amount in
INR (Crores)
Immovable property (market value INR 8.00 Crores) 5.00
Other fixed assets (net of depreciation of INR 1.00 Crores) 4.00
Inventory 2.00
Receivables and Loans and Advances 1.50
Deferred Advertisement Costs 0.50
Advance tax paid 1.00
Total Assets 14.00
Shareholders’ Funds (including 1,000,000 equity shares of INR 10 each, 4.00
fully paid up)
Provisions for taxation 0.50
Loans from Banks 3.00
Trade Payables (including provision for unascertained liabilities - INR 6.50
1 crore)
Total Liabilities 14.00

© The Institute of Chartered Accountants of India


PAPER – 6D: ECONOMIC LAWS 5

Other information:
(i) Contingent liabilities-INR 2.00 Crores (including INR 0.50 cores relating to arrears on
cumulative preference shares).
(ii) The Board of Directors has proposed a dividend payout of INR 1 crore to the equity
shareholders, which is pending approval of the shareholders.
The Bank, on noting the large transactions on Mr. Rohit's personal bank account, tipped
the Income tax authorities regarding the same and the Initiating Officer summoned
information from Mr. Rohit and Mr. Srinivas Rajan regarding the transactions to start
proceedings under the Prohibition of Benami Property Transactions Act, 1988 (PBPT Act,
1988).
Answer the following questions:
1.1 An Indian citizen resident outside India is permitted to transfer his agricultural property in
India to:
(A) any person resident in India
(B) a person resident in India if he is a citizen of India or a person of Indian origin
(C) a person resident in or outside India if he is a citizen of India or a person of Indian
origin
(D) any person who is resident in the country where the Indian citizen currently resides
(2 Marks)
1.2 Out of the below, what are the transactions that requires prior approval of the Government
of India ?
(A) Payment of "guaranteed" fee by Good Phones to Mr. David Smith and Ms. Emma
Drew.
(B) Payment of Royalty and Technical fees by Stay Connected to the foreign collaborator.
(C) Payment of hiring charges for the transponders by Stay Connected.
(D) Payment of INR 1.75 Crores by Mr. Rohit to acquire shares of Stay Connected from
the foreign collaborator (2 Marks)
1.3 Is the use of EUR 5,000 towards purchasing sweepstakes by Mr. Rohit as per the
provisions of FEMA, 2002 ?
(A) No, drawal of foreign exchange for purchasing lottery tickets, sweepstakes, etc. is
prohibited under the FEMA 2002

© The Institute of Chartered Accountants of India


6 FINAL (NEW) EXAMINATION: MAY 2019

(B) No, Mr. Rohit should have obtained the prior approval of the RBI before purchasing
the sweepstakes ticket
(C) FEMA 2002 will not be applicable, since the money was directly obtained by
Mr. Rohit from his Italian dealer outside the country
(D) None of the above (2 Marks)
1.4 As per the provisions of FEMA, 2002, Mr. William Rutherford is :
(A) a person resident in India in the financial year 2018-19 as per FEMA, "since he has
resided for more than 182 days during the year
(B) a person resident in India in the financial year 2019-20 as per FEMA, since he has
resided for more than 182 days during the previous financial year
(C) not a person resident in India since he is a foreign citizen
(D) not a person resident in India, since he is on a short term visit to India and is not on
a long term visa (2 Marks)
1.5 Is the purchase of Ranch in Texas by Mr. Rahul in accordance with FEMA, 2002?
(A) No, Rahul, as a citizen of India cannot purchase a Ranch outside India.
(B) Yes, there is no specific limit under FEMA 2002 with regard to purchase of
immoveable property outside India.
(C) No, Rahul can purchase assets outside India only if the purchase is jointly with a
relative, who is a resident outside India, and there is no outflow of funds.
(D) No, since Rahul has used funds from his INR account for making the payment to the
extent of USD 500,000. (2 Marks)
1.6 In case Mr. Rohit is proven guilty of violating the provisions of PBPT Act, 1988, what is the
maximum punishment that he is liable for under the PBPT Act, 1988 ?
(A) Rigorous imprisonment for a term of one to seven years, with fine which may extend
to 25 of the fair market value of the property.
(B) Rigorous imprisonment for a term of three to seven years, without fine.
(C) Rigorous imprisonment for a term upto seven years, with fine which may extend to
50% of the fair market value of the property.
(D) Fine which may extend to 25% of the fair market value of the property. (2 Marks)
1.7 Once the benami property acquired by Mr. Rohit and his family have been identified, which
authority has the power to confiscate and vest the property?

© The Institute of Chartered Accountants of India


PAPER – 6D: ECONOMIC LAWS 7

(A) Initiating Officer


(B) Administrator
(C) Approving Authority
(D) Adjudicating Authority (2 Marks)
1.8 Which of the following is a benami property under the PBPT Act, 1988 ?
(A) Property held by a member of a Hindu Undivided Family (not the Karta) which is held
for the benefit of the members of the family
(B) Property purchased by a person in the name of his spouse paid out of money received
from his father in law as a gift.
(C) Property purchased by a person in the name of his brother out of his own funds.
(D) Property purchased in the name of an individual, for which consideration was paid by
another person and such another person enjoys the possession of the property.
(2 Marks)
1.9 Assuming that the transactions relating to the receipt of INR 2 crores in the bank account
of Good Phones and the subsequent transactions are considered as benami transactions,
can the Initiating Officer take action against Mr. Srinivas Rajan ?
(A) Yes, he is the CFO of Good Phones and therefore, responsible for ensuring
compliance with the law.
(B) No, he has not received, held, or acquired the proceeds in his account or benefitted
from the same.
(C) Yes, since he abets Mr. Rohit in transferring the money from the bank account of
Good Phones to Mr. Rohit's personal account.
(D) No, he is responsible only for Good Phones and he has ensured that the funds are
not retained in the books of Good Phones / used by Good Phones for its business.
(2 Marks)
1.10 The Initiating Officer believes that Ms. Anjali is not a benamidar under the PBPT Act,
1988. What is your view?
(A) No, she is not a benamidar, since she has not purchased the diamond set, but
received as a Gift.
(B) Yes, she is a benamidar as she is in possession of a property acquired out of benami
funds.

© The Institute of Chartered Accountants of India


8 FINAL (NEW) EXAMINATION: MAY 2019

(C) No, she is not a benamidar, since the transaction is not a benami transaction.
(D) Yes, she is a benamidar, but will not be liable for any prosecution under the PBPT
Act, since she is not a party to any of the transactions but only a beneficiary.
(2 Marks)
1.11 Answer the following questions in the context of the provisions relating to the Foreign
Exchange Management Act, 2002 :
(i) Srinivas Rajan reaches out to you to confirm his views regarding inclusion / exclusion
of the items sent free of cost to the dealers in the export declaration.
(4 Marks)
(ii) Examine the validity / appropriateness of the sale of immoveable property by Mr.
Rahul to Mr. Stuart Cooper and Mr. Rajesh Subramanian. (4 Marks)
(iii) Srinivas Rajan reaches out to you and seek your support to evaluate if there is a non-
compliance with the FEMA regulations regarding the sale made to M/s Delayed
Ringtone and the receipt of the proceeds and if so, the quantum, the consequences
and the future course of action that needs to be taken by Good Phones relating to the
same. (4 Marks)
(iv) Arjun disagrees with the advice received from Srinivas Rajan and asks your views on
why he cannot purchase the home from Mr. Rohit and if not, whether the terms and
conditions of the lease are acceptable. (2 Marks)
1.12 Examine/advice regarding the below questions relating to the Prohibition of Benami
Property Transaction Act, 1988:
(i) Examine the appropriateness/impact of the PBPT Act 1988 on the 3 apartments
purchased by Mr. Rohit in Bangalore. How does the transfer back of the apartment
by Mr. Srinivas Rajan to Mr. Rohit affect your conclusion? (4 Marks)
(ii) The Initiating Officer, who is probing the transactions relating to the INR 2 crores
received and spent by Mr. Rohit, seeks your advice on identify the benami
properties/transaction, the benamidars, the beneficial owner. (4 Marks)
(iii) Assuming that the cost of acquisition and the market value based on discounted cash
flow method is INR 2 crores, calculated the fair market value of M/s. Stay Connected
in accordance with Rule 3 of the Prohibition of Benami Transactions Rules, 2016.
(4 Marks)
(iv) What is the process to be followed by the Initiating Officer for attachment of the
property under a benami transaction? (3 Marks)

© The Institute of Chartered Accountants of India


PAPER – 6D: ECONOMIC LAWS 9

Answer Case study 1


1.1 (A)
1.2 (C).
1.3 (A)
1.4 (D)
1.5 (D)
1.6 (A)
1.7 (D)
1.8 (D)
1.9 (C)
1.10 (B)
1.11 (i) As per the Regulation 3 of the Foreign Exchange Management (Export of Goods and
Services) Regulations, 2015, in case of exports taking place through Customs manual
ports, every exporter of goods or software to any place outside India, shall furnish to
the specified authority, a declaration as regards the export value of the Goods. In
respect of export of services to which none of the Forms specified in these
Regulations apply, the exporter may export such services without furnishing any
declaration, but shall be liable to realise the amount of foreign exchange whic h
becomes due or accrues on account of such export, and repatriate the same to India.
However, Regulation 4 of the said Regulation states of exemptions w.r.t. export of
goods / software may be made without furnishing the declaration on the following
items which are sent free of cost:
• trade samples of goods and publicity material supplied free of payment;
• by way of gift of goods accompanied by a declaration by the exporter that they
are not more than five lakh rupees in value.
Therefore, sending 5 sample mobile phones and fixed line phones to 1000 dealers is
exempted and does not require Good Phones to include in the export declaration.
With regard to sending mobile phones to the dealers as gift for a total value of
INR 50 lakhs, as per the above Regulation, the exemption for sending gifts by an
export is available only if the value of the goods are not more than ` 5 lakhs in value.
In the case study, since the value of the goods is more than the exemption limit, they
need to be included in the export declaration.
(ii) As per regulation 3 of the Foreign Exchange Management (Acquisition and Transfer
of Immovable Property in India) Regulations, 2018, an NRI or OCI, may acquire

© The Institute of Chartered Accountants of India


10 FINAL (NEW) EXAMINATION: MAY 2019

immovable property in India other than agricultural land/farm house/plantation


property. Provided that the consideration, if any, for transfer, shall be made out of (i)
funds received in India through banking channels by way of inward remittance from
any place outside India or (ii) funds held in any non resident account maintained in
accordance with the provisions of the Act, rules or regulations framed thereunder.
Mr. Staurt Cooper, being an Overseas Citizen of India is entitled to acquire an
apartment in Mumbai owned by Mr. Rahul.
Whereas in case of Mr, Rajesh Subramanian, being NRI, he may acquire immovable
property in India other than agricultural land/farm house/plantation property.
Provided further that no payment for any transfer of immovable property shall be
made either by traveler's cheque or by foreign currency notes or by any other mode
other than those specifically permitted under this regulation. Since with related
transactions, payment was made partly from FCNR account and in USD Traveler
cheques, which was against the mode of payment prescribed in the said regulation.
Therefore, sale of immovable property by Mr. Rahul to Mr. Stuart Cooper is valid,
whereas to Mr. Rajesh Subramanian, the said transaction is invalid.
(iii) Regulation 9 of the Foreign Exchange Management (Export of Goods and Services)
Regulations, 2015 specifies the Period within which export value of
goods/software/services to be realised.
According to it the amount representing the full export value of
goods/software/services exported shall be realised and repatriated to India within
nine months from the date of export.
Further that the Reserve Bank, or subject to the directions issued by that Bank in this
behalf, the authorised dealer may, for a sufficient and reasonable cause shown,
extend the period of nine months.
Due to delay in of the proceeds of sale, M/s Delayed Ringtone contravened provision
of this Act & Regulations.
According to Regulation 14 of the Foreign Exchange Management (Export of Goods
and Services) Regulations, 2015, where in relation to goods or software export of
which is required to be declared on the specified form and export of services, in
respect of which no declaration forms has been made applicable, the specified period
has expired and the payment therefor has not been made as aforesaid, the Reserve
Bank may give to any person who has sold the goods or software or who is entitled
to sell the goods or software or procure the sale thereof, such directions as appear
to it to be expedient, for the purpose of securing,
(a) the payment therefor if the goods or software has been sold and
(b) the sale of goods and payment thereof, if goods or software has not been sold
or reimport thereof into India as the circumstances permit, within such period as

© The Institute of Chartered Accountants of India


PAPER – 6D: ECONOMIC LAWS 11

the Reserve Bank may specify in this behalf;


Provided that omission of the Reserve Bank to give directions shall not have the effect
of absolving the person committing the contravention from the consequences thereof.
Therefore, in such situation, M/s Delayed Ringtone shall, upon adjudication under
Section 13 of the FEMA, be liable to a penalty up to thrice the sum involved in such
contravention where such amount is quantifiable, or up to two lakh rupees where the
amount is not quantifiable, and where such contravention is a continuing one, further
penalty which may extend to five thousand rupees for every day after the first day
during which the contravention continues.
Hence, Mr. Delayed will be liable to USD 6 million [2 million (sum involved in such
contravention) x 3)] and further penalty up to five thousand rupees for every day after
the first day during which the contravention continues.
(iv) As per Regulation 9 of the Foreign Exchange Management (Acquisition and Transfer
of Immovable Property in India) Regulations, 2018, prohibition on acquisition or
transfer of immovable property in India is cast on the citizens of certain countries.
Accordingly, no person being a citizen of Sri Lanka, without prior permission of the
Reserve Bank shall acquire or transfer immovable property in India, other than lease,
not exceeding five years.
Since in the given case, Arjun, being a citizen of Sri Lanka, is prohibited to acquire
property in India, except on lease for 5 years. If Mr. Rohit proposes to lease Mr. Arjun
for a period of 20 years, can do so only on the prior permission of the RBI.
1.12 (i) In the given case study, Mr. Rohit purchased 3 flats in Bangalore in the name of Ms.
Sonali, Mr. Srinivas Rajan, and jointly with his brother Sunil.
Apartment purchased in the name of Ms. Sonali- The property has been purchased
by Rohit in the name of his daughter Ms. Sonali. Therefore, although the
consideration for the purchase has been made by Mr. Rohit, since this is covered by
the exemtion provided (since the property has been purchased in the name of his
child), it is not covered as a benami transaction.
Apartment purchased in the name of Mr. Srinivas Rajan- A benami transaction is
defined as a transaction where a property is transferred to or held by one person and
consideration is paid by some other person. In this case, the property is in the name
of Srinivas Rajan although the consideration is paid by Rohit. Therefore, this is a
benami transaction.
Apartment purchased jointly in the name of Rohit and his brother Sunil- A property
jointly held in the name of brother and they appear as joint owners. Hence, this is not
a benami transaction.

© The Institute of Chartered Accountants of India


12 FINAL (NEW) EXAMINATION: MAY 2019

First two flats were rented to a large multi-national bank. After 4 years, Mr, Srinivas
Rajan transferred the property back in the name of Mr. Rohit without any
consideration.
As per section 6 of the Prohibition of Benami Property Transactions Act, 1988, no
person, being a benamidar shall re-transfer the benami property held by him to the
beneficial owner or any other person acting on his behalf. In cases where any property
is re-transferred, then such a transaction of a property shall be deemed to be null and
void.
In the said above case transaction of transfer back of the apartment by Mr. Srinivas
Rajan to Mr. Rohit is void.
(ii) The Initiating Officer, who is probing the transactions relating to the INR 2 crores
received and spent by Mr. Rohit, seeks your advice on identifying the benami
properties/ transactions, the benamidars, the beneficial owner.
The following are the benami transactions and benamidars:
Transaction Benamidar
Receipt of INR 2 crores in the bank Good Phones
account of Good Phones
Transfer of INR 2 crores from the Mr. Rohit (person abetting the
bank account of Good Phones to Mr. transaction, Mr. Srinivas Rajan)
Rohit’s personal bank account
Acquisition of shares of Stay Shares of Stay Connected becomes
Connected using the benami money benemi property
Purchase of Jewellery as gift for Ms. The jewellery becomes benami property.
Anjali Writer Mrs. Anjali also becomes Benamidar.
The original beneficial owner is not identified- the person who should have been the
original recipient of the funds, may be the original beneficial owner, although these
funds are not being held by Rohit/ Anjali for their benefit.
(iii) According to section 2(16) of the Prohibition of Benami Property Transaction Act,
1988, fair market value", in relation to a property, means—
(1) the price that the property would ordinarily fetch on sale in the open market on
the date of the transaction; and
(2) where the price referred to in sub-clause (i) is not ascertainable, such price as
may be determined in accordance with such manner as may be prescribed in
Rule 3 of the Prohibition of Benami Property Transaction Rules, 2016.
As per the said Rule, the price of unquoted equity shares shall be the higher of-
(i) its cost of acquisition;
(ii) the fair market value of such equity shares determined, on the date of

© The Institute of Chartered Accountants of India


PAPER – 6D: ECONOMIC LAWS 13

transaction, by a merchant banker or an accountant as per the Discounted Cash


Flow method; and
(iii) the value, on the date of transaction, of such equity shares as determined by
the formula given in the Rules.
The value of (iii) above is determined as below:
Particulars Amount Value to be Remarks
(` In considered for
Crores) calculation
(` In Crores)
Immovable Property 5.00 8.00 Market value to be
considered
Other fixed assets 4.00 4.00 Book value net of
depreciation
Inventories 2.00 2.00 Book value
Receivables and Loans and 1.50 1.50 Book value
Advances
Deferred Advertisement Costs 0.50 0.00 Not to be considered
Advance tax paid 1.00 0.00 Not to be considered
Total Value of Assets 15.50
Shareholders’ funds -4.00 0.00 Share capital and
Reserves not to be
considered
Provision for taxation -0.50 0.00 Not to be considered
Loans from Banks -3.00 -3.00
Trade Payables -6.50 -5.50 Provision for
unascertained
liabilities not to be
considered
Contingent Liabilities -2.00 -0.50 Arrears of divided on
cumulative preference
shares to be
considered
Proposed dividend on equity -1.00 -0.00 Not to be considered
shares
Total Value of Liabilities -9.00
Fair Market Value (Asset- 6.50
Liabilities) *Paid up Equity

© The Institute of Chartered Accountants of India


14 FINAL (NEW) EXAMINATION: MAY 2019

Capital/ Paid up value of equity


shares
Value of equity shares 1.30
acquired i.e. 20% of total

In the said question, the cost of acquisition is assumed at ` 2.00 crores, the value,
on the date of transaction, of such equity shares as determined by the formula given
in the Rules is ` 1.30 crores and the market value based on discounted cash flow
method is given as ` 2 crores. Thus, the fair market value of the acquisition in M/s.
Stay Connected will be ` 2 crores being highest of above and ` 10 crores for the
company as a whole (i.e. 2.00/0.20).
(iv) As per section 24 of the Prohibition of Benami Property Transactions Act, 1988, where
the Initiating Officer, on the basis of material in his possession, has reason to believe
that any person is a benamidar in respect of a property, he may, issue a show cause
notice to the person.
Where the notice specifies any property as being held by a benamidar, a copy of the
notice shall also be issued to the beneficial owner if his identity is known.
Where the Initiating Officer is of the opinion that the person in possession of the
property held benami may alienate the property during the period specified in the
notice, he may, with the previous approval of the Approving Authority, by order in
writing, attach provisionally the property in the manner as prescribed in Rule 4 of the
Benami Transactions Prohibition Rules, 2016, for a period not exceeding ninety days
from the date of issue of notice.
The Initiating Officer, after making such inquires and calling for such reports or
evidence as he deems fit and taking into account all relevant materials, shall, within
a period of ninety days from the date of issue of notice -
(i) pass an order continuing the provisional attachment of the property with the prior
approval of the Approving Authority, till the passing of the order by the
Adjudicating Authority; or
(ii) revoke the provisional attachment of the property with the prior approval of the
Approving Authority;
where provisional attachment has not been made, pass an order provisionally
attaching the property with the prior approval of the Approving Authority, till the
passing of the order by the Adjudicating Authority; or decide not to attach the property
as specified in the notice, with the prior approval of the Approving Authority.
Where the Initiating Officer passes an order continuing the provisional attachment of
the property or passes an order provisionally attaching the property, he shall, within
fifteen days from the date of the attachment, draw up a statement of the case and
refer it to the Adjudicating Authority.

© The Institute of Chartered Accountants of India


PAPER – 6D: ECONOMIC LAWS 15

CASE STUDY 2
Winner Builders Private Limited ("Winner") is a premium real estate builder who specializes in
constructing mid-sized apartment complexes (20 - 40 apartments) in South India. Winner was
started in the year 2004 by Mr. Vijay Nair, Managing Director and has its head office in Kochi,
Kerala with branches in Trivandrum, Bengaluru, Chennai and Tirupati. Mr. Vijay Nair has been
in the real estate business for more than 30 years and comes from a family of civil engineers
who are highly respected by their customers. Mr. Arun Nair, son of Mr. Vijay Nair, is a Chartered
Accountant and is the Chief Financial Officer of Winner. Mr. Vijay and Mr. Arun together own 60
of the share capital of Winner and the balance is held by a large private equity investor.
Although the company is a private limited company, the affairs of the company are handled in
the most professional manner akin to a listed company and Mr. Arun ensures that the financial
statements are properly prepared and presented to the Board of Directors (Mr. Vijay, Mr. Arun
and a representative of the PE investor) on a quarterly basis. The financial performance of
Winner has been reasonable and being a conservative person, Mr. Vijay was never in the mind-
set of taking aggressive positions with regard to business. Over the last few months, the PE
investor has been pushing the company in making changes in the operational mechanism, sale
prices etc. to increase the profits of the company and ensure decent return on their investment.
Due to this, Mr. Vijay and Mr. Arun are under tremendous pressure to complete the ongoing
projects fast and start new projects immediately and increase the revenues / profits of the
Company.
In June 2018, Winner announced a new 80 apartment project in Kochi named as "Winner
Shikaram", an ultra-modem luxury apartment complex with a variety of amenities including
swimming pool, skating rink, basketball court, fully equipped club house with all amenities, etc.
As per RERA regulations the Company applied for registration of the project on 15 th June 2018.
On 20th June 2018, the Company announced the launch of the project and commenced a big
advertisement campaign in the TV media and also through release of promotion material
through social media. It also collaborated with a regional TV Channel and announced a free
home in "Winner Shikaram" for the first prize winner of a popular reality show. The property was
registered by RERA on 10th July 2018 after scrutiny of the information provided by the promoter.
Based on the past performance of the Winner group and the general image of Mr. Vijay Nair,
there was tremendous demand for the apartments in the project and all the apartments were
booked within 1 month from the date of launch (20 th June, 2018). The following were some of
the conditions mentioned in the agreement to sale entered into by Winner with its allottees:
1. Expected date of completion of construction -31st March, 2020.
2. Expected date of handover-31s t May, 2020, subject to a grace period of 4 months.
3. Booking Advance amount to be paid prior to entering into agreement to sale – 20% of
total cost of apartment
4. Open car parking cost-INR 200,000

© The Institute of Chartered Accountants of India


16 FINAL (NEW) EXAMINATION: MAY 2019

5. Any delay in payment of dues by the allottees will liable for interest on such delayed
payments.
6. Return of booking amount shall not be entertained for any reason whatsoever.
7. Winner Group shall be liable for any deficiency in quality of construction for a period
of 3 years from the date of handing over the apartments.
Winner Group collected a total amount of INR 80 crores from the allottees and deposited an
amount of INR 60 crores in an escrow account for exclusive use for construction of the complex.
Separately, an amount of INR 5 lakhs each was collected from the 80 allottees in cash,
aggregating to INR 400 lakhs towards interior work, modular kitchen, supplying fans and lights,
etc. This money was accounted as receipt in a separate company, M/s. Wonderful Interiors,
which was owned by Ms. Anusha Nair, daughter of Mr. Vijay Nair and Mr. Arun Nair.
Although the construction was proceeding apace, the Company encountered severe rock
formations under the ground in one section of the land area which was previously not known
and due to the same, the Company concluded that the swimming pool could not be constructed
as designed and the size of the same had to be reduced. Winner got in' touch with the allottees
and proposed that the reduction of the size of swimming pool will be compensated suitably by
Winner by providing a Jacuzzi and Spa inside the club house. This was accepted by majority
(45 of the 80) of the allottees and, accordingly, Winner proceeded with the construction based
on the amended plan.
Few of the allottees reached out to Mr. Vijay Nair and stated that the carpet area for their
apartments was lesser than the size stipulated in the sale agreement and therefore, wanted to
be compensated. Mr. Vijay Nair mentioned to them that the reduction in the area was on account
of the exterior walls appurtenant to their apartments and this is the case with all the apartments
and not specific to their homes alone.
Mr. Arun Nair attended one of the real estate conclaves held in Bangalore, in which he met one
Mr. Henry Stewart, who runs an interior designing warehouse in Dubai UAE and showed quite
a few exhibits to Arun. Arun was impressed by the designs and the prices quoted by Mr. Henry.
Mr. Henry was also amenable to receive funds in cash in India through an intermediary and then
provide the material to Arun from UAE. Based on the same, Arun arranged for making cash
payment to the extent of INR 200 lakhs (Out of the INR 400 lakhs received by M/s Wonderful
Interiors) to an intermediary in Delhi, and the material was received from Henry in a month.
During his visit to India, Henry noted that his UAE passport got expired and he did not realise
the same. Since he did wanted to leave India immediately, he got in touch with a travel agent,
Mr. Anil Kumar, who helped him get a forged passport, for which Mr. Henry paid INR 2 lakhs in
cash.
Out of the balance INR 200 lakhs cash available with Wonderful Interiors, Arun used cash
amounting to INR 25 lakhs to pay amounts to various intermediaries to facilitate timely and
smooth registration process of thie apartments of Winner Shikharam, which was paid by the

© The Institute of Chartered Accountants of India


PAPER – 6D: ECONOMIC LAWS 17

intermediaries to the officials of the Sub- Registrar. With Henry's help, Arun transferred the
balance amount of INR 175 lakhs to an intermediary in Delhi and invested the amount to
incorporate a shell company in the Cayman Islands. The funds were then transferred back by
the Shell Company to the bank account of Winner. For this purpose, Mr. Arun raised export
invoices in the books of Winner on the Shell Company for providing professional services
relating to real estate business. Based on these invoices, Winner claimed export incentives
under the relevant laws in India and received INR 20 lakhs as export incentive.
On 30th March 2019, a meeting was organised by the Company and all the allottees during which
Mr. Vijay Nair provided a status update on the project and stated that bulk of the construction
activities will be completed by the timeline mentioned in the sale agreement (31 st May, 2020)
and the apartments will be handed over by 31 st July, 2020 (i.e. within the grace period). The
common areas will be completed in parallel and handed over by 30 th September, 2020. The
slight delay in completion was on account of non-availability of quality labour and he wanted
only the best labour to work on the project to ensure that the home owners have a happy life
after hand over. He also mentioned that the labour rates increased by 15% after the sale
agreements were entered and the Company did not ask for increase in prices from the allottees
only for good will reasons. The allottees were unhappy with the delay but, accepted the same,
since there was no other choice.
As one of the shareholders of Wonderful Interiors, Ms. Anusha Nair decided to visit Dubai to
see the interior designs and then place an order for the upcoming projects. During her visit, she
purchased 500 grams worth of gold (costing INR 15 lakhs) and since, she did not have enough
money, she asked Mr. Arun Nair to make the payment through the intermediary in Delhi. Based
on the discussion with the intermediary, Mr. Arun Nair provided an antique painting which he
got from one of his social friends to the intermediary as consideration for the gold purchased by
Ms. Anusha Nair in Dubai. Based on the same, Ms. Anusha brought the gold with her through
the green channel.
One of the employees of Wonderful Interiors, noting the substantial amount of cash transactions,
informs the Bank regarding the same, which in turn informs the enforcement directorate. The
ED has issued a show cause notice to all the parties regarding the above transactions.
Answer the following questions:
2.1 RERA authorities sent a notice to Winner that their advertisement campaign was not
in accordance with the RERA 2016. Evaluate.
(A) Valid, Since Winner decided to use Social media platform for promotion, without
obtaining specific approval from RERA.
(B) Valid, Since Winner collaborated with a TV channel to give a free home in Winner
Shikaram when the construction itself was not complete.
(C) Valid, Since Winner launched the project and commenced marketing even before the
project received registration from RERA.

© The Institute of Chartered Accountants of India


18 FINAL (NEW) EXAMINATION: MAY 2019

(D) Not Valid, Since Winner applied for the registration prior to the launch of campaign
and the registration was ultimately received within the stipulated period. (2 Marks)
2.2 As per RERA, Winner is required to enable the formation of the association of allottees
of Winner Shikaram within-months.
(A) 3 months of the majority of the allottees having booked their apartment.
(B) 3 months of the receipt of occupancy certificate.
(C) 3 months of the majority of the allottees registering their apartments with the sub-
registrar.
(D) 3 months of all the allottees making the full payment for the apartments. (2 Marks)
2.3 After registering the apartments in the name of the allottees, Winner informed the
allottees that they need to pay the water and electricity charges to the concerned
departments for their apartments. Evaluate.
(A) The registration of the apartments denote that the allottees are now the legal owners
of the apartments and hence, need to bear the water and electricity charges.
(B) The promoter is liable for making payment for the water and electricity charges until
the physical possession is transferred to the allottees.
(C) This is dependent on the terms of the agreement of sale between Winner and the
allottees.
(D) This amount need to be paid equally by Winner and the allottees, since the
registration is completed and only transfer of physical possession is pending.
(2 Marks)
2.4 The time limit within which the allottees of winner Shikaram are required to take
physical possession of the apartment after issuance of occupancy certificate is:
(A) Three months
(B) One month
(C) Five months
(D) Two months (2 Marks)
2.5 As per provisions of RERA, collection of cash by Wonderful Interiors for interior work,
modular kitchen, supplying fans and lights, etc. :
(A) May be appropriate, since RERA does not specify the mode of collection.

© The Institute of Chartered Accountants of India


PAPER – 6D: ECONOMIC LAWS 19

(B) May not be appropriate, since collection should be done as per the stipulations of
RERA.
(C) May be appropriate, since provisions of RERA are not applicable.
(D) May not be appropriate, since Wonderful Interiors are not registered with RERA.
(2 Marks)
2.6 What are the three distinct stages of Money Laundering?
(A) Information, Interrogation, Indictment
(B) Placement, Layering, Integration
(C) Planning, Comingling, Profiting
(D) Monitoring, Adjudicating, Punishing (2 Marks)
2.7 Which of the following are not circumstances which need to be considered by the
Director of Enforcement for performing search of the offices of Winner and other parties
mentioned in the case study?
(A) Possession of any property related to crime
(B) Possession of any records relating to money laundering
(C) Possession of records relating to RERA compliance by Winner
(D) Possession of any proceeds of crime involved in money laundering (2 Marks)
2.8 Ms. Anusha Nair brought gold jewellery worth INR 15 lakhs from D ubai through the
green channel. Is this an offence under the PMLA 2002 ?
(A) Yes, because she came through the green channel and evaded duty of customs.
(B) No, whilst it is an offence, it is not actionable under the PMLA 2002.
(C) No, she did not pay any cash for the purchase.
(D) Yes, since purchase of gold from gulf countries requires specific consent as per the
agreement entered with foreign countries as per Section 56 of PMLA 2002. (2 Marks)
2.9 As per RERA 2016, what is the minimum amount that Winner was required to deposit
in the escrow account ?
(A) INR 50 crores
(B) INR 56 crores
(C) INR 54 crores

© The Institute of Chartered Accountants of India


20 FINAL (NEW) EXAMINATION: MAY 2019

(D) INR 58.8 crores (2 Marks)


2.10 Of the below, which of the practices are not common schemes of money laundering?
(A) Bribery and Corruption
(B) False declarations under Customs act
(C) Usage of false trade Marks/copyrights
(D) Possession of foreign currency over and above permitted limit (2 Marks)
2.11 Answer the following questions in the context of the prov isions relating to the Real
Estate (Regulation & Development) Act, 2016 (RERA 2016).
(i) Examine the appropriateness of the conditions mentioned in the agreement to sale,
in the context of the provisions of RERA 2016. (4 Marks)
(ii) What are the provisions in RERA 2016 relating to the changes in design of the
construction from the sanctioned plans ? Analyse if the changes made by Winner are
appropriate in this context. (4 Marks)
(iii) What would be your advice if the customers of Winner reach out to you for your views
with regard to the validity of the explanations provided by Mr. Vijay Nair on the
reduction of carpet area? (3 Marks)
(iv) Evaluate the statements made by Mr. Vijay Nair in the meeting with the allottees on
30th March 2019 regarding the delay and the increase in labour costs in the context
of provisions of RERA 2016. (4 Marks)
2.12 Examine / advice regarding the below questions relating to the Prevention of Money
Laundering Act, 2002 (PMLA 2002).
(i) As a leading consultant on PMLA matters, the enforcement directorate has sought
your advice on identifying :
(a) the offences (b) the parties involved and (c) the punishment for the offence of
money laundering. (8 Marks)
(ii) The Bank, in which Winner holds its bank account, has reached out to you to
understand their obligations for maintaining and reporting of transactions under the
PMLA 2002. Advise. (4 Marks)
(iii) When the Enforcement Directorate proposed to take action against Mr. Vijay Nair
under the PMLA 2002, Mr. Vijay Nair contended that he was not a party to any of the
alleged offences and he was managing the real estate business of Winner only.

© The Institute of Chartered Accountants of India


PAPER – 6D: ECONOMIC LAWS 21

Examine whether his statement is valid. What would be the position of t he nominee
director of the PE investor? (3 Marks)

Answer Case study 2


2.1 (C)
2.2 (A)
2.3 (B)
2.4 (D)
2.5 (C)
2.6 (B)
2.7 (C)
2.8 (B)
2.9 (B)
2.10 (C)
2.11 (i) 1. Expected date of completion of construction- 31st March, 2020 -This
condition is valid.
2. Expected date of handover- 31st May 2020, subject to a grace period of 4
months. -This condition is valid.
3. According to Section 13, a promoter shall not accept a sum more than ten
per cent of the cost of the apartment, plot, or building as the case may be,
as an advance payment or an application fee, from a person without first
entering into a written agreement for sale with such person and register the
said agreement for sale, under any law for the time being in force.
Hence, the condition in the agreement for sale for booking advance amount
to be paid prior to entering into agreement to sale @20% of total cost of
apartment is not valid.
4. Section 2(n) of RERA, 2016 defines ‘common areas’ to include ‘open
parking areas’, thus open parking areas cannot be sold to the allottees.
Hence, the condition in the agreement for sale for open car parking cost
` 2,00,000 is not valid.
5. As per section 19(7) of RERA, 2016, the allottee shall be liable to pay
interest, at such rate as may be prescribed, for any delay in payment
towards any amount or charges to be paid.
Hence, the condition about any delay in payment of dues by the allottees
will be liable for interest on such delayed payments, is valid.

© The Institute of Chartered Accountants of India


22 FINAL (NEW) EXAMINATION: MAY 2019

6. The allottee shall be entitled to claim the refund of amount paid along with
interest at such rate as may be prescribed and compensation in the manner
as provided under this Act, from the promoter, if the promoter fails to
comply or is unable to give possession of the apartment, plot or building,
as the case may be, in accordance with the terms of agreement for sale or
due to discontinuance of his business as a developer on account of
suspension or revocation of his registration under the provisions of this Act
or the rules or regulations made thereunder.
Hence, the condition for return of booking amount shall not be entertained
for any reason whatsoever is not valid.
7. The builder has to provide five-year warranty for any structural defects in
the building. They are liable to pay equal rate of interest in case of default
or delays as home buyers.
Hence, the condition that Winner Group shall be liable for any deficiency
in quality of construction for a period of 3 years from the date handing over
the apartments is not valid.
(ii) Adherence to sanctioned plans and project specifications by the promoter
(Section 14)
(1) The proposed project shall be developed and completed by the promoter
in accordance with the sanctioned plans, layout plans and specifications
as approved by the competent authorities.
(2) Notwithstanding anything contained in any law, contract or agreement,
after the sanctioned plans, layout plans and specifications and the nature
of the fixtures, fittings, amenities and common areas, of the apartment, plot
or building, as the case may be, as approved by the competent authority,
are disclosed or furnished to the person who agree to take one or more of
the said apartment, plot or building, as the case may be, the promoter shall
not make—
(i) any additions and alterations in the sanctioned plans, layout plans
and specifications and the nature of fixtures, fittings and amenities
described therein in respect of the apartment, plot or building, as the
case may be, which are agreed to be taken, without the previous
consent of that person.
Provided that the promoter may make such minor additions or
alterations as may be required by the allottee, or such minor changes
or alterations as may be necessary due to architectural and structural
reasons duly recommended and verified by an authorised Architect or
Engineer after proper declaration and intimation to the allottee.
Explanation.—For the purpose of this clause, "minor additions or

© The Institute of Chartered Accountants of India


PAPER – 6D: ECONOMIC LAWS 23

alterations" excludes structural change including an addition to the


area or change in height, or the removal of part of a building, or any
change to the structure, such as the construction or removal or cutting
into of any wall or a part of a wall, partition, column, beam, joist, floor
including a mezzanine floor or other support, or a change to or closing
of any required means of access ingress or egress or a change to the
fixtures or equipment, etc.
(ii) any other alterations or additions in the sanctioned plans, layout plans
and specifications of the buildings or the c ommon areas within the
project without the previous written consent of at least two-thirds of
the allottees, other than the promoter, who have agreed to take
apartments in such building.
Explanation.—For the purpose of this clause, the allottees,
irrespective of the number of apartments or plots, as the case may
be, booked by him or booked in the name of his family, or in the case
of other persons such as companies or firms or any association of
individuals, etc., by whatever name called, booked in its name or
booked in the name of its associated entities or related enterprises,
shall be considered as one allottee only.
(3) In case any structural defect or any other defect in workmanship, quality or
provision of services or any other obligations of the promoter as per the
agreement for sale relating to such development is brought to the notice of
the promoter within a period of five years by the allottee from the date of
handing over possession, it shall be the duty of the promoter to rectify such
defects without further charge, within thirty days, and in the event of
promoter's failure to rectify such defects within such time, the aggrieved
allottees shall be entitled to receive appropriate compensation in the
manner as provided under this Act.
In the instant case, the proposal of Winner for reduction of the size of
swimming pool and the same to be compensated by providing a Jacuzzi
and spa inside the club house was accepted by majority (45 of the 80) of
the allottees and accordingly, Winner proceeded with the construction
based on the amended plan.
According to the above provisions, the promoter shall not make any other
alterations or additions in the sanctioned plans, layout plans and
specifications of the buildings or the common areas within the project
without the previous written consent of at least two-thirds of the allottees,
other than the promoter, who have agreed to take apartments in such
building.
Hence, approval by majority (45 of 80) is not valid.

© The Institute of Chartered Accountants of India


24 FINAL (NEW) EXAMINATION: MAY 2019

(iii) As per section 2(k) of the Real Estate (Regulation & Development)Act, 2016
"carpet area" means the net usable floor area of an apartment, excluding the
area covered by the external walls, areas under services shafts, exclusive
balcony or verandah area and exclusive open terrace area, but includes the area
covered by the internal partition walls of the apartment.
Accordingly, Sale of property will be on carpet area, not super built area.
Therefore, the homebuyer will have to pay only for the carpet area, that is the
area within walls, and the builder cannot charge for the super built-up area.
Therefore, the explanations provided by Mr. Vijay Nair on the reduction of the
carpet area was invalid. So, home buyers/ customers are liable to pay only for
the carpet area, that is the area within walls.
(iv) As given in the question that on 30th March, 2019, meeting was organized by
the company with all the allottees. During the meeting, Mr. Vijay Nair provided
a status update on the project and of the construction activities to be completed
and the other information mentioned in the sale agreement. As per the Section
11 of Real Estate (Regulation & Development) Act, 2016, it is the duty of the
promoter, to alter a project plan, structural design and specifications of the plot,
apartment or a building, the promoter has to get the consent of minimum two-
third allottees (buyers) after the necessary disclosures.
Since in the given case no approval of 2/3rd of the allottees was taken w.r.t. to
delay and the increase in labour costs i.e., as to the updation of the status of the
said project. This act of Mr. Vijay Nair is not in compliance with the Law.
2.12 (i) (a) Offences: The term “scheduled offence" has been defined in clause (y) of sub-
section (1) of section 2. It means –
(a) the offences specified under Part A of the Schedule; or
(b) the offences specified under Part B of the Schedule if the total value
involved in such offences is one crore rupees or more; or
(c) The offences specified under Part C of the Schedule.
The Schedule to the Act gives a list of all the above offences. The Schedule is
divided into three parts- Part A, Part B and Part C, which are given in Annexure
to the Chapter.
(b) The parties involved: Clause (p) of sub section (1) of section 2 provides that
"money-laundering" has the meaning assigned to it in section 3. Moving to
section 3, it is observed that whosoever directly or indirectly attempts to indulge
or knowingly assists or knowingly is a party or is actually involved in any process
or activity connected with the proceeds of crime including its concealment,
possession, acquisition or use and projecting or claiming it as untainted property
shall be guilty of offence of money laundering.

© The Institute of Chartered Accountants of India


PAPER – 6D: ECONOMIC LAWS 25

(c) The punishment for the offence of money laundering: Section 4 provides for the
Punishment for Money-Laundering - Whoever commits the offence of money-
laundering shall be punishable with rigorous imprisonment for a term which shall
not be less than three years but which may extend to seven years and shall also
be liable to fine.
But where the proceeds of crime involved in money-laundering relate to any
offence specified under paragraph 2 of Part A of the Schedule (i.e. Offences
under the Narcotic Drugs and Psychotropic Substances Act, 1985), the
maximum punishment may extend to ten years instead of seven years.
(ii) Obligation of Banking Companies, Financial Institutions and Intermediaries
Reporting entity to maintain records
Section 12 provides for the obligation of Banking Companies, Financial Institutions
and Intermediaries.
1. Maintenance of records: According to sub-section (1), every reporting entity
shall –
(a) maintain a record of all transactions, including information relating to
transactions covered under clause (b), in such manner as to enable it to
reconstruct individual transactions;
(b) furnish to the Director within such time as may be prescribed, information
relating to such transactions, whether attempted or executed, the nature
and value of which may be prescribed;
(c) verify the identity of its clients in such manner and subject to such
conditions, as may be prescribed;
(d) identify the beneficial owner, if any, of such of its clients, as may be
prescribed;
(e) maintain record of documents evidencing identity of its clients and
beneficial owners as well as account files and business correspondence
relating to its clients.
2. Confidentiality: Every information maintained, furnished or verified, save as
otherwise provided under any law for the time being in force shall be kept
confidential.
3. Maintenance of records (for clause a): The records referred to in clause (a)
of sub-section (1) shall be maintained for a period of five years from the date of
transaction between a client and the reporting entity.
4. Maintenance of records (for clause e): The records referred to in clause (e)
of sub-section (1) shall be maintained for a period of five years after the business
relationship between a client and the reporting entity has ended or the account

© The Institute of Chartered Accountants of India


26 FINAL (NEW) EXAMINATION: MAY 2019

has been closed, whichever is later.


5. Exemption by the Central Government: The Central Government may, by
notification, exempt any reporting entity or class of reporting entities from any
obligation under this chapter.
Access to information [Section 12A]
1. The Director may call for from any reporting entity any of the records referred to
in sub-section (1) of section 12 and any additional information as he considers
necessary for the purposes of this Act.
2. Every reporting entity shall furnish to the Director such information as may be
required by him under sub-section (1) within such time and in such manner as
he may specify.
3. Save as otherwise provided under any law for the time being in force, every
information sought by the Director under sub-section (1), shall be kept
confidential.
(iii) Offences by companies [Section 70]
1. Where a person committing a contravention of any of the provisions of this Ac t
or of any rule, direction or order made thereunder is a company, every person
who, at the time the contravention was committed, was in charge of and was
responsible to the company, for the conduct of the business of the company as
well as the company, shall be deemed to be guilty of the contravention and shall
be liable to be proceeded against and punished accordingly.
Nothing contained in this sub-section shall render any such person liable to
punishment if he proves that the contravention took place without his knowledge
or that he exercised all due diligence to prevent such contravention.
2. Notwithstanding anything contained in sub-section (1), where a contravention of
any of the provisions of this Act or of any rule, direction or order made
thereunder has been committed by a company and it is proved that the
contravention has taken place with the consent or connivance of, or is
attributable to any neglect on the part of any director, manager, secretary or
other officer of any company, such director, manager, secretary or other officer
shall also be deemed to be guilty of the contravention and shall be liable to be
proceeded against and punished accordingly.
In the instant case, Mr. Vijay Nair contended that he was not a party to any of
the alleged offences and he was managing the real estate business of Winner
only.
His statement is not valid on the grounds of section 70 of the PMLA 2002.
The position of the nominee director of the PE investor would be same as of

© The Institute of Chartered Accountants of India


PAPER – 6D: ECONOMIC LAWS 27

Mr. Vijay Nair and he shall be deemed to be guilty of the contravention and shall
be liable to be proceeded against and punished accordingly.
CASE STUDY 3
(Part-A)
The appellant "M/s Transmission Corporation of Andhra Pradesh Limited" is a successor of
Andhra Pradesh State Electricity Board (for short, 'APSEB') and is in the activities relating to
transmission of electricity. It had awarded certain contracts to the respondent "M/s Equipment
Conductors & Cables Limited" herein for supply of goods and services. Some disputes arose
and the respondent initiated arbitration proceedings. As many as 82 claims were filed by the
respondent before Haryana Micro and Small Enterprises Facilitation Council (hereinafter
referred to as 'Arbitral Council'). These proceedings culminated into Award dated June 21 st,
2010. The Arbitral Council came to the conclusion that the claims made on the basis of Invoice
Nos. 1 - 57 were barred by law of limitation and, therefore, no amount could be awarded against
the said claims. In respect of Invoice Nos. 58-82, the award was passed in favour of the
respondent. Against the aforesaid award rejecting claims in respect of Invoice Nos. 1 - 57 as
time barred, the respondent herein filed an application under Section 34 of the Arbitration and
Conciliation Act before the Additional District Judge, Chandigarh. The Additional District Judge
passed the order dated August 28, 2014 in the said application thereby remanding the case
back to the Arbitral Council for fresh decision. Against this order, the appellant filed the appeal
before the High Court of Punjab and Haryana at Chandigarh. This appeal was allowed by the
High Court by its order dated January 29, 2016 thereby setting aside the direction of the
Additional District Judge remanding the matter to Arbitral Council for fresh consideration.
The respondent herein filed execution petition for execution of judgment dated January 29, 2016
passed by the High Court of Punjab and Haryana as well as the award dated June 21, 2010
passed by the Arbitral Council. In so far as award of Arbitral Council is concerned, as noted
above the respondent's claim pertaining to Invoice Nos. 58 - 82 was allowed and the execution
thereof was sought. The respondent, however, filed another execution petition seeking
execution of amount in respect of Invoice Nos. 1 - 57 also. This application was entertained and
both the petitions were directed to be dealt with simultaneously vide orders dated August 17,
2016. The High Court vide its order dated November 08,2016 allowed the said Revision Petition
holding that there was no award in respect of claim towards Invoice Nos. 1 - 57 and, therefore,
it was not permissible for the respondent to seek the execution. When the things rested at that,
the respondent approached the NCLT by means of a Company Petition under Section 9 of IBC,
2016 read with Rule 6 of Insolvency and Bankruptcy (AAA) Rules, 2016. In this petition, the
respondent stated that it had served demand notice dated October 14, 2017 upon the appellant
under the provisions of the IBC, thereby claiming the amount of ` 45,69,31,233/- which was not
paid by the appellant. As mentioned above, this petition was dismissed by the NCLT filed under
section 9 of IBC vide its order dated April 09, 2018 being non maintainable on account of
existence of a dispute between the parties and this assertion of the NCLT is based on the fact
that these very claims of the respondent were subject matter of arbitration and the award was
passed rejecting these claims as time barred. Against this order, the respondent has filed appeal

© The Institute of Chartered Accountants of India


28 FINAL (NEW) EXAMINATION: MAY 2019

before the NCLAT in which impugned orders dated September 04,2018 have been passed. The
Honourable NCLAT passed an order stating "Prima facie case has been made out by the
Appellant in view of the part decree awarded by the competent court under Section 34 of the
Arbitration and 'Conciliation Act, 1996. However, taking into consideration the fact that if appeal
is allowed and Corporate Insolvency Resolution Process is initiated against the Respondent -
"Transmission Corporation of Andhra Pradesh Ltd. ", the government undertaking may face
trouble. Therefore, by way of last chance we grant one opportunity to respondents to settle the
claim with the appellant, failing which this Appellate Tribunal may pass appropriate order on
merit."
This very order of the National Company Law Appellate Tribunal, (for short, 'NCLAT) dated
September 04, 2018 is the subject matter of challenge before the Honourable Supreme Court
by the appellant M/s Transmission Corporation of Andhra Pradesh Limited and prays that the
same be reversed as there exists a Dispute and the application under IBC cannot be accepted.
(Part-B)
One Shri Rajendra Singh (Informant') filed an information under Section 19(1)(a) of the
Competition Act, 2002 (the 'Act') against Ghaziabad Development Authority (‘OP' /'GDA')
alleging contravention of the provisions of Section 4 of the Act. As per the information, the
Informant is an allottee of a flat under the Adarsh Vihar residential housing scheme for the
Economically Weaker Sections (EWS) ('Scheme') being developed by the OP in Ghaziabad, U.
P. in 2008. It is informed that OP is constituted under Section 4 of the Urban Planning and
Development Act, 1973 of Uttar Pradesh and is, inter alia, engaged in the activity of
development and sale of real estate in Ghaziabad, U. P.
It is further stated that the OP had conducted a lottery draw for allotment of EWS flats under the
aforesaid scheme. On being successful in the said lottery draw, the Informant was allotted a flat
bearing no. A-1/222 and accordingly, an allotment letter dated 04.05.2009 was issued in favour
of the Informant mentioning the final price of the flat as ` 2,00,000 and other conditions relating
to payment plan, date of giving possession, penal interest in case of delay in the payment of the
balance amount etc. As per the condition of the scheme, the Informant paid
` 20,000 to the OP as registration amount constituting 10% of the total price of the said house.
It is averred by the Informant that on 27.11.2015, the OP issued a letter to all the allottees of
the aforesaid scheme asking them to pay ` 7,00,000 as sale price of the flats allotted to them
failing which their allotment would stand cancelled. It is alleged that the OP has arbitrarily
increased the sale price of the said flat to ` 7,00,000 from ` 2,00,000 which was mentioned in
the allotment letter dated 04.05.2009. As per the Informant, the OP has indulged in unfair and
arbitrary practices and has misused its dominant position in the market. It is further averred that
the OP has indulged in the said practice even after knowing that the allottees of the scheme
belong to the weaker sections of the society and they are not in a position to challenge the OP
for its unfair and arbitrary conduct. Further, it is stated that the allottees of the said scheme are
dependent upon the OP for the residential flats under the said scheme. The Informant has

© The Institute of Chartered Accountants of India


PAPER – 6D: ECONOMIC LAWS 29

averred that the alleged conduct of OP is in contravention of the provisions of Section 4(2)(a)
of the Act.
Based on the above submissions, the Informant has prayed before the Commission to initiate
an inquiry against the OP under the provisions of the Act, set aside the impugned letter dated
27.11.2015 of the OP demanding ` 7,00,000 for the aforementioned flat, and direct the OP to
deliver possession of the flat to the informant under the said scheme at the price of ` 2,00,000
per flat. Besides hearing the parties on 27.12.2016, the Commission has perused the
information available on record and the documents submitted by the OP. From the facts of the
case, it appears that the grievance of the Informant relates to the letter dated 27.11.2015 of the
OP demanding a higher price of ` 7,00,000 for a EWS flat allotted to the Informant under the
aforesaid scheme as compared with the price of ` 2,00,000 as declared in the scheme's initial
brochure and intimated to the Informant vide allotment letter dated 04.05.2009. It is the case of
the Informant that the OP has abused its dominant position by arbitrarily increasing the price of
the said flat in contravention of the provisions of Section 4 of the Act.
The Commission observes that GDA is established under Section 4 of the Urban Planning and
Development Act, 1973 of the State of Uttar Pradesh. It has a common seal with power to
acquire, hold or dispose of both movable and immovable properties. The Urban Planning and
Development Act, 1973 of Uttar Pradesh empower GDA to pursue activities for promoting and
securing development of Ghaziabad in a planned manner. GDA has the power to acquire, hold,
manage and dispose of land and other properties in Ghaziabad and to carry out building,
engineering, mining and other operations, etc. Further, GDA is, inter alia, engaged in the
activities of development and sale of buildings, flats, complexes etc. for residential, commercial,
institutional and other purposes and with regard to the relevant geographic market. The
Commission is of the view that the geographic area of Ghaziabad district of the State of Uttar
Pradesh exhibits homogeneous and distinct market conditions for the development and sale of
low cost residential flats under affordable housing schemes for EWS and can be distinguished
from the conditions of competition prevailing in other adjacent areas of Ghaziabad such as Delhi,
Noida, etc. It may be noted that a consumer intending to buy a low cost residential flat under
affordable housing scheme for EWS in Ghaziabad may not prefer to purchase the same in other
adjacent areas of Ghaziabad because of factors such as difference in regulatory authorities (and
hence, different rules, regulation and taxes), distance to locations frequently commuted,
regional or personal preferences, transport connectivity etc.
Simultaneously, the Competition Commission of India (CCI) received a complaint from the Tamil
Nadu State Government alleging that two companies, M/s Sun Limited, a company engaged in
the business of manufacturing solar panels, and M/s Shine Limited, a company engaged in the
sale, installation and maintenance of solar energy generation plants, have entered into an
informal agreement to limit or control the production, supply and marketing of the products to
ensure maximum price realisation. M/s Sun Limited sells its manufactured panels on an
exclusive basis to M/s Shine Limited, which is India's largest solar power generation company
supplying solar plants to more than 60% of the current market.

© The Institute of Chartered Accountants of India


30 FINAL (NEW) EXAMINATION: MAY 2019

Therefore, it is the case of the Tamil Nadu State Government that the agreement between M/s
Sun Limited and M/s Shine Limited is anti-competitive and has an adverse effect on competition
since the entities have abused their dominance in the market.
Answer the following questions:
3.1 Which of the following are not duties of the Competition Commission of India?
(A) To promote and sustain competition in markets in India.
(B) To protect the interests of consumers.
(C) To ensure freedom of trade carried on by Indian suppliers in global market.
(D) To eliminate practices having adverse effect on competition. (2 Marks)
3.2 Notwithstanding anything contained in sub-regulation (2), the Commission may, after
recording reasons, invalidate a notice filed under regulation 5 or regulation 8 of The
Competition Commission of India (Procedure in regard to the transaction of business
relating to combinations) Regulations, 2011 as amended when it comes to the knowledge
of the Commission that such notice is not valid as per sub-regulation (1) and, in that case,
the Secretary shall convey the decision of the Commission to the parties to the combination
within _____________.
(A) Seven days of such decision of the Commission.
(B) Fourteen days of such decision of the Commission.
(C) Seven working days of such decision of the Commission.
(D) Fourteen working days of such decision of the Commission. (2 Marks)
3.3 Operate independently of competitive forces prevailing in the relevant market is-------------
--- component.
(A) Abuse of Dominance
(B) Anti-Competition agreements
(C) Combinations Regulation
(D) Competition Advocacy (2 Marks)
3.4 Which of the following are not functions of Insolvency Professional Agencies (IPAs)?
(A) Monitoring, Inspecting and Investigating members.
(B) Recommending Insolvency Professionals to Committee of Creditors.
(C) Drafting detailed standards and code of conduct for insolvency professionals.

© The Institute of Chartered Accountants of India


PAPER – 6D: ECONOMIC LAWS 31

(D) Addressing grievances, hearing complaints and taking suitable action. (2 Marks)
3.5 Following are the liabilities of M/s A Limited, which is under insolvency process under IBC
2016.
(i) Loan from Bank - INR 100 crores.
(ii) Secured Debentures issued to M/s B Limited - INR 20 crores.
(iii) Trade Payable (10 creditors, including B Ltd., whose outstanding is ` 2 crores) - INR
14 crores.
(iv) Amounts payable to workmen - INR 4 crores. (2 Marks)
Calculate the voting share of M/s B Limited in the Committee of Creditors.
(A) 15.9420%
(B) 16.6667%
(C) 16.4179%
(D) 16.1290% (2 Marks)
3.6 The liquidation process relating to corporate debtors under IBC 2016 will not be initiated
under which of the following circumstances ?
(A) The Committee of Creditors do not approve a resolution plan within 180 days.
(B) The NCLT rejects the resolution plan submitted to it on technical grounds.
(C) The Committee of Creditors resolve to liquidate the debtor with a majority (> 50%).
(D) The debtor contravenes the agreed resolution plan and an affected person makes an
application to the NCLT to liquidate the debtor. (2 Marks)
3.7 The liquidator of M/s Wrongway has sought your help in prioritizing the claims against M/s
Wrongway, as per IBC 2016:
(1) Costs payable to liquidator and resolution professional.
(2) Property tax payable to Government of Goa.
(3) Salary payable to the Finance team for past 6 months.
(4) Amounts payable to M/s Dhara Bank towards secured loans, where security
was relinquished.
(5) Amounts payable to Holding company of M/s Wrong way for Royalty fees.
(A) (1), (2), (4), (3), (5)

© The Institute of Chartered Accountants of India


32 FINAL (NEW) EXAMINATION: MAY 2019

(B) (2), (1), (4), (5), (3)


(C) (1), (4), (2), (3), (5)
(D) None of the above (2 Marks)
3.8 Whether an operational creditor can assign or legally transfer any operational debt to a
financial creditor ?
(A) Yes. However, the transferee shall be considered as an operational creditor to such
extent of transfer.
(B) Yes but the transferee shall be considered as a financial creditor in relation to such
transfer.
(C) No. An operational creditor cannot assign or legally transfer any operational debt to
a financial creditor.
(D) No. An operational creditor can assign or legally transfer an operational debt only to
an operational creditor. (2 Marks)
3.9 Which of the following agenda items should be taken up in the first meeting of committee
of creditors (COC) ___________.
(A) Appointment of interim resolution professional as insolvency professional or
replacement of the interim resolution professional by another resolution professional
(B) Preparation of draft resolution plan.
(C) Discussion of the status of the corporate debtor as on the present date and the road
map ahead.
(D) Collection of information on corporate debtor from independent sources (2 Marks)
3.10 What is quorum in case of meeting of committee of creditors (CoC)?
(A) Members of the Committee representing at least 33% of the voting rights present
either in person or video conference or other audio visual means.
(B) Members of the Committee representing at least 50% of the voting rights are present
either in person or proxy.
(C) Members of the Committee representing at least 50% of the voting rights are present
either in person or video conference or other audio visual means or proxy.
(D) Members of the Committee representing at least 66% of the committee present in
person or proxy. (2 Marks)

© The Institute of Chartered Accountants of India


PAPER – 6D: ECONOMIC LAWS 33

3.11 What is to be construed as a "Dispute" under the Insolvency & Bankruptcy Code, 2016 ?
State its significance for the maintainability of an application filed under section 9 of the
Code.
In the given case study whether the appellant M/s Transmission Corporation of Andhra
Pradesh Limited will succeed in its appeal ? Decide. (11 Marks)
3.12 (i) Examine the provisions of the Competition Act, 2002:
(a) Decide whether the agreement between Sun Limited and Shine Limited is
covered under the scope of the Act with reasons. Also, clarify the nature of the
agreement based on facts provided. (4 Marks)
(b) What factors shall the CCI consider while evaluating the views of the
Government of Tamil Nadu? (6 Marks)
(c) What orders can the CCI pass on completion of the inquiry? (4 Marks)
(ii) In the given case study, decide with reasons whether Rajendra Singh (Informant) will
succeed against the Opposite Party (OP) for alleged violation of Section 4(2)(a) of
the Competition Act, 2002 ? (6 Marks)
Answer Case Study 3
3.1 (C)
3.2 (C) Note: Prior to Amendment in the combination Regulation, it was Option A.
3.3 (A)
3.4 (B)
3.5 (B)
3.6 (C)
3.7 (D)
3.8 (A)
3.9 (A)
3.10 (A)
3.11 Meaning of dispute: As per section 5(6) of the Insolvency and Bankruptcy Code, 2016 the
word “Dispute” includes a suit or arbitration proceedings relating to—
(a) the existence of the amount of debt;
(b) the quality of goods or service; or
(c) the breach of a representation or warranty;
Significance of “dispute” for filing of an application by operational creditor under
section 9 of the IBC: If there is any dispute about debt, the corporate debtor is required to

© The Institute of Chartered Accountants of India


34 FINAL (NEW) EXAMINATION: MAY 2019

reply within ten days of receipt of copy of invoice, existence of a dispute, or record of the
pendency of the suit or arbitration proceedings filed before the receipt of such notice or invoice
in relation to such dispute [section 8(2)(a) of Insolvency & Bankruptcy Code, 2016].
After the expiry of the period of ten days from the date of delivery of the notice or invoice
demanding payment, if the operational creditor does not receive payment from the
corporate debtor or notice of the dispute, the operational creditor may file an application
before the Adjudicating Authority for initiating a corporate insolvency resolution process.
The operational creditor shall, along with the application furnish the relevant documents,
containing an affidavit to the effect that there is no notice given by the c orporate debtor
relating to a dispute of the unpaid operational debt. The Adjudicating Authority shall, within
fourteen days of the receipt of the application, by an order admit the application and
communicate such decision to the operational creditor and the corporate debtor if no notice
of dispute has been received by the operational creditor or there is no record of dispute in
the information utility.
In the given case study, appellant M/s Transmission Corporation, filed an appeal against
the order of NCLAT before the Supreme court on the ground of existence of a dispute, so
the application under IBC cannot be accepted and so order passed in the favour of
respondent (M/s Equipment Conductors & Cables Ltd.) to be revered.
As per the facts given in the case study, respondent filed petition seeking execution of
amount in respect of Invoice Nos. 1-57. Vide order dated Nov, 8, 2016 high court held that
this revision petition holding that there was no award in respect of claim towards Invoice
Nos. 1- 57 and therefore, it was not permissible for the respondent to seek the execution.
Against this order, respondent approached NCLT on the ground that it has served demand
notice dated October 2014, 2017. This application was dismissed by the NCLT Vide Order
April 9, 2018. Against this order of NCLT, respondent filed appeal before NCLAT. NCLAT
challenged the orders and passed an order dated 4 th September 2018.
Cause of action arised when, high court rejected on the execution of the petition which was
holding that there was no award in respect of claim towards Invoice Nos. 1- 57, was
passed, which means that still the dispute is pending. Against this order, respondent,
served demand notice dated October 14, 2017.
As per the Code, if there is dispute about claim of debt between parties prior to issue of
demand notice by operational creditor, application cannot be admitted. On the basis of this
ground, Appellant challenged the subject matter of the order passed by NCLAT dated 4th
September 2018 before Supreme Court.
In the judicial pronouncement, it was held that, application by operational creditor to initiate
insolvency process was accepted when it was found that there was no existing dispute
prior to date of demand notice and dispute raised after receipt of demand notice was not
genuine [Badjate Stock v. Snowblue Trexim (2018) 145 SCL 441 = 89 taxmann.com 64
(NCLT)].

© The Institute of Chartered Accountants of India


PAPER – 6D: ECONOMIC LAWS 35

Also, If appeal has been filed under section 34 of Arbitration Act, the proceedings are
pending as appeal is continuation of the adjudication proceedings. Hence, application for
insolvency resolution is not maintainable. [CG Power & Industrial Solutions Ltd. v. ACC
Ltd. (2018) 91 taxmann.com 363 (NCLT)].
Therefore, in the light of the given facts and circumstances, Appellant M/s Transmission
Corporation of Andhra Pradesh Limited will succeed in its appeal.
3.12 (i) (a) Anti-Competitive Agreements [Section 3]:
It shall not be lawful for any enterprise or association of enterprises or person
or association of persons to 'enter' into an agreement in respect of production,
supply, storage, distribution, acquisition or control of goods or provision of
services, which causes or is likely to cause an appreciable adverse effect on
competition within India. All such agreements entered into in contravention of
the aforesaid prohibition shall be void.
Any agreement entered into between enterprises or associations of enterprises
or persons or associations of persons or between any person and enterprise or
practice carried on, or decision taken by, any association of enterprises or
association of persons, including cartels, engaged in identical or similar trade of
goods or provision of services, shall be presumed to have an appreciable
adverse effect on competition, which—
(a) directly or indirectly determines purchase or sale prices;
(b) limits or controls production, supply, markets, technical development,
investment or provision of services;
(c) shares the market or source of production or provision of services by way
of allocation of geographical area of market, or type of goods or services,
or number of customers in the market or any other similar way;
(d) directly or indirectly results in bid rigging or collusive bidding.
In the instant case, M/s Sun Limited and M/s Shine Limited have entered into an
informal agreement to limit or control the production, supply and marketing of
the products to ensure maximum price realization. M/s Sun Ltd. sells its
manufactured panels on an exclusive basis to M/s Shine Ltd. which is India’s
largest solar power generation company supplying solar plants to more than
60% of the current market.
The above agreement is covered under the scope of the Competition Act, 2002
as it is an Anti-competitive agreement under section 3.
(b) Dominant position of enterprise: The Commission shall, while inquiring
whether an enterprise enjoys a dominant position or not, have due regard to all
or any of the following factors, namely:—
(a) market share of the enterprise;

© The Institute of Chartered Accountants of India


36 FINAL (NEW) EXAMINATION: MAY 2019

(b) size and resources of the enterprise;


(c) size and importance of the competitors;
(d) economic power of the enterprise including commercial advantages over
competitors;
(e) vertical integration of the enterprises or sale or service network of such
enterprises;
(f) dependence of consumers on the enterprise;
(g) monopoly or dominant position whether acquired as a result of any statute
or by virtue of being a Government company or a public sector undertaking
or otherwise;
(h) entry barriers including barriers such as regulatory barriers, financial risk, high
capital cost of entry, marketing entry barriers, technical entry barriers,
economies of scale, high cost of substitutable goods or service for consumers;
(i) countervailing buying power;
(j) market structure and size of market;
(k) social obligations and social costs;
(l) relative advantage, by way of the contribution to the economic
development, by the enterprise enjoying a dominant position having or
likely to have an appreciable adverse effect on competition;
(m) any other factor which the Commission may consider relevant for the inquiry.
(c) Orders by Commission after inquiry into agreements or abuse of domina nt
position [Section 27]
Where after inquiry the Commission finds that any agreement referred to in
section 3 or action of an enterprise in a dominant position, is in contravention of
section 3 or section 4, as the case may be, it may pass all or any of the following
orders, namely:—
(a) direct any enterprise or association of enterprises or person or association
of persons, as the case may be, involved in such agreement, or abuse of
dominant position, to discontinue and not to re-enter such agreement or
discontinue such abuse of dominant position, as the case may be;
(b) impose such penalty, as it may deem fit which shall be not more than ten
per cent. of the average of the turnover for the last three preceding financial
years, upon each of such person or enterprises which are parties to such
agreements or abuse.
In case any agreement referred to in section 3 has been entered into by a
cartel, the Commission may impose upon each producer, seller, distributor,
trader or service provider included in that cartel, a penalty of up to three

© The Institute of Chartered Accountants of India


PAPER – 6D: ECONOMIC LAWS 37

times of its profit for each year of the continuance of such agreement or
ten per cent. of its turnover for each year of the continuance of such
agreement, whichever is higher.
(c) Omitted
(d) direct that the agreements shall stand modified to the extent and in the
manner as may be specified in the order by the Commission;
(e) direct the enterprises concerned to abide by such other orders as the
Commission may pass and comply with the directions, including payment
of costs, if any:
(f) Omitted
(g) pass such other order or issue such directions as it may deem fit.
While passing orders under this section, if the Commission comes to a finding,
that an enterprise in contravention to section 3 or section 4 of the Act is a
member of a group as defined in clause (b) of the Explanation to section 5 of
the Act, and other members of such a group are also responsible for, or have
contributed to, such a contravention, then it may pass orders, under this section,
against such members of the group.
(ii) Section 4(2)(a) of the Competition Act, 2002 says there shall be an abuse of
dominant position under sub-section (1), if an enterprise or a group,—
(a) directly or indirectly, imposes unfair or discriminatory—
(i) condition in purchase or sale of goods or services; or
(ii) price in purchase or sale (including predatory price) of goods or service; or
Explanation.—For the purposes of this clause, the unfair or discriminatory
condition in purchase or sale of goods or services referred to in sub-clause (i)
and unfair or discriminatory price in purchase or sale of goods (including
predatory price) or service referred to in sub-clause (ii) shall not include such
discriminatory conditions or prices which may be adopted to meet the
competition.
As Opposite Party (OP) has increased the price of the flat allotted to Rajendra
Singh from ` 2,00,000 to ` 7,00,000. OP has been indulged in unfair and
arbitrary practices and has misused its dominant position in the market.
Hence, OP has violated section 4(2)(a) of the Competition Act, 2002.

© The Institute of Chartered Accountants of India


ELECTIVE PAPER 6D: ECONOMIC LAWS

CASE STUDY 1

Mr. Bhanu Pratap Taneja is a leading real estate developer based in Delhi. In the
last decade, his company Garvit Bhoomi Developers Pvt. Limited having registered
office in Bhikaji Cama Place, New Delhi, had successfully developed four housing
projects – two in Gurgaon and one each in Jaipur and Lucknow. They had a robust
management team. Having been the name behind developing more than five
thousand luxurious apartments with modern amenities, they had the reputation of
delivering the projects well within the promised time.
In the beginning of the year 2015, they launched another project in Indirapuram, UP
by the name Omega Capetown Residency in which 1000 residential units consisting
of 2BHK and 3BHK apartments were to be developed. They were to be completed in
all respects by January 2018 and delivered to the consumers by that date. This
project was being carried on smoothly when the Real Estate (Regulation and
Development) Act, 2016 came to be enacted w.e.f. 1 st May, 2016. Section 3, which
was enacted later w.e.f. 1 st May 2017. Since Omega Capetown Residency consisted
of 1,000 residential units, it was required to be registered and so the company
submitted the requisite documents with concerned authorities.
As the application for registration was found to be complete in all respects, the
Omega Capetown Residency Project was granted registration by RERA (UP) within
the statutory period and was provided with a registration number including a log-in
ID for assessing the website of the Authority and to create webpage.
In the meantime, Mr. Taneja was approached by some of the influential developers
that an understanding had been reached among them to control the price of
apartments to be built by them. However, because of legal tangles such
understanding could not be brought into writing and it was also not intended to be
enforced by legal proceedings. Mr. Taneja did not agree to the proposal because
even though the understanding was not in writing and it was not intended to be
enforceable by legal proceedings, it was still illegal as per the Competition Act, 2002.
This revelation made by Mr. Taneja discouraged the intending developers and they
desisted from being a party to this proposal.
Mr. Taneja’s son Garvit, who was a commerce graduate and holder of law degree,
had a college friend Rohit whose father Mr. Dev Kumar dealt in sale, purchase and
renting of properties under the title ‘Dev Property Dealers’ from the Yusuf Sarai
market. Since Rohit had joined his father’s business, it was thought prudent to
convert the existing proprietary business into a registered partnership firm titled as
‘Dev & Sons Property Dealers’. Because of the enactment of Real Estate Act, Rohit
consulted his friend Garvit regarding its implications in case of real estate agents.
Accordingly, the firm was got registered as real estate agent with the help of Garvit’s
legal advisor.
Further, Garvit made a proposal to Rohit and his father that they could associate
themselves with his Omega Capetown Residency, a registered RERA project in
Indirapuram for facilitating sale of apartments which they readily accepted. Garvit
also cautioned them that as per the Act, since their firm was now a registered real
estate agent they were not supposed to facilitate sale/purchase of any plot,
© The Institute of Chartered Accountants of India
apartment or building in a real estate project being sold by the promoter in any
planning area, if such project was not registered with RERA of the concerned State.
In addition, Garvit’s legal advisor told them that as required by Section 10, a
registered real estate agent would maintain and preserve proper books of accounts
and other necessary documents. Further, such agent would not involve himself in
any unfair trade practice like making a false statement regarding services to be
provided by him. He would also not permit the publication of any advertisement
whether in any newspaper or otherwise of services that were not intended to be
offered. Besides, the agent would also have to help the intending buyers in getting
the required information and documents to which they were entitled, at the time of
booking of any property.
Rohit had a friend Tarun whose father Dr. Sreenivas Sharma was a surgeon in a
government hospital and was residing in a rented government flat in the hospital
campus itself. He had an intense desire to have a luxurious flat of his own. Tarun
had joined IBM after doing MBA from IIFT, New Delhi. So, with the combined salary
of both, they decided to buy a flat. Tarun contacted Rohit to help him in searching a
suitable apartment for his family. In turn, Rohit informed him that one particular 3BHK
flat at an ideal location was available in Omega Capetown Residency in Indirapuram
as the original allottee had withdrawn from the scheme; otherwise the booking under
this project was already full. Dr. Sharma got interested in the information and went
to the Omega Capetown Residency along with his family to see the concerned
apartment. He liked its strategic location and gathered more information regarding
sanctioned plan, layout plan along with the other specifications, etc. He then asked
for stage-wise time schedule of completion of the project and also enquired regarding
provision of water, sanitation and other amenities. Since, Rohit personally knew
Garvit and his father Mr. Taneja - the promoters of the project - Dr. Sharma and his
family had a lively and fruitful meeting with them. Subsequently, he and his son jointly
entered into an agreement for sale with the promoters of the project and made
payment of 75% of the cost of the apartment, while remaining 25% of the cost was
to be paid at the time when the apartments were ready for occupation.
A few months after booking the apartment, Dr. Sharma got a notice from the
promoters of Omega Capetown Residency that due to unforeseen circumstances
they were not in a position to complete the project and needed the allottees’ consent
for transferring of their majority rights and liabilities to another reputed developer
M/s. Sai Developers Pvt. Limited of New Delhi. In case any of the allottees was not
agreeable to this proposal he could get his money refunded. Since Dr. Sharma was
very much attached to the location of the flat, he accepted the proposal after
enquiring with Rohit and his father. He also learnt that 95% of the allottees had
already given their written permission. Further, the Authority had given its written
approval to the proposal for transfer and completion of Project by M/s. Sai
Developers Pvt. Limited. Dr. Sharma was also assured by Mr. Bhanu Pratap Taneja,
the erstwhile promoter with whom he had earlier interacted satisfactorily, that all the
pending obligations would be fulfilled by the new developer and in no case the date
of completion of the project would be extended; otherwise it would attract penalty. It
was also disclosed by Mr. Taneja that the new promoter would rectify any structural
defect if it occurred within a period of five years from the date of handing over the
possession of the apartments. Dr. Sharma, thus felt relieved.
© The Institute of Chartered Accountants of India
M/s. Sai Developers completed the project on time and received Completion
Certificate from the Competent Authority. As per the agreement for sale, Dr. Sharma
made payment of the remaining 25% of the cost. Thereafter, he received Occupancy
Certificate and took physical possession of the apartment well before two months
since the allottees were supposed to take physical possession within statutory period
of two months from the issue of Occupancy Certificate. He was also given other
necessary documents and plans, including that of common areas. He also became
a member of the RWA formed by the allottees. In the meantime, the promoter
executed a registered conveyance deed in favour of each of the allottees along with
the undivided proportionate title in the common areas to the RWA.
I. Multiple Choice Questions
1. Registration of a real estate project shall not be required –
(a) where the area of land proposed to be developed does not exceed
five hundred square meters or the number of apartments proposed to
be developed does not exceed eight.
(b) where the area of land proposed to be developed does not exceed
five thousand square meters or the number of apartments proposed
to be developed does not exceed eighty.
(c) where the area of land proposed to be developed does not exceed
two hundred fifty square meters or the number of apartments
proposed to be developed does not exceed four.
(d) where the area of land proposed to be developed does not exceed
three hundred square meters or the number of apartments proposed
to be developed does not exceed three.
2. Who is required to submit a copy of duly obtained approvals and
commencement certificate for getting the project registered with RERA:
(a) Allottee
(b) Promoter
(c) Real Estate Agent
(d) None of the above
3. A registered real estate agent shall -
(a) Facilitate the sale/purchase of any plot, apartment or building, being
sold by the promoter in any planning area, which is registered with
the Authority;
(b) maintain and preserve prescribed books of account, records and
documents;
(c) not involve himself in any unfair trade practices
(d) All of the above.

© The Institute of Chartered Accountants of India


4. The promoter is required to rectify any structural defect if it occurs within a
period of ----- years from the date of handing over the possession of the
apartments to allottees –
(a) Two years
(b) Three years
(c) Four years
(d) Five years
5. Registration of on-going project for which completion certificate is yet to be
received is mandatory -
(a) Yes, if the area of land (developed or to be developed) exceeds five
hundred square meters or the number of apartments (developed or
to be developed) exceeds eight.
(b) No, irrespective of the area of land or the number of apartments
(c) Can’t say
(d) None of the above
6. A real estate developer can leave the project mid-way by selling that project
to another developer if he has taken a written approval of ---------- of allottees
along with approval of the Authority.
(a) 2/3rd
(b) 1/3rd
(c) 3/4th
(d) 1/4 th
7. The time limit within which an allottee is required to take physical possession
of the apartment after issue of occupancy certificate is -
(a) one month
(b) two months
(c) three months
(d) four months
8. A certificate certifying that the real estate project has been developed
according to the sanctioned plan, layout plan and specifications as approved
by the competent authority under the local laws is called -
(a) Commencement Certificate
(b) Completion Certificate
(c) Occupancy Certificate
(d) None of the above

© The Institute of Chartered Accountants of India


9. The flat purchased by Dr. Sharma jointly with his wife Mrs. Neelima Sharma
though funded by him would be held as ‘benami transaction’ under the
Prohibition of Benami Property Transactions Act, 1988 -
(a) Yes
(b) No
(c) Can’t say
(d) None of the above
10 As per the Competition Act, 2002 ‘Agreement’ includes any arrangement or
understanding or action in concert:
(a) Whether or not, such arrangement, understanding or action is formal
or in writing; or
(b) Whether or not such arrangement, understanding or action is
intended to be enforceable by legal proceedings.
(c) Whether or not, such arrangement, understanding or action is formal
or in writing; or whether or not such arrangement, understanding or
action is intended to be enforceable by legal proceedings.
(d) None of the above.
II. Descriptive Questions
1. (i) Examine the following given aspects with reference to the allottee in
the situation given in the case study :
(a) Rights exercised by Dr. Sharma as an allottee.
(b) Duties fulfilled by Dr. Sharma as an allottee.
(c) Right which was not exercised by him and duty which was not
required to be fulfilled by Dr. Sharma.
(ii) The promoters of Omega Capetown Residency transferred majority
of rights and liabilities to Sai DeveIopers Pvt Ltd. for the completion
of the project. Advise as to the validity of such transfer of a real estate
project to a Sai Developer’s Pvt Ltd in the case study?
2. In the given case study Omega Capetown Residency has got itself registered
under the Real Estate Regulatory Authority, as it consisted of 1,000
residential units. However, if Omega Capetown Residency consisted of only
250 residential units, then was it necessary to get itself registered under the
Real Estate (Regulation and Development) Act, 2016: if yes, name the
various important documents and declarations which are required to be
submitted by a ‘real estate developer’ while registering a project with the
Real Estate Regulatory Authority (RERA) having only 250 residential units
and not 1,000 residential units.
3. Mr. Bhanu Pratap Taneja was approached by some of the influential
developers to join their association so as to reach an understanding whereby
they could inflate the price of the apartments built by them. Even though the
deal was in favour of Mr. Bhanu Pratap Taneja, he rejected the proposal from
other developers. In the light of the provisions of the Competition Act, 2002,
discuss whether the decision of Mr. Bhanu Pratap Taneja is lawful?
© The Institute of Chartered Accountants of India
ELECTIVE PAPER 6D – ECONOMIC LAWS
SUGGESTED SOLUTION – CASE STUDY 1

I. ANSWERS TO OBJECTIVE TYPE QUESTIONS

1. (a) [Hints: Refer Section 3 of the Real Estate (Regulation and Development) Act, 2016]

2. (b) [Hint: Refer Section 4 (2) (c) of the Real Estate (Regulation and Development) Act,
2016]

3. (d) [Hint: Refer Section 10 of the Real Estate (Regulation and Development) Act, 2016]

4. (d) [Hint: Section 14 (3) of the Real Estate (Regulation and Development) Act, 2016]

5. (a); [Hint: Refer Section 3 of the Real Estate (Regulation and Development) Act, 2016.
It may not be mandatory in a particular State if the State has granted exemption to
such on-going project]

6. (a); [Hint: Section 15 of the Real Estate (Regulation and Development) Act, 2016. It is to
be noted that if a consumer or his family holds more than one unit in the project then
he will be considered as one consumer only]

7. (b); [Hint: Refer Section 19 (10) of the Real Estate (Regulation and Development) Act,
2016]

8. (b); [Hint: Refer Section 2 (q) of the Real Estate (Regulation and Development) Act,
2016]

9. (b); [Hint: Refer Section 2 (9) (iii) of the Prohibition of Benami Property Transactions Act,
1988]

10. (c). [Hint: According to Section 2 (b) of the Competition Act, 2002, ‘Agreement’ includes
any arrangement or understanding or action in concert:

(i) Whether or not, such arrangement, understanding or action is formal or in


writing; or

(ii) Whether or not such arrangement, understanding or action is intended to be


enforceable by legal proceedings.]

© The Institute of Chartered Accountants of India


1
II. ANSWERS TO DESCRIPTIVE QUESTIONS

1. (i) (a) With reference to Section 19 of the Real Estate (Regulation and
Development) Act, 2016, Dr. Sharma, as an allottee, exercised the following
rights:

(I) Obtained the information relating to sanctioned plans, layout plans


along with the specifications as approved by the competent
authority.

(II) Demanded to know stage-wise time schedule of completion of the


project, including the provisions for water, sanitation, electricity and
other amenities.

(III) Claimed physical possession of the said apartment.

(IV) Obtained the necessary documents and plans, including that of


common areas, after getting the physical possession of the
apartment from the promoter.

(b) With reference to Section 19 of the Real Estate (Regulation and


Development) Act, 2016, Dr. Sharma, as an allottee, fulfilled the following
duties:

(i) Made necessary payments within the time as specified in the


agreement for sale.

(ii) Became a member of the RWA formed by the allottees.

(iii) Took physical possession of the apartment within a period of two


months from the issue of Occupancy Certificate.

(iv) Participated towards registration of the conveyance deed of the


apartment.

(c) (i) With reference to Section 19 of the Real Estate (Regulation and
Development) Act, 2016, Dr. Sharma, as an allottee, did not exercise
the following right:

The right to claim the refund of amount paid along with prescribed
rate of interest. It was so because the promoter was able to give

© The Institute of Chartered Accountants of India


2
possession of the apartment in accordance with the terms of
agreement for sale.

(ii) With reference to Section 19 of the Real Estate (Regulation and


Development) Act, 2016, Dr. Sharma, as an allottee, was not
required to fulfill the following duty:

The duty to pay interest at prescribed rate for delay in making any
payment. It was so because he had made the payments in
accordance with the terms of agreement for sale.

(ii) As per section 15 of the Real Estate (Regulation and Development) Act, 2016, a
promoter is permitted to transfer his majority rights and liabilities in respect of a real
estate project to a third party.

The provisions given below are to be adhered to by the promoter for transfer:

(a) Obtain prior written consent from two-third of allottees. Such consent will not
include the consent given by the promoter.

(b) Also obtain prior written approval of the Authority.

Note: It is to be ensured that such transfer shall not affect the allotment or sale of
the apartments, plots or buildings, as the case may be, in the real estate project
developed by the promoter.

(i) After obtaining the required consent of both allottees and the Authority, the
new promoter shall be required to independently comply with all the pending
obligations under the provisions of the Act or the rules and regulations made
thereunder.

(ii) The new promoter is also required to comply with the pending obligations as
per the agreement for sale entered into by the erstwhile promoter with the
allottees.

(iii) Further, the new promoter must note that any transfer so permitted shall not
result in extension of time to him to complete the real estate project.

Note: In case of default, he shall be liable to the consequences for delay, as per the
provisions of the Act or the rules and regulations made thereunder.

© The Institute of Chartered Accountants of India


3
Since in the given case study , 95% of the allottees had already given their written
permission. Further, the Authority had given its written approval to the proposal for
transfer and completion of Project by M/s. Sai Developers Pvt. Limited in compliance
with the requirements given in the said provisions. Such transfer of a real estate
project to a Sai Developer’s Pvt Ltd. is valid.

2. According to proviso to section 3 of the Real Estate (Regulation and Development) Act, 2016,
projects that are on going on the date of commencement of the Act, and for which the
completion certificate has not been issued, the promoter of the project are required to make
and application to the concerned Authority for the registration of the said project within a
period of 3 months from the date of commencement of the Act.

Further, the section provides that no registration of real estate project shall be required where
the area of land proposed to be developed does not exceed 500 square meters or the number
of the apartments proposed to be developed does not exceed 8 inclusive of all phases.

Hence, the Act requires registration of on-going projects where completion certificate was
yet to be obtained as well as new projects, if the area to be developed exceeded 500 sq.
mtrs. or apartments to be built under the project exceeded eight. Thus, registration of Omega
Capetown Residency was must with the Real Estate Regulatory Authority of UP (RERA, UP),
as consisted of 1,000 residential units.

Further, even if Omega Capetown Residency consisted of only 250 residential units (i.e.
more than 8 units), it will be compulsory to get itself registered under the Act. The process
of registering a project with the Real Estate Regulatory Authority (RERA) which consists of
1,000 units or 250 units is same which is given under section 4 of the Act.

With reference to Section 4, various important documents and declaration required to be


submitted while registering a project with RERA are as under:

• Details of the project such as name, address, type, names and photographs of the
promoters, etc.

• Details of the project which were already launched by the real estate developer in
the preceding 5 years and their present status.

• Approvals and commencement certificates obtained from the competent authority for
each phase of the project separately.

© The Institute of Chartered Accountants of India


4
• Sanctioned layout plan, the development plan for the project and details of basic
facilities being made available like drinking water, electricity etc.

• Proforma of allotment letter, agreement for sale and conveyance deed to be signed
with the consumers.

• Location of the project with clear demarcation of the land for the project.

• Number, type and carpet areas of units to be sold.

• The details of open areas if any like terraces, balconies etc.

• Details of associated engineers, contractors, architects and intermediaries in the


project.

• a declaration, duly supported by an affidavit, stating the following important matters:

o that the promoter has a legal title to the land and it is free from all encumbrances
along with legally valid documents;

o the time period required for completion of the project;

o that seventy per cent. of the amount realised from the allottees, from time to
time, shall be deposited in a separate escrow account and shall be used only
for the purpose of completion of project;

o that the promoter shall get his accounts audited within six months after the end
of every financial year by a chartered accountant in practice; and shall take all
the pending approvals on time from the competent authorities; etc.

3. According to section 2(b) of the Competition Act, 2002, ‘Agreement’ includes any
arrangement or understanding or action in concert:

(i) Whether or not, such arrangement, understanding or action is formal or in writing, or

(ii) Whether or not such arrangement, understanding or action is intended to be


enforceable by legal proceedings.

Further, section 2(c) of the Competition Act, 2002, "Cartel" includes an association of
producers, sellers, distributors, traders or service providers who, by agreement amongst
themselves, limit, control or attempt to control the production, distribution, sale or price of,
or, trade in goods or provision of services.

© The Institute of Chartered Accountants of India


5
An association for the welfare of the trade or formed for any other purpose not mentioned in
the aforesaid definition will not be a cartel Thus, it is only when an association, by agreement
amongst themselves, limits control or attempts to control the production, distribution, sale or
price of, or, trade in goods or provision of services, that it will be a cartel.

Hence, an agreement which prohibits an enterprise or person or their association for entering
into an agreement in respect of production, supply, distribution, storage, acquisition or
control of goods or services, which causes or is likely to cause an appreciable adverse affect
on competition. Such agreements entered in contravention of the above are void. These
agreements are presumed to have an appreciable adverse affect on competition.

Here, in the given situation, the agreement between Mr. Bhanu Pratap Taneja and other
builders would have fallen into the ambit of section 2(b) and 2(c) of the Competition Act,
2002 as the aim of the association was to increase the price of the apartments. Thus, such
an association would be void.

© The Institute of Chartered Accountants of India


6
ELECTIVE PAPER 6D: ECONOMIC LAWS

CASE STUDY - 2

Mr. Manohar Mehta, renowned builder of Mumbai, owns a reputed building construction company known
as “Sri Ram Building Construction and Real Estate”. Due to his vast business empire, he is known as the
“King of the Property World”.

On the personal front, Mr. Mehta in his family has an elderly mother, wife, and three children. His father,
Mr. Sri Ram Mehta, had recently expired after prolonged sickness. Mr. Manohar Mehta’s mother, Mrs.
Rama Devi, is a religious lady always dedicating her time in worship and holy works. Wife, Urvashi, is a
home maker and a socialite. She is mostly involved in all the social activities and runs her own NGO
named “AAWAZ”. Mr. Mehta regularly give financial contribution to his wife’s NGO. These minor donations
gave Mr. Mehta a Noble man tag in the social circle and better business prospects.

Sonia, eldest daughter of Mr. Mehta, is married and well settled with her husband in Australia. She has
recently started her own import export business in Australia with the help of her father. Mr. Mehta would
transfer the amount to his daughter and she regularize the amount in the books of accounts of her
business.

Recently, a project was started by Mr. Mehta in Marol area of Mumbai. The project was named as “Shubh
Appartment”. Under this project a 5 storey building comprising of 2 flats (2 BHK) at each floor were
constructed. The actual construction cost of each flat was Rs. 50 lakh. The flat was sold at Rs. 60 Lakh.
The advance booking charges of Rs. 6 lakh for each flat was collected from the buyers by cheque. Proper
receipt was issued to all the buyers for the advance payment. Out of the 10 flats, 4 flats were sold at an
increased price of Rs. 62 Lakh. Rs. 2 lakh each was taken in cash from the 4 buyers. Therefore, he earned
in total Rs. 8 Lakh for these 4 flats. This amount of Rs. 8 lakh was send to Sonia via an independent agent.
Sonia utilized this amount in her business and taken into record via some entry in her books of accounts.

Mr. Manohar Mehta has two sons, Rohit and Sorav. Sorav is the youngest son. He is pursuing his
graduation from one of the best universities of Chicago. For his education, Mr. Mehta remitted foreign
exchange of USD 2, 00,000 through authorized person. During course of his studies, Sorav was caught
with the seasonal influenza, so there he required an emergency medical treatment. Mr. Mehta transmitted
additional amount of USD 70,000 for treatment through authorized person who was well known to him for
hassle free transfer.

Rohit, the elder son, after successful completion of his M.B.A. Finance degree, is now actively supporting
his father in his real estate business. To give a start to his career, Mr. Mehta handed over the project

1
© The Institute of Chartered Accountants of India
“Royal Aashiana” to be constructed in Kharghar. The said project was proposed to be developed in 1000
sq. mts. Rohit was working on the project under guidance of Mr. Mehta. He marketed about the said
project and invited persons to purchase the flats in the Royal Ashiana. It was an ongoing project, Rohit
without registration of the project made an agreement to sell some of the flats.

As per Mr. Mehta’s regular morning routine, he one day read his favorite column “Property for Sale” in the
newspaper. He came across one advertisement regarding the sale of the residential plot in Panvel district
of Maharashtra. He discussed about the advertisement with his manager, Mr. Shyam Pareekh. He asked
his manager to visit the actual site of the mentioned property.

Mr. Pareekh called the land owner, Mr. R. Thakker, and took the appointment for the visit. He went to
Panvel to meet the owner and see the property. It was a 10,000 square feet plot near the city area. Mr.
Thakker quoted a price of Rs. 1crore for selling his property. After two rounds of meeting the final
negotiation with the land owner was done and deal was locked for Rs. 90 Lakh. On mutual consensus
between them, down payment of Rs. 20 lakh was made to Mr. Thakker in cash. Further, a payment of Rs.
70 Lakh was done by cheque and the property was registered in the name of his (Mr. Mehta) mother.
Being a sacred woman, she was not interested in all such types of transactions or arrangements made on
her name by Mr. Mehta.

After few months, Mr. Manohar Mehta from his sources came to know that an agricultural land is on sale by
a farmer, Mr. Bhima Singh. The farmer’s 5 acres of agricultural land was located in Thane district of
Mumbai. Mr. Manohar Mehta thought it would be a great deal to buy the agricultural land around the lush
green vicinity of the Thane district. He further thought that he can resale this property after converting it to
farm houses to the potential buyers.

After the detail discussion with his management regarding the purchase of land, Mr. Manohar Mehta went
to Thane to see the agricultural land. The land was just 500 meters away from the highway. After visiting
the land Mr. Mehta became keen to buy the property. They had a talk with the farmer, Mr. Bhima Singh.
The farmer being illiterate hardly knew about the legal sale/purchase of the land. Mr. Manohar Mehta and
Mr. Pareekh negotiated and finalized the deal in Rs. 80 Lakh.

Since Mr. Manohar Mehta required funds for purchasing the Thane property, he decided to sell his Panvel
plot which was in the name of his mother. He retransferred the Panvel property (Which was in the name of
his mother) to himself and then sold the Panvel plot for Rs. 1.10 Crore. He took partial amount by cheque
and rest by cash. This way he safeguarded himself from showing the capital gain on financial record. Mr.
Mehta received Rs. 80 Lakh in cheque and rest Rs. 30 Lakh in cash.

2
© The Institute of Chartered Accountants of India
Whereas, Mr. Mehta induced Bhima Singh, and paid him Rs. 50 Lakh through cheque and Rs. 30 lakh
through cash.

Mr. Manohar Mehta was still having Rs. 30 Lakh out of sale of 1.10 Crore panvel property, at his disposal.
Mr. Mehta decided to deposit Rs. 2,000 each to his wife, two sons and mother, saving accounts every
month. He would continue deposition of Rs. 2000 each per month for next couple of years.

During one of the corporate parties while having a discussion, Mr. Mehta’s friend advised him to invest the
remaining amount in the shell company outside India. Mr. Mehta liked the suggestion and decided to send
Rs. 10 Lakh to invest in the shell company in Singapore via Hawala. He learned about Mr. Varun Das who
runs a business of hawala under the veil of running a financial company. Mr. Mehta contacted Varun Das
who agreed to transfer the fund via Hawala on 1% commission basis. In this way Mr. Mehta managed to
circulate the amount in the shell company outside India.

Mr. Manohar Mehta also donated Rs. 50,000 in cash to his wife’s NGO ÄAWAZ”.

After few months, Mr. Mehta decided to buy a new car, worth Rs. 50 Lakh. He did the down payment of Rs.
5 Lakh via cheque. For the remaining Rs. 45 lakh he took 3 years auto loan, so that he can deposit the
monthly installment in the bank. Hence in this way the remaining Rs. 10 Lakh, which he gained from the
sale of the Panvel property, was utilized..

Due to frequent transactions of hefty amount and his conduct of other financial activities in a year, Mr.
Mehta bank accounts and his family members account of transactions were in the scrutiny of the Income
Tax Department.

On further investigation it was discovered that Mr. Mehta, Mr. Thakkar, Rohit and Sonia being guilty for
different offences punishable under the different Acts.

I. Multiple Choice Questions

1. Sonia plans to make investment in India. She was permitted to do so as per the FEM
regulation in -

a. Trading in transferable development rights

b. Real Estate business and construction of farm houses

c. Agricultural or plantation activities

d. Chit funds subscribed through banking channel and on non-patriation basis

3
© The Institute of Chartered Accountants of India
2. Sonia ordered exports of goods from India for her business. The amount (export value) of
good shall be released and repatriated to India within period -

a. 3 Months from date of export

b. 6 Months from date of export

c. 9 Months from date of export

d. 9 Months from date of invoice covering such export

3. Amount released for the real estate project from allottees in separate account can be
withdrawn by promoter after it is certified by-

a. Cost accountant and an Architect

b. Engineer, and a Chartered Accountant

c. an Architect and an Engineer

d. Engineer, an Architect, and a Chartered Accountant in practice

4. "Who according to the Provision of Prevention of Money Laundering Act is/ are held to be
liable in dealing of Panvel property-

a. Mr. Manohar Mehta


b. Mr. Thakkar
c. Both (a) and (b)
d. Mr. Shyam Pareekh
5. Who among the following is liable for an offence of money laundering as per the Part C of
the Schedule given in the Prevention of money laundering Act-

a. Mr. Mehta
b. Mr. Thakkar
c. Mr. Atul
d. Ms. Sonia
6. How much amount of penalty Mr. Mehta has to pay on illegal remittance of money
transferred to Sorav

a. USD 6,00,000
b. USD 1,80,000
c. USD 60,000

4
© The Institute of Chartered Accountants of India
d. USD 50,000
7. Suppose any project started by Mr. Mehta was not completed within preferred time due to
force majeure. Remedy available to Mr. Mehta in this situation

a. Registration may be interim cancelled by Authority


b. Registration need to be freshly applied
c. Registration granted may be extended for period not exceeding 1 year on
application
d. Registration may be extended for reasonable period on application.
8. Mr. Mehta sells a flat of Royal Aashiana to Mr. X for the amount Rs. 75 lakh. Mr. X made
the advance payment. The correct value of the advance payment is

a. Rs. 7.5 Lakhs


b. Rs. 8 Lakhs
c. Rs. 8.5 Lakhs
d. Rs. 9 Lakhs
9. In case of dispute between Mrs. Rama Devi and Mr. Mehta, can Mr. Mehta legally claim
her right over the Panvel Plot?

a. Yes, because he is the beneficial owner in the transaction


b. No, because the transaction is Null and void
c. Yes, because he paid consideration for the transaction
d. None of the above
10. Mr. Mehta files an application for initiation of voluntary liquidation proceeding of his Real
Estate construction company. Mr. X, a home buyer of a flat in one of the project of Mr.
Mehta claimed for the re-fund of paid amount or demanded for handover of possession of
flat. Which amongst following is not incorrect statement

a. X cannot claim the amount due to pending of Insolvency process


b. X can file a suit for the default committed by Mr. Mehta under the Insolvency &
Bankruptcy code
c. X’s right & interest is protected after execution of an agreement to sale till the
conveyance of the flat
d. None of the above

5
© The Institute of Chartered Accountants of India
II. Descriptive Question

1. What would be the consequences in the following given situations:

(a) Where if Mr. Mehta remitted Foreign Exchange USD 2,00,000 and USD 70,000 as
educational and medical expenses to Sorav .

(b) Sorav used USD 20,000 out of the remitted medical expenses (i.e., USD 70,000)
and used remaining amount to purchase immovable property jointly with Mr.
Mehta in Chicago.

2. Suppose Royal ashiana, is a 15 year old building of Mr . X. It was purchased by Mr .


Mehta in January 2016. He planned to re-develop the said building into residential
apartments. He launched the project in end of January 2016. During the course, the
Government enacted the Real estate(Regulation and Development) Act, 2016.Mr. Mehta
seek advise of his legal consultant on the issues related to the registration of the said
Project-

(1) Is the Registration for the re-development of society is mandatory?

(2) If he plans to develop a new society under new name with new allotments . Is the
registration mandatory of the project?

(3) State the position where project have been completed and obtained the certificate
of completion before the commencement of RERA?

(4) Where if the project is to be devolved into phases ?

(5) Where if the authority has not taken any decision on application for the registration
within the prescribed period.

3. (i) What remedy is available to Mr. Mehta, in case he want to compound for the
commission of illegal remittance to Sorav under FEMA Act 1999?

(ii) According to the case study the property bought by Mr. Mehta in the name of his
mother, Mrs. Rama Devi, is a Benami Transaction and will be confiscated.
Suppose the property rightfully belonged to Mrs. Rama Devi then what measures
she can take to save the property from confiscation. Explain?

6
© The Institute of Chartered Accountants of India
ELECTIVE PAPER 6D – ECONOMIC LAWS
SUGGESTED SOLUTION – CASE STUDY 2

I. ANSWERS TO OBJECTIVE TYPE QUESTIONS

1. (d) [Hint: As per regulation 4 (b) explanation (ii) of the FEMA (permissible capital
account transaction) Regulation 2000.]

2. (c) [As per regulation of FEMA (Export of goods and services) regulation 2016]

3. (d) [Hint: Section 4 (2), proviso to (D) of clause (l) of the RERA, 2016]

4. (c) [Hint: Refer section 3]

5. (d) [Hint: Refer Part C of the Schedule to the Prevention of Money Laundering Act, 2002]

6. (c) [Hint: Refer Section 13 of Foreign Exchange Management Act 1999 along with
Schedule III of the FEM (Permissible Current Account Transactions) Regulations,
2000]

7. (c) [Hint: Refer Section 6 of RERA, 2016]

8. (a) [Hint: As per Section 13 of RERA, 2016]

9. (b) [Hint: Section 6 Prohibition of Benaim Property Transactions Act, 1988 ]

10. (c) [Hint: Section 11 (4) read with section 89 of the RERA, 2016]

II. ANSWERS TO DESCRIPTIVE QUESTIONS

1 (a) As per section 13 (1) of the FEMA, 1999, If any person contravenes any provision of
this Act, or contravenes any rule, regulation, notification, direction or order issued in
exercise of the powers under this Act, or contravenes any condition subject to which
an authorisation is issued by the Reserve Bank, he shall, upon adjudication, be liable
to a penalty up to thrice the sum involved in such contravention where such amount
is quantifiable, or up to two lakh rupees where the amount is not quantifiable.

Any Adjudicating Authority adjudging any contravention to above provisions, may, if


he thinks fit in addition to any penalty which he may impose for such contravention
direct that any currency, security or any other money or property in respect of which
the contravention has taken place shall be confiscated to the Central Government

© The Institute of Chartered Accountants of India


1
and further direct that the foreign exchange holdings, if any of the persons
committing the contraventions or any part thereof, shall be brought back into India
or shall be retained outside India in accordance with the directions made in this
behalf.

According to the above provisions, Mr. Mehta will be penalized thrice of the extra
amount (USD, 20,000) remitted above the prescribed limit (USD 2, 50,000). Hence
liable to pay a penalty of USD 60,000 to the Government.

(b) The second issue is related to sections 13(1A), 13(1C) & 37A of the FEMA Act, 1999
read with Regulation 5 of the FEM(Acquisition & transfer of immovable property
outside India)Regulation , 2015.

As per section 13(1A), if any person is found to have acquired any foreign exchange,
foreign security or immovable property, situated outside India, of the aggregate value
exceeding the threshold prescribed under the proviso to sub-section (1) of section
37A, he shall be liable to a penalty up to three times the sum involved in such
contravention and confiscation of the value equivalent, situated in India, of the
foreign exchange, foreign security or immovable property.

13(1C) of FEMA says that if any person is found to have acquired any foreign
exchange, foreign security or immovable property, situated outside India, of the
aggregate value exceeding the threshold prescribed under the proviso to sub-section
(1) of section 37A, he shall be, in addition to the penalty imposed under sub-section
(1A), punishable with imprisonment for a term which may extend to five years and
with fine.

According to Section 37A of the FEMA, upon receipt of any information or otherwise,
if the Authorised Officer prescribed by the Central Government has reason to believe
that any foreign exchange, foreign security, or any immovable property, situated
outside India, is suspected to have been held in contravention of section 4, he may
after recording the reasons in writing, by an order, seize value equivalent, situated
within India, of such foreign exchange, foreign security or immovable property:

Provided that no such seizure shall be made in case where the aggregate value of
such foreign exchange, foreign security or any immovable property, situated outside
India, is less than the value as may be prescribed.

© The Institute of Chartered Accountants of India


2
As per regulation 5 of the FEM (Acquisition & transfer of immovable property outside
India) Regulation, 2015, a person resident in India may acquire immovable property
outside India jointly with a relative who is a person outside India. Provided there is
no outflow of funds from India.

Since in the given case, Mr. Mehta remitted Foreign exchange to Sorav in excess to
the limit prescribed under the FEMA. Sorav partially used USD 20,000 for medical
treatment and rest USD 50,000 to purchase property outside India jointly with Mr.
Mehta. So Both Mr. Mehta and his son Sorav will be liable under sections 13(1),
13(1A), 13(1C) of the FEMA, 1999.

2. (1) According to section 3(2) of the Real Estate (Regulating ) Authority Act, 2016, no
registration of the real estate project shall be required for the purpose of renovation
or repair or re-development which does not involve marketing, advertising selling or
new allotment of any apartment, plot or building, as the case may be, under the real
estate project. So Registration for the re-development of society (Royal Ashiana)
was not required.

(2) According to the above provision no registration is required when any project is
renovated or repair or re-development and it does not involve marketing, advertising
selling or new allotment of any apartment, plot or building, as the case may be, under
the real estate project. However, in the given situation in the question, Mr. Mehta
plans to develop it as a new society under new name with new allotments. So
registration of the said project was necessaited as the Act.

(3) As per the proviso to section 3(1) of the RERA, projects that are ongoing on the date
of commencement of this Act and for which the completion certificate has not been
issued, the promoter shall make an application to the Authority for registration of the
said project within a period of three months from the date of commencement of this
Act. In the given case, where the project have been completed and obtained the
certificate of completion before the commencement of RERA, such project shall not
require registration.

(4) As per the explanation to section 3 of the RERA, where the real estate project is to
be developed in phases, every such phase shall be considered a stand alone real
estate project, and the promoter shall obtain registration under this Act for each

© The Institute of Chartered Accountants of India


3
phase separately. Therefore if the said project is to be developed in phases , it needs
separate registration for each phase.

(5) As per Section 5 of the RERA, the Authority has to decide on the application within
30 days of its receipt. It further provides that in case the Authority fails to take a
decision within the said period of 30 days the project shall be deemed to be
registered.

3. (i) According to provision of Section 15 (1)

(1) Any contravention under section 13 may, on an application made by the


person committing such contravention, be compounded within one hundred
and eighty days from the date of receipt of application by the Director of
Enforcement or such other officers of the Directorate of Enforcement and
officers of the Reserve Bank as may be authorised in this behalf by the
Central Government in such manner as may be prescribed."

(2) Where a contravention has been compounded under sub-section (l), no


proceeding or further proceeding, as the case may be, shall be initiated or
continued, as the case may be, against the person committing such
contravention under that section, in respect of the contravention so
compounded.

As per the above mention provision Mr. Mehta will submit the application to the
concerned authority for compounding of the offences committed in contravention to
the FEMA Act.

(ii) The Section 5 of PMLA authorizes the Director or any other officer not below the
rank of Deputy Director to attach the property. Section 8 of PMLA lays down an
elaborate procedure for adjudication of complaint under Section 5 of PMLA. It calls
for a show cause notice to be issued to the offender/ person from whom property
has been seized, so as to give the person an opportunity to make a case against
attachment. Such a person in order to avoid confiscation, can demonstrate the
legitimate source of his income/earning or assets, out of which or by means of which
he has acquired the property attached. The evidence on which he realized and other
relevant information and particulars, and to basically convince the authority about
the property which should not be declared to be the property involved in money
laundering.

© The Institute of Chartered Accountants of India


4
If the authority reached the conclusion that the offense has not taken place it shall
order release of such property to the person entitled to receive it.

Hence as per the above provision, Mrs. Rama Devi aggrieved by the provisional
attachment may file her objection before the adjudicating authority.

© The Institute of Chartered Accountants of India


5
FINAL COURSE
ELECTIVE PAPER 6D: ECONOMIC LAWS

CASE STUDY - 3

Mr. Krishnakant Mathur was a lecturer in agricultural college of Pushkar. After 25 years of his service he retired
from his job last month. To utilise his time, Mr. Mathur keeps the accounts of his society and has also started a
coaching classes for the needy and poor students. Mr. Mathur has three children, two daughters Jaya & Lata and
son Neeraj. His wife Asha Devi is a house wife. Neeraj is eldest of all the three children. Jaya and Lata are still
studying in Class 12th and 10th respectively. At the time of retirement of Mr. Mathur, Neeraj has completed his
engineering degree.
During his work tenure, Mr. Mathur purchased one property (area 2000 Sq feet), at Mansarover colony, Jaipur in
the name of his son Neeraj Mathur. The property already has a constructed house over it. Mr. Mathur also owns
an ancestral property in his village Titari. Since he was posted at Jaipur, so he resided at Mansarover colony house
with his family.
Neeraj got a good job in Maharashtra State Electricity Board (MSEB) as a Junior Engineer. He was posted at Pune.
He was very desperate to earn more money and become rich. For clearing the contractors billing and giving
necessary approvals he started giving favours to the contractors and vendors. Various projects were in the hands
of MSEB, they authorised Neeraj to finalise the tenders related to supply of required articles, goods and services
for attainment of the government projects. Many contractors pleased him by cash or kind for acceptance of their
tenders for the projects. Neeraj used his position in the said department by manipulating the bidding process.
Soon, Neeraj got promotion. He decided to celebrate his grand success and planned for a trip to Hongkong with
his friend. There in Hongkong, he came to know about XBL Company, which was a joint ventures abroad of Indian
Companies. He thought to make an equity investment in XBL Company. He talked with the Indian Service Providers
for the investment of funds to Securities Exchange of Hongkong for buying equity in XBL Company. Neeraj applied
for drawl of foreign exchange, but due to legal compliances, authorised dealer denied for the said transaction.
Neeraj, through known authorise dealer, on payment of commission exported the foreign exchange for the equity
investment in XBL Company. So in this way, Neeraj made an overseas investments.
In the meantime, Neeraj, while residing in Pune, met a girl in his office named Shalini. They both fell in love with
each other. After couple of months they both decided to marry without informing any of the family members as they
both are of different caste. As a witness Rahul, Neeraj’s friend, was the only one who knew about his marriage.
Rahul is located in Dubai and works as a Senior Manager with Al-Aadil Works LLC.
Rahul insisted him to come to Dubai. So Neeraj and Shalini planned their visit to Dubai. Shalini discussed about
her trip with her friend. Her friend told that Dubai is a beautiful place and is a hub for Gold and electronics. Since
Shalini had a keen interest in buying gold and Neeraj had a restriction in carrying foreign currency, he came up
with an idea. He gave Rahul’s family Rs. 1 Lakh in cash, in India and took the equivalent foreign currency from
Rahul in Dubai. They both had a good time in Dubai. They visited the Gold Souq in Deira. Shalini purchased gold
jewellery worth Rs. 1.5 Lakh. Rahul also took them around to see some latest Electronic items. Neeraj found that
the price of the Smart Television was less and the shopkeeper proposed further discount without a bill. So Neeraj
also bought a new Television worth Rs. 60000. After their return, they did not declare it to the Indian customs and
passed through the Green channel. Custom officials on the matter of doubt withheld them at the airport and
interrogated. After compliance with the required formalities under the legal prospects, the matter was sorted.
In 2016, during the period of demonetisation, Neeraj was holding a cash of Rs. 10 Lakh. He deposited Rs. 3 lakh
is his and his wife’s account within the permissible limit imposed by the government. To settle the rest of the amount
he took the help of his friend Jai Bakshi. Jai is a renowned hotelier based in Mumbai. As per the discussion Jai
told that he could manage Rs. 2 Lakh by depositing it in his account. Further Jai told that he will charge 10 percent

1
© The Institute of Chartered Accountants of India
of Rs. 2 Lakh to accommodate the amount in his record books. Also, he will return this amount only after six months
due to scrutiny of income tax department. Neeraj found that the market rate of changing the demonetised 500 and
1000 rupees notes were 50% of the given amount. So he thought it is better to agree to the terms and conditions
of Jai and gave him Rs. 2 Lakh. With the remaining Rs. 5 Lakh, he booked a flat in Shubh Laxmi apartment near
Badlapur Mumbai in the name of his wife Shalini. Neeraj on record showed above that paid amount of Rs. 5 lakh
as a loan taken from Shalini’s uncle.
Whereas, Shubh Laxmi Apartment, constructed by J.K. Builders was registered with the Maharashtra Real Estate
Regulatory Authority. The flat was sold by Himmat Chand to Neeraj. In the meantime J.K. Builders filed an
application for bankruptcy and shed of their responsibilities as to the completion and handover of the possession
of the flats to the buyers. Neeraj approached to the office of J.K. Builders and they denied from their responsibilities
by saying that the said flat was sold by Himmat Chand, so he owns the responsibility. Whereas Himmat chand
took the plea that ultimate responsibility lies with the J.K. Builders, being a promotor. Neeraj went to the consumer
forum for the relief.
In the meantime, due to the tip off received from unknown sources and on grounds of noticing the suspicious
activities of Neeraj, Income Tax Department issued a scrutiny notice to him. According to the Notice, Neeraj was
asked to clarify the mode of payment for the loan amount from his uncle as there was no entry of cash credited to
Shalini’s account. Neeraj and Shalini, however, managed and came safely out of a situation on benefit of doubt.
After few months, Neeraj went to Jaipur with his wife Shalini to meet his family. Mr. Mathur was extremely angry
and shattered to see his son married without his consent. Mr. Mathur and his son had a heated argument and he
turned both of them out of his house. Neeraj told that this house is in his name and legally belongs to him. Hence
his father has no right to throw them away.
Mr. Mathur files a case for claiming ownership over the property as he does not want to give the possession of his
house to Neeraj. As per plaint allegation, Mr. Mathur has purchased the property in 18/2/2015 in the name of his
son Neeraj Mathur. As per plaint allegation, he has purchased the property out of his own income. He has claimed
relief that he be declared as owner of the property and Neeraj should be permanently restrained from interfering
in possession of the property. Plaint clearly reveals that Mr. Mathur has purchased the property out of his own
income in the name of his son Neeraj. It was his own property and he had claimed declaration of ownership right
over the property. He has also prayed for permanent injunction against his son. Entire plaint allegation does not
whisper that it was joint Hindu family property or purchased by Mr. Mathur for joint Hindu family.
On the other hand, Neeraj claimed that he has purchased the property from his own income, his father was not
having any right over the property.
During pendency of the suit, Mr. Mathur died. According to the procedure of the Tribunal in case of death of one
of several plaintiffs or of sole plaintiff, the right to sue survives. The Tribunal, on an application made in that behalf
shall cause the legal representative of the deceased plaintiff to be made a party and shall proceed with the suit.
So, his wife and two daughters were impleaded as legal representatives of the deceased. His legal representatives
have amended the plaint and have claimed declaration that the property is Joint Hindu family property. By detail
amendment, made in the plaint, his legal representatives pleaded that Mr. Mathur was “Karta” of Joint Hindu
Family.
The Tribunal held that in the present case, as per the claim of Mr. Mathur he purchased the property in the name
of defendant i.e. the defendant was Benamidar. So, Mr. Mathur was not entitled to claim any right over such
property in the light of Section 4 of the Prohibition of Benami Property Transactions Act, 1988.
Further, the Tribunal held that Late Mr. Mathur or legal representatives of the plaintiff have neither pleaded nor
proved that the defendant was holding the property as a trustee or in a fiduciary capacity for the benefit of other
persons for whom he was a trustee or was standing in a fiduciary capacity, inter alia, as per original pleading and
amended pleading, the property was owned by Mr. Mathur as his self-acquired property or property was owned by
Joint Hindu Family. Nothing has been pleaded by original plaintiff or his legal representatives that Neeraj was a

2
© The Institute of Chartered Accountants of India
trustee or standing in a fiduciary capacity for the present legal representatives, original plaintiff and for others. Inter
alia, it has been specifically pleaded that the owner was the plaintiff and he has purchased the property in the
name of the present defendant.
Objective Type Questions:
1. In the given case study, Neeraj and Shalini bought gold jewellery worth Rs. 1.5 Lakh from Dubai. They have
custom clearance through green channel. State whether the given act will constitute an offence under the
Prevention of Money Laundering Act, 2002?
a. Yes, because the gold bought is beyond the permissible limit
b. Yes, because of an evasion of duty chargeable thereon goods on an & above the permissible limit.
c. No, because they are carrying the original bill of the purchased gold
d. No, because gold bought is within the permissible limit
2. Will Shalini’s uncle be liable for punishment under the Money Laundering Act, if he lends the loan amount
from his known sources of income?
a. Yes, because he is knowingly associating in the crime
b. No, because he is not a party to the crime
c. Yes, because he is actually involved in the process
d. Not sure
3. During the period of demonetisation Neeraj deposited Rs. 2 Lakh in Jai’s account. who is the beneficial person
in the light of the Prevention of Money-Laundering Act, 2002?
a. Neeraj
b. Jai
c. Both a and b
d. Neither a nor b
4. Suppose Mr. Mathur acquired a property from undisclosed & unaccounted sources of funds. Later, he created
a trust of his entire property for his family benefit and appointed Neeraj as his trustee. Will Neeraj be held
liable for such transaction made by Mr. Mathur-
a. Yes
b. No
c. Partially Liable
d. Not Sure
5. Neeraj in the course of his duty took commission in clearing the bill. This act can be termed as?
a. Money laundering through Prevention of Corruption Act, 1988
b. Money laundering through unlawful activities (Prevention) Act, 1967
c. Money laundering through customs
d. Money laundering through Indian Penal Code, 1860
6. Can Neeraj resale his house located in Jaipur to his father?
a. No, it is prohibited under Benami Transaction

3
© The Institute of Chartered Accountants of India
b. Yes, Because Neeraj has the right to sale the property
c. Yes, because Neeraj is an owner
d. Both b and c
7. State which statement as to the drawl of the foreign exchange by Neeraj for transaction related to equity
investment in XBL Company is correct –
a. Neeraj can do such transaction through authorised dealer by providing commission on export for equity
investment in XBL Company.
b. Neeraj can do such transaction by directly buying equity in XBL Company
c. Neeraj cannot transact for equity investment in XBL Company
d. Drawl of foreign exchange by Neeraj for payment of commission on exports towards equity investment
in XBL Company, is prohibited.
8. Neeraj can claim relief for the completion and handover of the possession of the flat purchased in Shubh
Lakshmi Apartment against-
a. J.K. Builders
b. Himmat Chand
c. Both (a) & (b)
d. Maharashtra Real Estate Authority
9. When any transaction cannot said to be benami transaction-
a. If a person deposits old currency in account of another person in an understanding that account
holder will return the money in new currency
b. Individual held property in joint name with his grand children with the consideration paid by an
individual.
c. Company raising share capital through fictitious shareholders
d. Person takes a loan and not able to prove the genuinity of the lender
10. Suppose there are 3 bidders X, Y, & Z in a tender process initiated for MSEB. X & Y bidders were removed
by neeraj unethically showing them incompetent to make bid in the tender process for supply of goods to
MSEB for its project. This relates to-
a. Regulation of combinations
b. Abuse of dominant position
c. Controlling affairs or management
d. All of the above
Descriptive Questions:
1. a. Neeraj without consulting his Chief Engineer Authority, affixes his sign and seal to a document
certifying Mr. X (Fictitious Client) as a registered dealer of electronic goods. He there by obtains
his share for certifying the said document.
Mr. X misappropriates this forged document for his business to obtain various projects of
government for supply of electric goods.
Examine the given situation as to the nature of offence committed under the Prevention of Money
Laundering Act, 2002.

4
© The Institute of Chartered Accountants of India
b. Neeraj While returning to India from Dubai trip gave wrong declaration about his gold and electronic
purchase at the airport. State the nature of liability of Neeraj for the commission of the above act?
2. a. Neeraj was given an offer by a company vendor to disclose him the lowest bid quoted by other
vendors. Neeraj accessed the computer of his Executive Director and passed on the lowest
quotation to the vendor and thus helped him in quoting the lowest among all the bids . Examine and
analyse the situation and conclude how Neeraj will be held liable under PMLA?
b. If suppose Neeraj with one of his friend registers a company and quotes bid to get a tender of
MSEB to get an extra income. Neeraj being at the back foot helps his friend unofficially to get
maximum tenders allotted to their company. They need to raise a share capital of the company. So
Neeraj decided to invest from his unknown sources of his income in the name of fictitious
shareholders.
Determine in the given scenario, the liability of Neeraj being covered under which Act?
3. (i) The Competition Commission of India (CCI) has received a complaint from a State Government
alleging that Shubh Limited and Mangal Limited have entered into an informal agreement, not
enforceable at law, to limit or control production, supply and market, to determine the sale price of
their products. Such an action of these companies has an appreciable effect on competition.
Examining the provisions of the Competition Act, 2002:
(A) Decide whether the above agreement has appreciable effect on competition.
(B) What factors shall the Competition Commission of India consider while taking the above
decision?
(C) What orders can the Competition Commission of India pass on completion of the inquiry?
(ii) Explain the restrictions, if any, under Foreign Exchange Management Act, 1999 in respect of the
following issue and transfer of shares:
Issue of Equity Shares of ` 1 crore at face value accounting for 45 percent of post-issue capital to
non-resident Indians in U.S.A. on non-repatriation basis. The shares are issued by M/s ABC
Knitwear Limited to finance the modernization of its plant.

5
© The Institute of Chartered Accountants of India
ELECTIVE PAPER 6D- ECONOMIC LAWS

SOLUTION – CASE STUDY 3

I. ANSWERS TO OBJECTIVE TYPE QUESTIONS

1. (b)

[Hint: As per Schedule I of PMLA, 2002 it is a predicate offence related to custom Act]

2. (b)

[Hint: Section 3 of the Prevention of Money-Laundering Act, 2002 states of commission of an


offence of money-laundering.—Whosoever directly or indirectly attempts to indulge or knowingly
assists or knowingly is a party or is actually involved in any process or activity connected with
the proceeds of crime and projecting it as untainted property shall be guilty of offence of money-
laundering. Since Shalini’s uncle has given the loan from the known sources of his income so
he is not a part of this crime.]

3. (b)

[ Hint: 2(fa) of PMLA, 2002 ]

4. (b)

[ Hint: As per exception of Section 2 (9) (A) Neeraj is not liable for Benami Transaction as he
stand in a fiduciary capacity for the benefit of other person.]

5. (a)

[Hint: Refer the schedule of PMLA, 2002]

6. (a)

[Hint: Refer Section 6 of the Prohibition of Benami Property Transactions Act, 1988]

7. (d)

[Hint: Refer Schedule I of FEM (Current Account Transaction ) Rules, 2000]

8. (c)

[Hint: Refer section 2(v) of the RERA]

9. (b)

[Hint: Refer section 2(9) of the Prohibition of Benami Property Transactions Act, 1988]

© The Institute of Chartered Accountants of India


10. (b)

[Hint: Refer section 4 of the Competition Act, 2002]

II. ANSWERS TO DESCRIPTIVE QUESTIONS

1. a. According to provision of the PMLA, 2002 the money earned by Neeraj is not from the
legitimate sources. Since Neeraj forged the sign and seal of his Chief Engineer so the
money earned by him is proceed of crime. According to Schedule Part A of Para 1 of
PMLA, Neeraj has committed an offence under section 472 and 473 of Indian Penal
Code. These Section deals with the offence of making or possessing counterfeit seal,
etc., with intent to commit forgery. Whoever makes or counterfeits any seal, plate or
other instrument for making an impression, intending that the same shall be used for
the purpose of committing any forgery which would be punishable under section 467 of
this Code and under any other section under this Code, or, with such intent, has in his
possession any such seal, plate or other instrument, knowing the same to be counterfeit,
shall be punished with imprisonment for life, or with imprisonment of either description
for a term which may extend to seven years, and shall also be liable to fine.

b. Neeraj is liable under the provision of PMLA Act. His act is covered under the provision of
Part B of the Schedule. Part B of the Schedule refers to offence under the Customs Act,
1962. Section 132 of the Customs Act states that whosoever makes sign, or use or cause
to be made, sign or use any declaration, statement or document in relation to customs
knowingly or having reasons to believe that such declaration statement etc. is false shall be
punishable for a term which may extend to two years or fine or both.

2. a. Neeraj has contravened the Prevention of Money Laundering Act under Part A Para 22
of the Information Technology Act 2000. According to the provision of Section 72 of
Information of Technology Act 2000, if any person who, in pursuance of any of the
powers conferred under this Act, rules or regulations made thereunder, has secured
access to any electronic record, book, register, correspondence, information, document
or other material without the consent of the person concerned discloses such electronic
record, book, register, correspondence, information, document or other material to any
other person shall be punished with imprisonment for a term which may extend to two
years, or with fine which may extend to one lakh rupees, or with both.

Neeraj in the given case, without the consent of his Executive Director accessed the
electronic records and passed on the official information to the vendor without permission.
This information can produce large profits and legitimize the ill-gotten gains through money

© The Institute of Chartered Accountants of India


laundering. Hence it is punishable under the Section 72 of the Information of Technology
Act, 2000.

b. The given issues falls within the ambit of the Prohibition of Benami Property Transactions
Act, 1988. According to Section 2 (26) "Property" means assets of any kind, whether
movable or immovable, tangible or intangible, corporeal or incorporeal and includes any
right or interest or legal documents or instruments evidencing title to or interest in the
property and where the property is capable of conversion into some other form, then the
property in the converted form and also includes the proceeds from the property.

According to Section 2 (10) "benamidar" means a person or a fictitious person, as the case
may be, in whose name the benami property is transferred or held and includes a person
who lends his name.

According to Section 2 (9) (B), a transaction or an arrangement in respect of a property


carried out or made in a fictitious name; or (D) a transaction or an arrangement in respect
of a property where the person providing the consideration is not traceable or is fictitious.
Hence, according to all the above provisions, Neeraj has done Benami transactions. He has
done the investment in the share as a Benamidar as he holds the share in the fictitious
name of shareholders. Hence this is a Benami transaction and is liable for punishment
under the Benami Transaction Act.

3. (i) (A) The term ‘agreement’ as defined in section 2 (b) of the Competition Act, 2002,
includes any arrangement or understanding or action in concert.

(i) whether or not such arrangement, understanding or action is formal or in


writing, or

(ii) whether or not such arrangement, understanding or action is intended to


be enforceable by legal proceedings.

Thus an agreement between Shubh Ltd. and Mangal Ltd. satisfies the above
ingredients of an agreement as per section 2 (e) of the Act, so agreement has
appreciable effect on competition.

(B) Factors to be considered:

(1) creation of barriers to new entrants in the market.

(2) driving existing competitors out of the market.

(3) foreclosure of competition by hindering entry into the market.

(4) accrual of benefits to consumers.

© The Institute of Chartered Accountants of India


(5) improvements in production or distribution of goods or provision of
services.

(C) Orders of CCI: If after enquiry by the Director General, the Commission finds the
agreement entered into by Shubh Ltd. and Mangal Ltd. are in contravention of
section 3, it may pass all or any of the following orders:

(1) direct Shubh Ltd. and Mangal Ltd. to discontinue and not to re-enter such
agreement.

(2) impose such penalty as it may deem fit which shall not be more than 10% of
the average of the turnover for the last 3 preceding financial years, upon each
of such person or enterprises which are parties to such agreement or abuse;

(3) direct that agreement shall stand modified to the extent and in the manner
as may be specified in the order by the commission

(4) direct Shubh Ltd. and Mangal Ltd to abide by such other orders as the
commission may pass and comply with the directions including payment
of cost, if any.

(5) pass such other orders or issue such directions as it may deem fit.

(ii) Issue of equity shares to NRI’s and transfer of shares by NRIs are capital account
transactions.

RBI may in consultation with the Central Government specify any class or classes of
transactions which are permissible [Section 6(2)(a)].

According to Regulation 3(1) of the Foreign Exchange Management (Permissible capital


Account Transactions) Regulations, 2000 issued by RBI, Investment in India by a person
resident outside India is a permissible capital account transactions (Schedule II).

Further RBI is empowered under Section 6(3)(b) to prohibit, restrict or regulate, by


regulations, transfer or issue of any security by a person resident outside India.

According to Regulation 5(3)(ii) of the said regulations a NRI may purchase shares of an
Indian Company which is not engaged in Print Media Sector on non-repatriation basis
without any limit (para 2 of Schedule 4). The shares may be issued by the company either
by public issue or private placement. The only condition is that the amount of consideration
for purchase of shares shall be paid by way of inward remittance through normal banking
channels from abroad or out of funds held in NRE/FCNR/NRO/NRSR/N&NR account
maintained with an authorized dealer or as the case may be with an authorised bank in India
(Para 3 of Schedule 4).

© The Institute of Chartered Accountants of India


FINAL COURSE
ELECTIVE PAPER 6D: ECONOMIC LAWS
CASE STUDY - 4
Albert John, right from his childhood had high dreams and aspirations. His hobbies included travelling, scuba
diving, gliding, trekking, and other adventurous sports. To go around the globe was his biggest wish.
Therefore, after his schooling, to satisfy his prime interest in travelling, he opted for a two years’ Diploma and
thereafter, a 18 months’ Post Graduate Diploma in Tourism and Travel Industry Management, from the
University of Mumbai.
After post-graduation he had a lot of options like to become a travel agent or join any immigration and customs
services, travel agencies, airline catering or laundry services, etc. Initially, however, he got a chance to join
a reputed travel agency where he gathered on-hand experience and continued with the job for about two
years but his innermost desire was to do his own business and therefore, he opened up a proprietary travel
agency under the name of ‘John Travel Agents’ in Mumbai. Within a period of about five years he could earn
substantially from his business. He used to arrange foreign currency also through his contacts for needy
tourists who did not want to use banking channels for this purpose.
In the meantime, he got married to Neelima George and was comfortably settled in his 2BHK flat in Mumbai.
Keeping in view the future expansion of his travel agency business, he decided to form a private limited
company by the name ‘John and George Travel Agency Pvt. Ltd. having its registered office in Mumb ai. After,
it was got registered with ROC, Mumbai with the authorised capital of Rs. 10 lacs, Albert wrapped up its
business conducted so far under ‘John Travel Agents’.
His business was flourishing. By now he had seen most of the touring destinations of E urope and East Asia
by taking his clients around. One of such clients was Chimanbhai Patel, a leading and famous exporter of
Mumbai. He was a rich and dynamic businessman dealing in gold, diamonds and precious as well as semi -
precious stones. He had three companies i.e. Shining Gold Jewellery Pvt. Ltd., which dealt in gold jewellery;
Red Star Pvt. Ltd. which dealt in diamonds and jewellery studded with diamonds; and Blue Sapphire Pvt. Ltd.
which dealt in precious and semi-precious stones. He was the owner of two palatial bungalows situated at
Bandra and Juhu.
Once he took Chimanbhai, his wife and both of their married daughters as well as sons -in-law to Europe for
a family vacation trip. He arranged for them Royal Caribbean’s cruise liner ‘Harmony of th e Seas’ which was
like a five star hotel at sea, for a ‘seven night’ cruise starting from Barcelona. While on board, Chimanbhai
proposed him a business deal which required him to deliver gold biscuits worth Rs. one crore to one of his
close friends on his next visit to Hong Kong and for accomplishing this job he would get some hefty
commission. After some hesitation he agreed to do the assigned work and the deal was done. Albert managed
to somehow pass on the tainted wealth as directed by Chimanbhai. In return he got rich kickbacks in the form
of commission; and also admired this new way of earning quick money. The bond between Albert and Patel
grew intense and he accomplished many such assignments including converting of Indian currency
representing black money into foreign currency and delivering it outside India to a safe haven.
This way slowly and gradually Albert entered into money laundering activities. Time passed on. He was a rich
person now. He purchased a new 3BHK flat in the same locality and rented out his old flat. He also acquired
properties in Uttrakhand and Rajasthan and at the same time invested additional funds in purchasing gold
jewellery and diamonds including buying a rust coloured Mahindra XUV500. Though the current line of activity
helped him in fulfilling his high dreams and aspirations but in actuality he was converting proceeds of crime
to make them appear as legitimate money. He was a changed person now keeping the moral ethics at bay.

© The Institute of Chartered Accountants of India


Once, while travelling in a Vayu Airways flight from Hong Kong to Mumbai he was impressed by the hospitality
provided by Neetu Bhatia, a member of the cabin crew. An idea clicked him. Albert knew very well that flight
attendants had access to secure venues at airports. At times, they did not require baggage screening. The
cabin crews underwent minimum security check and therefore, needle of suspicion and surveillance was also
minimal in their case. Thinking so, Albert befriended her and through his mesmerizing talks he could gather
that she was a resident of Vile Parle, Mumbai and a regular employee on this route. He developed a story
which reflected his persona as that of a business tycoon. Neetu was highly impressed by the sweet talks and
manners exhibited by Albert and they exchanged mobile numbers.
A few days later, Chimanbhai sent requisite Indian currency to Albert for conversion into around one lac US
Dollars and its deliverance to his business associate in Hong Kong. This time Albert thought of Neetu, called
her and they met at a high profile restaurant ‘Green Tea Day’ in Worli. Albert disclosed her about delivering
of USD one lac in exchange for handsome commission to one of his known and trusted business associates
who would get the money lifted from the Hong Kong International Airport itself. Initially, Neetu was a little bit
hesitant but the desire to earn some quick cash without putting in much effort prevailed over her good senses
and she relented. Both of them, however, knew that it was a criminal conspiracy.
Albert had drawn a very simple modus operandi for her. He wrapped the stacks of dollars in aluminium foils
and carbon sheets to dodge x-ray machine at the Mumbai airport. After placing them in her suitcase he put
her make-up kit and clothes over them. At the security check, as he had anticipated, the foils were passed
off as chocolates. After landing at Hong Kong Airport, Albert’s local conduit picked up the cash from there.
As promised, Neetu got the commission for deliverance. Not being caught in her first operation, Neetu’s
confidence level rose to a considerable extent. A few other consignments, delivered through her, were a no
glitch operation but in the seventh one Albert’s luck ran out and this operation was spoiled by Enforcement
of Directorate (ED) officials who caught her before the plane could take off from the Chhatrapati Shivaji
International Airport, Mumbai.
After being caught, Neetu got frightened and spilled the beans. She was taken under custody by Directorate
of Enforcement Officers and her thorough investigation revealed the involvement of Albert. Her offence of
carrying foreign currency on the behest of Albert was considered to be a Scheduled Offence falling in Part A
of the Schedule to the PML Act (i.e. criminal conspiracy involving Section 120B of the IPC). After following
due procedures including filing a complaint before the jurisdictional Magistrate for taking cognizance of the
scheduled offence, her residence was also searched and gold jewellery worth Rs. 21 lacs was recovered and
Dy. Director duly authorised by the Director took steps to provisionally attach the recovered jewellery in the
presence of two independent witnesses.
Simultaneously, following due procedures, a search team headed by Dy. Director raided the house of Albert.
By the time the officers of ED entered his house, Albert was almost ready to go on a trip to Dubai as a part
of routine job but with a special mission. The officers could smell a rat and took him to his rust coloured
Mahindra XUV500. Immediate search of his car gave way to the recovery of 24 kgs. of gold which was going
to be smuggled out of India through various conduits. It transpired from Albert tha t the gold belonged to
Chimanbhai Patel, a famous exporter of Mumbai.
From the search of Albert’s residence, various incriminating documents were also recovered. In one of the
almirahs, there was a hidden bottom drawer but the hawk eyes of ED officers were able to detect it. Albert
was asked to open it but he did not oblige giving lame excuse that the keys were misplaced. This compelled
the officers to break open it. When opened forcibly, this secret drawer contained five silver pouches where
narcotic drugs were securely kept. On further enquiry it was found that he had two lockers in two different
banks. A search of the lockers gave way to the recovery of fixed deposits receipts worth Rs. 1.25 crores, hard
cash Rs. 50 lacs and property papers showing properties in Uttrakhand and Rajasthan. In both the properties
his name was not registered as the owner. The title documents of residential property at Uttrakhand contained

© The Institute of Chartered Accountants of India


the name of Raj Karan, his driver and the property in Rajasthan was a farm house wh ich was in the name of
Sanju who was the husband of the full time maid-servant Rani working at his house. Both the properties
seemed to be benami properties. Recovery of a green diary from one of the lockers confirmed the name of
Chimanbhai Patel and the various transactions Albert had with him. Time was up for both of them.
Since dealing in drugs was a Scheduled offence under the PML Act, the ED Officers, filed a complaint before
the jurisdictional Magistrate for taking cognizance of the scheduled offence. Thereafter, following the property
attachment procedures, the Dy. Director duly authorised by the Director provisionally attached and seized all
the movable and immovable properties as well as records. Identification marks were placed and an inventory
was made in respect of seized property and records. This was done in the presence of two independent
witnesses.
At the time when Albert revealed the involvement of Chimanbhai, immediately, a search team under the
supervision of duly authorised Dy. Director was sent to the palatial bungalow of Chimanbhai Patel situated at
Bandra. It was found that the bungalow was spread over 5000 sq.ft. approximately. A search of the basement
of his bungalow revealed presence of narcotic drugs and psychotropic substances. In between the wooden
partitions used in the basement, they also found counterfeit Indian currency valuing Rs. 40 crores. It was a
Scheduled offence falling in Part A of the Schedule to the PML Act where amount involved had no
consideration. It was alleged by the Dy. Director that Chimanbhai Patel possessed proceeds of crime but
tried to project the same as untainted property and therefore he was guilty of offence of money-laundering
under Section 3 of the PML Act.
The other - Jal Tarang Mahal residence - a 7 BHK villa of Chimanbhai Patel at Juhu was also raided
concurrently by the ED officials. It was really a humongous, palatial sea-facing bungalow covering
approximately 25,000 sq.ft. area and fully done up with imported and handpicked interiors. The building had
basement, ground floor and a first floor. In a two-day long search, the officials seized, inter-alia, thirty diamond
rings worth Rs. 30 crores; fifty watches worth Rs. five crores; the choicest of rare paintings by M. F. Hussain,
Hebbar, Tyeb Mehta and Amrita Shergil valuing approximately Rs. 21.5 crores which were displayed in a
special air-tight hall so that moisture in the air could not damage them; high end and antique jewellery valued
at Rs. 46 crores; high end cars which included Rolls Royce Ghost, Mercedes Benz, Porsche Panamera, Ford
Mustang, Toyota Fortuner and Innova. The total attachment and seizure of diamonds, gold, precious and
semi-precious stones and other movable and immovable assets stood at Rs. 6562 crores. His various
companies were also searched and a number of incriminating documents, files, computers, etc. were seized.
The intensive search revealed that Chimanbhai used to bring his own black money from about twenty shell
companies based at Hong Kong and Dubai into the accounts of his three main companies in India as foreign
direct investment. He subsequently diverted these funds into the accounts of various shell companies
describing transfers as unsecured loans from where the funds were siphoned off through various means
including cash withdrawals.
An investigation was also conducted under FEMA, 1999 for alleged violations of Sections 3 and 4 of FEMA
for dealing in and acquiring and holding foreign exchange in his account with United Royal Bank of
Switzerland whose value in Indian currency was approximately Rs. 3,600 crores.
Being a Scheduled offence falling in Part A, it was required of authorised ED Officers, to file a complaint
before the jurisdictional Magistrate for taking cognizance of the offence which was done immediately.
Thereafter, following the provisions of Section 17, the ED Officers seized all the movable and immovable
properties as well as records in the presence of two independent witnesses. As in the case of Albert,
identification marks were placed and an inventory of the seized property was also made.
All of them were arrested by the authorised ED Officers, since ED officers, on the basis of material in their
possession, had reason to believe that they were guilty of an offence punishable under PML Act. I mmediately

© The Institute of Chartered Accountants of India


after their arrest, the officers forwarded a copy of the order along with the material in their possession to the
Adjudicating Authority in a sealed envelope, in the prescribed manner. Further, the guilty persons were, within
24 hours, taken to the jurisdictional Magistrate.
As we have noted earlier, keeping in view Section 5 (1), in all the above cases, the ED Officers, through
written orders provisionally attached the properties because it was suspected that they were derived from the
proceeds of crime. The ED Officers knew that the maximum period of attachment would be limited to 180
days from the date of the order.
Thereafter, the ED Officers forwarded the copies of the orders provisionally attaching the properties of Neetu,
Albert and Chimanbhai Patel along with the various documents in his possession to the Adjudicating Authority
in a sealed envelope.
The ED Officers also filed complaints stating the facts of such provisional attachments before the Adjudicating
Authority within thirty days of such attachments.
The Adjudicating Authority served on Neetu, Albert and Chimanbhai Patel notices to explain in not less than
30 days their source of income, earning or assets out of which they had acquired the attached property.
The attachment of the properties was confirmed by the Adjudicating Authority bearing in mind that such properties
were involved in money laundering being obtained through the proceeds of crime. However, such confirmation was
made only after considering the replies of the aggrieved persons as well as after hearing them.
In terms of confirmation order passed by the Adjudicating Authority, the ED Officers, forthwith took the
possession of the attached properties.
The trial of the above money laundering offences is being done by the jurisdictional Special Cou rt. The Central
Government in consultation with the Chief Justice of the High Court is empowered to designate one or more
Courts of Sessions as Special Court or Special Courts for trial of offence of money laundering .
Under Section 4, if Neetu, Albert and Chimanbhai Patel are found to have committed the offence of money-
laundering, then they shall be punishable with rigorous imprisonment which shall be minimum three years
and maximum seven years and shall also be liable to fine. In case it is proved that th e proceeds of crime
involved in money-laundering relate to any offence specified under paragraph 2 of Part A of the Schedule,
the maximum punishment may extend to ten years instead of seven years.
On conclusion of a trial, if the Jurisdictional Special Court finds that the offence of money-laundering has
been committed, it shall order that the properties involved in the money laundering shall stand confiscated to
the Central Government.
If on conclusion of a trial, the Special Court finds that the offence of money laundering has not taken place,
it shall order release of such property to the person entitled to receive it.
Required (MCQ of 2 marks each)
Select the most appropriate answer from the options given for each question:
1. Whether the 2BHK flat owned by Albert but rented out can be considered to have been derived from the
proceeds of crime:
(a) Yes it can be considered because Albert, the owner, is involved in money laundering activities;
(b) No, it cannot be considered because Albert did not purchase it from funds obtained through money
laundering activities;
(c) No, it cannot be considered because Albert has rented it out;
(d) None of the above.

© The Institute of Chartered Accountants of India


2. Adjudicating Authority may serve a notice of not less than --------- on Neetu, Albert and Chimanbhai
Patel who are believed to have committed offence of money laundering to explain their source of income,
earning or assets out of which they had acquired the attached property.
(a) 14 days
(b) 30 days
(c) 60 days
(d) None of the above
3. After provisional order of attachment is confirmed by the Adjudicating Authority, the Director shall
forthwith -------------.
(a) confiscate the attached properties;
(b) take the possession of the attached properties;
(c) seize the attached properties;
(d) None of the above
4. Provisional attachment of property of Chimanbhai Patel suspected to be involved in money laundering
ensures that he is prohibited to:
(a) transfer the attached property;
(b) convert the attached property;
(c) dispose of the attached property;
(d) All of the above
5. A complaint with the Adjudicating Authority is to be filed within a period of --------- days by the Director
who provisionally attaches the property involved in money laundering.
(a) 15
(b) 20
(c) 30
(d) 60
6. If on conclusion of a trial by the Jurisdictional Special Court, the guilt of Neetu, Albert and Chimanbhai
Patel is proved, it shall make an order to --------
(a) Freeze the attached property;
(b) Confiscate the attached property;
(c) Seize the attached property;
(d) None of the above.
7. Chimanbhai Patel was found to have been in possession of counterfeit Indian currency which is a
Scheduled offence belonging to:
(a) the Unlawful Activities (Prevention) Act, 1967;
(b) the Indian Penal Code;
(c) the Prevention of Corruption Act, 1988;
(d) None of the above.
8. An offence specified in Part B of the Schedule shall be considered as Scheduled offence under PML Act
only if the total value involved in such offence is --------.
5

© The Institute of Chartered Accountants of India


(a) Rs. 30.00 lacs or less;
(b) Rs. 50.00 lacs or less;
(c) Rs. 1.00 crore or more;
(d) None of the above.
9. Both Raj Karan and Sanju are to be considered as Benamidar because they are the:
(a) fictitious persons who have not made any payment for purchase of properties;
(b) persons in whose name the benami properties are held without making any payment by them;
(c) persons who have lent their names to be owners of the properties without making any payment by
them;
(d) All of the above.
10. Any property found to be involved in money laundering cannot be provisionally attached by the Director,
ED for more than ---------------.
(a) 30 days
(b) 60 days
(c) 90 days
(d) 180 days
Required (Descriptive Questions of 10 marks each)
1. As per the facts, Albert through laundered money purchased 3 BHK. Suppose if the said flat is purchased
by him jointly on his and his wife’s name, Neelima Goerge. Examine in the light of the Prevention of
Money Laundering Act 2002, the following situtaions:
(a) Will Neelima be also liable for holding of the such joint property.
(b) If property is claimed by a person, other than whom the notice has been issued. Discuss the legal
position of the person claiming the property.
2. (a) Suppose Mr. X, a non-resident Indian, purchases a flat of Albert in India, for Rs. 50,00,000 and
paid 30,00,000 in by account payee cheque of his own account and rest in cash. The registry was
done at a value of Rs 30,00,000 which was paid by cheque. Discuss the nature of the transac tion.
(b) (i) Albert was assigned by Chiman Bhai to deliver counterfeit currency-notes to one of his close
friends to Honkong for which hefty commission was fixed by the Chiman Bhai. Discuss,
whether the said act can be considered as money laundering. Who shall be liable for the
commission of the money Laundering.?
(ii) State whether maintenance of foreign currency accounts in India and outside India by Albert
is permittable in FEMA, due to his nature of business.
3. (i) Chiman bhai is a person resident in India. He has different business units as to manufacturing &
designing of jewellary in Hongkong which is owned by him. How will you determine whether a
particular business units of Chiman bhai is a ‘person resident in India’ under the Foreign Exchange
Management Act, 1999?
(ii) Suppose if ‘Blue Sapphire Pvt. Ltd.” is a Singapore based company having several business units
all over the world. It has a unit for cutting & manufacturing precious and semi-precious stones in
the form to be used for the jewellary with its Headquarters in Mumbai. It has a Branch in Dubai
which is controlled by the Headquarters in Mumbai. What would be the residential status under the
FEMA, 1999 of units of Blue Sapphire Pvt. Ltd in Mumbai and that of Dubai branch?

© The Institute of Chartered Accountants of India


ELECTIVE PAPER 6D- ECONOMIC LAWS

SOLUTION – CASE STUDY 4

I. ANSWERS TO OBJECTIVE TYPE QUESTIONS

1. (b)

[ Hint: Based on Section 2 (1) (u) of the Prevention of Money-Laundering Act, 2002]

2. (b);

[Hint: Refer Section 8 of the Prevention of Money-Laundering Act, 2002].

3. (b);

[Hint: Refer section 8 (4) of the Prevention of Money-Laundering Act, 2002].

4. (d);

[Hint: Based on Section 2 (1) (d) of the Prevention of Money-Laundering Act, 2002]

5. (c);

[Hint: Refer Section 5 (5) of the Prevention of Money-Laundering Act, 2002].

6. (b);

[Hint: Refer Section 8 (3) and 8 (6) of the Prevention of Money-Laundering Act, 2002].

7. (b);

[Hint: Refer Schedule to the Prevention of Money-Laundering Act, 2002]

8. (c);

[Hint: Refer Section 2 (1) (y) of the Prevention of Money-Laundering Act, 2002].

9. (d);

Hint: Refer Section 2 (10) of the Prohibition of Benami Property Transactions Act, 1988.

10. (d):

[Hint: Refer Section 5 (1) of the Prevention of Money-Laundering Act, 2002].

II. ANSWERS TO DESCRIPTIVE QUESTIONS

1. According to section 8 of the Prevention of Money Laundering Act, 2002, on receipt of a


complaint or applications, if the Adjudicating Authority has reason to believe that any person

© The Institute of Chartered Accountants of India


has committed an offence of money laundering or is in possession of proceeds of crime, he may
serve a notice of not less than thirty days.

Such person shall be called upon to indicate the sources of his income, earning or assets, out
of which or by means of which he has acquired the property so or, seized or frozen.

However, where a notice specifies any property as being held by a person on behalf of any other
person, a copy of such notice shall also be served upon such other person. Where if, such
property is held jointly by more than one person, such notice shall be served to all persons
holding such property.

The Adjudicating Authority shall, after hearing the aggrieved person and the Director or any
other officer authorised by him in this behalf, and taking into account all relevant materials
placed on record before him, by an order, record a finding whether all or any of the properties
referred to in the notice issued , are involved in money-laundering. Provided that if the property
is claimed by a person, other than a person to whom the notice had been issued, such person
shall also be given an opportunity of being heard to prove that the property is not involved in
money-laundering.

According to the above stated provisions, following are the answers:

(a) Since in the given case, Alberts holds the property jointly in his and his wife’s name i.e.
Neelima George. As per the above law, such notice shall be served to all persons holding
such property. So accordingly, Neelima will also be served the notice, and being heard.
Taking into account all relevant materials placed on record before him, by an order, record
a finding whether all or any of the properties referred to in the notice issued , are involved
in money-laundering, then in such case Neelima will also be liable for holding of the joint
property.

(b) If property is claimed by a person, other than whom the notice has been issued therein,
such person shall also be given an opportunity of being heard to prove that the property is
not involved in money-laundering.

2. (a) According to the Explanation given to section 2(9) of the Prohibition to Benami
Transaction Act, Benami transaction shall not include any transaction involving the
allowing of possession of any property to be taken or retained in part performance of a
contract, where—

(i) consideration for such property has been provided by the person to whom
possession of property has been allowed but the person who has granted
possession thereof continues to hold ownership of such property;

(ii) stamp duty on such transaction or arrangement has been paid; and

© The Institute of Chartered Accountants of India


(iii) the contract has been registered

Since the property is in the name of Mr. X and not in others name and it is registered on
duly paid stamp duty, it is not a Benami Transaction.

(b) (i) As per the Prevention of Money Laundering Act, 2002, whosoever directly or
indirectly attempts to indulge or knowingly assists or knowingly is a party or is
actually involved in any process or activity connected with the proceeds of crime
including its concealment, possession, acquisition or use and projecting or
claiming it as untainted property shall be guilty of offence of money laundering
(Section 3).

“Proceeds of crime” means any property derived or obtained, directly or indirectly,


by any person as a result of criminal activity relating to a scheduled offence or the
value of any such property [Section 2(1)(u)].

Every Scheduled Offence is a Predicate Offence. The occurrence of the scheduled


Offence is a pre requisite for initiating investigation into the offence of money
laundering.

In the given case , Chiman Bhai assigned Albert to deliver counterfeit currency
notes to be given to his friends in Hongkong , which is an offence falling within the
purview of scheduled offence in Part A of the PMLA, 2002 under section 489B of
the IPC. This section deals with the using as genuine, forged or counterfeit
currency-notes or bank-notes. According to the section whoever sells to, or buys
or receives from, any other person, or otherwise traffics in or uses as genuine, any
forged or counterfeit currency-note or bank-note, knowing or having reason to
believe the same to be forged or counterfeit, shall be liable under the Prevention of
Money Laundering Act.

Hence, Albert, Chiman Bhai and his friends in Hongkong, all are said to be liable
under the Prevention of Money Laundering Act.

(ii) The Foreign Exchange Management (Permissible Capital Account Transactions)


Regulations, 2000 specifies list of transactions, which are permissible in respect of
persons resident in India in Schedule I.

Schedule I states the list of permissible classes of transactions made by persons


resident in India. List specifies, maintenance of foreign currency accounts in India
and outside India by a person resident in India. Accordingly, maintenance of foreign
currency accounts in India and outside India by Albert is permittable in FEMA.

© The Institute of Chartered Accountants of India


3. (i) Person resident in India

Section 2(v) of FEMA, 1999 defines the term “person resident in India”. According to Section
2(v) (iii), all business units in India will be “resident in India” even though these units are
owned or controlled by a person resident outside India.

Similarly all business units outside India will be ‘resident in India’ provided the business
units are either owned or controlled by a person resident in India [Section 2(v) (iv)].

It is necessary to determine the residential status of the person (i.e.,Chiman bhai) who
owns or controls the business units in outside India.

(ii) Blue Sapphire Pvt. Ltd., being a Singapore based company would be person resident
outside India [(Section 2(w)]. Section 2 (u) defines ‘person’ under clause (vii) thereof, as
person would include any agency, office or branch owned or controlled by such person. The
term such person appears to refer to a person who is included in clause (i) to (vii).
Accordingly Blue Sapphire Pvt. Ltd. unit in Mumbai, being a branch of a company would be
a ‘person’.

Section 2(v) defines a person resident in India. Under clause (iii) thereof person resident in
India would include an office, branch or agency in India owned or controlled by a person
resident outside India. Blue Sapphire Pvt. Ltd unit in Mumbai is owned or controlled by a
person resident outside India, and hence it, would be a ‘person resident in India.’

However, Dubai Branch though not owned, is controlled by Blue Sapphire Pvt. Ltd. unit in
Mumbai which is a person resident in India. Hence prima facie, it may be possible to hold a
view that the Dubai Branch is a person resident in India.

© The Institute of Chartered Accountants of India


ELECTIVE PAPER 6D: ECONOMIC LAWS

Case Study 5
Mrs. Shakuntala Bisht was a dynamic woman entrepreneur running her factory of manufacturing designer candles
and other items made of wax as a proprietary concern in Dehradun (Uttrakhand) titled as M/s. Bisht Designer
Candles since 2003. She had appointed a number of dealers pan India for selling her designer products.
She was residing in a bungalow on Subhash Road in Dehradun along with her family. Her husband Mr. O. P. Bisht
was joint-secretary in Uttrakhand Sachivalaya. Her son Varun had done his B.E. (Bachelor of Engineering) from
IIT, Kharagpur and thereafter MBA from IIM, Kolkata in the year 2013. Her daughter Latika was pursuing B.Sc.
(Hons.) in Physics from DIT University, Dehradun.
Varun, being a brilliant student, secured a job in Accenture through Campus placement. He attended a three
months’ residential training programme and joined as Assistant Manager (Operations) in Pune branch of the
company. He took a one BHK flat on rent at Hinjewadi locality, purchased some furniture and other daily household
items and got himself settled in the new atmosphere. He was happy and content as the package offered to him
was very lucrative.
Mrs. Bisht had high aspirations and was desirous of expanding her business further. Therefore, in the year 2013,
she thought of exporting her products to various countries crossing the borders of India. After discussing with her
family members, she decided to convert her proprietary concern into a private limited company. Accordingly, she
got registered her company under the title Bisht Designer Candles Pvt. Ltd. in which she and her daughter were
directors while all of the four family members were shareholders. Thereafter, she completed various formalities
required for exporting her product which, inter-alia, included obtaining a ten digit importer-exporter code (IEC)
number from Directorate General of Foreign Trade (DGFT).
In the year 2015, she sent her first export consignment of designer candles to a foreign buyer in Berlin, Germany.
The order amounted to € 20800 and the importer was required to make payment in three months after shipment.
As per the terms and conditions a Letter of Credit (L/C) was opened by the Deutsche Bank on behalf of the importer.
Before shipping goods, Mrs. Bisht had to fill requisite export declaration form since the consignment did not fall in
exempted category as mentioned in Regulation 4 of the FEM (Export of Goods and Services), Regulations, 2015.
After shipment of goods, she submitted the documentary bill of exchange drawn under L/C to Syndicate Bank,
Dehradun and got it discounted under her sanctioned bills discounting limit. On the due date Syndicate Bank
received the export payment and squared off her liability.
Subsequently, she explored candle market in the USA and came in contact with M/s. Williams’ Art Gallery in Boston
which had a five storey departmental store. In this store, one of the floors was meant only for designer candles and
other items made of wax. After due negotiations with the CEO Mr. Williams, she managed to get an advance of
50,000 USD being 50% of the total export value. It was well within her knowledge that in case an advance was
received against export to a foreign buyer, the shipment of goods was to be made within one year of receipt of
advance and the export documents were required to be routed through the same authorised dealer which received
the advance on her behalf. She shipped the goods much before one year and also got payment well within the
statutory period of nine months from the date of export.
On the of successful settlement of her first export consignment to M/s. Williams’ Art Gallery of Boston, she took
steps to complete another export order from the same party for USD 1,00,500. However, this time no advance
payment was made by the importer and on the basis of his firm order, she dispatched the consignment of designer
candles. After shipment of goods, she submitted the documentary B/E to Syndicate Bank, Dehradun for
discounting. As per the agreement, the importer was to make payment on the completion of five months from the

© The Institute of Chartered Accountants of India


receipt of consignment at his godown. However, by the time five months were over, the importer could make
payment of only 40% of the total export value.
Being in need of funds, she started raising and collecting funds from various sources. In one of the cases she had
given an unsecured loan of Rs. 5 lacs to a private limited company in which a distant relative of her husband was
a director. However, when she demanded her loan back from the company, it was transpired that the company
was under liquidation process before the National Company Law Tribunal under Insolvency and Bankruptcy Code,
2016.
Varun was doing his job at Pune to the complete satisfaction of his superiors. In the next three years’ time after
joining Accenture, Varun could save a lot of money as he was a man of few needs. One day, a casual talk with the
local grocer Ajay Gupta gave him an idea to buy a flat in a housing society. Ajay gave him the phone number of a
known property dealer, Mr. Rajnikant. Thereafter, a meeting was fixed in the office of Mr. Rajnikant where he
noticed a Certificate of Registration hanging on the wall of his office. On enquiry, he was told that now it was
mandatory for the property agents to get themselves registered under Real Estate (Regulation and Development)
Act, 2016. After seeing the certificate Varun could conclude that he was dealing with a genuine person. After due
negotiations, a ground floor 2BHK apartment was finalised in Vayudoot Apartments at a cost of Rs. 62.35 lacs. He
himself arranged Rs. 30 lacs out of his savings; obtained a housing loan of Rs. 20 lacs from Axis Bank while the
remaining amount of Rs. 12.35 lacs was given by his father out of his personal savings. The title deeds got
registered in his name after making payment of stamp duty and other statutory dues. On an auspicious day Varun
shifted to his new flat.
About after a month of shifting to his own flat, Varun’s boss called him and informed that recognizing his hard work
and devotion towards the company he was being transferred to Boston, USA on promotion as Manager
(Operations). He was beaming with happiness and thanked his boss from the bottom of his heart. He was supposed
to join within next one month.
He went back to Dehradun, completed various formalities including obtaining of visa, packed his belongings and
bade goodbye to his family. On the advice of his father he leased out his flat on rent to a reputed private company
and then flew to Boston and joined his job. Over there, he was provided with a furnished apartment by the company
in the suburbs of Boston. As daily commuting was a bit difficult, he purchased a second-hand SUV. Slowly and
gradually he settled in his new home, new office and new country.
Here in Dehradun, Mrs. Bisht was pursuing vigorously to obtain export payment from M/s. Williams’ Art Gallery
because the statutory period of nine months was over long back and the remaining payment was yet to be received.
In the meantime, the authorities at Syndicate Bank also started pressurizing Mrs. Bisht to get the foreign exchange
realised at the earliest since the statutory period of nine months was already over. They opined that in cases of
default the Reserve Bank of India may also issue appropriate directions for the purpose of securing the payment if
the goods were sold in USA or if they were still unsold to get them re-imported into India within the specified period.
Though the Reserve Bank had not so far issued any directions but according to her bankers, omission on the part
of RBI to give directions did not absolve her from the consequences of committing the contravention. Therefore,
she was duty bound to realise the export payment as early as possible.
Besides taking various steps, she also persuaded her son Varun who was already in Boston to follow the matter
vigourously and advised him to meet Mr. Williams personally and settle the case. A meeting was fixed and during
conversation, it was transpired that though Mr. Williams had sold whole of the consignment, the purchaser was yet
to make payment because of some mismanagement. However, on the vigourous persuasion of Varun, Mr. Williams
exerted pressure on the local purchasers and within next one month, remaining payment along with interest was
realised and repatriated to India.

© The Institute of Chartered Accountants of India


Varun had a school friend Raman Verma in India who had done MBA from Symbiosis, Pune after his graduation
from Dehradun and had joined sales team of LIC at Shimla. From time to time after joining Accenture in Boston,
Varun was persuading him to visit Boston and nearby areas along with his wife Vaishnavi Verma. At last, Raman
and his wife agreed for the foreign visit and both of them obtained visa.
Raman approached Canara Bank, Shimla for purchase of USD 12,000 for a private visit to the USA. The bank
without much formalities gave him the required amount in foreign currency since it was well within USD 2,50,000,
i.e. an amount which could be remitted by a resident individual in a financial year under Liberalised Remittance
Scheme (LRS). Moreover, the foreign currency was not required to be remitted for any prohibited current account
transaction [mentioned in Schedule I to the FEM (Current Account Transactions) Rules, 2000] like participation in
lottery schemes or lottery like schemes existing under different names like money circulation scheme or
remittances for the purpose of securing prize money/awards, etc. He was asked to submit a simple letter containing
the basic information, viz., his name, address and that of beneficiary (i.e. self), SB account number, amount to be
remitted and the purpose of remittance along with a cheque of equivalent amount in rupees. In no time, both of
them reached the USA.
Varun received Raman and his wife with open heart at the Logan International Airport, Boston and all of them
drove to his residence. The next ten days were full of fun and frolic. They visited a number of famous sites which
included John F. Kennedy Presidential Museum & Library, Boston Public Library which was opened in 1852 as the
first free publicly-supported municipal library in America, Museum of Fine Arts having world's most comprehensive
art collections, Boston Public Garden famous for its Swan Boats and having over 600 varieties of trees, Old North
Church & Historic Site where the two famous signal lanterns were hung launching the American Revolution, New
England Holocaust Memorial where its six glass towers represented the six million Jews who perished in the
holocaust, Bunker Hill Monument, etc. In between, they had an overnight stay at New York as well.
Varun helped them in purchasing some nice dresses, chocolates, perfumes, cosmetic items and also some
souvenirs for their relatives and friends in India. They enjoyed their trip to USA to the fullest and flew back to India
with nice memories.
Raman still had with him unspent amount of USD 3500. On enquiry with his bankers regarding surrender of this
amount he was informed that he could surrender to the bank any unspent foreign exchange within a period of 180
days from the date of his return to India. Even if he approached the bank after this period, the bank would not
refuse to purchase unspent foreign exchange merely because the prescribed period of 180 days had expired. He
was further informed that he was permitted to retain with him foreign currency notes up to USD 2000 and foreign
coins without any ceiling beyond 180 days and he could utilize this amount for his subsequent visit abroad.
Varun wanted to be inform regarding sale of his flat in Pune if he was to settle down in the USA permanently since
his family at Dehradun was not that much inclined to keep the flat. He once again approached Mr. Rajnikant and
enquired whether he, as NRI, could sell his flat. Mr. Rajnikant after obtaining necessary information from one of
his lawyer friends, informed him that he was permitted to sell his flat in India to a person resident in India. Further,
he could also sell the flat (since it was not an agricultural or plantation property or farm house) to a person resident
outside India who is an Indian citizen or to a person of Indian origin resident outside India. Such permission was
available under Regulation 3 of FEM (Acquisition and Transfer of Immovable Property in India), Regulations, 2000.
As regards purchase of immovable property at Boston, Varun was informed that FEMA did not restrict such
acquisition by a non-resident Indian and he had to follow local laws in this respect. However, if his family members
in India remitted to him funds under the Liberalised Remittance Scheme (LRS) for purchasing immovable property
outside India, then the said property should be in the name of all the members who made the remittances. Even
as per Section 6(4) of the FEMA, if he becomes a person resident in India in future, he would be allowed to hold,
own or transfer the immovable property situated outside India because such property was acquired by him when
he was resident outside India.
3

© The Institute of Chartered Accountants of India


I. Objective Type Questions (2 marks each)
Select the most appropriate answer from the options given for each question:
1. Which of the following remittance would require prior approval of the Reserve Bank of India?
(a) Donation exceeding 0.5% of foreign exchange earning during the previous three financial years or USD
40,00,000, whichever is less for contribution to funds promoted by educational institutes,
(b) Commission per transaction to agents abroad for sale of commercial plots in India of USD 20,000 or 4%
of the inward remittance whichever is more,
(c) Remittance exceeding USD 10,00,000 per project for other consultancy services procured from outside
India.
(d) Remittance of 4% of investment brought into India or USD 90,000 whichever is higher, by an entity in
India by way of reimbursement of pre-incorporation expenses.
2. Mr. O. P. Bisht’s name does not appear in the registration papers relating to Pune apartment purchased by
Varun though he contributed Rs. 12.35 lacs towards the cost of the apartment.
(a) It is a benami transaction to the extent of Rs. 12.35 lacs.
(b) It is wholly a benami transaction.
(c) It is not a benami transaction
(d) None of the above
3. Export of the following goods/software would require furnishing of the declaration under FEMA, 1999?
(a) Goods imported free of cost on re-export basis
(b) Publicity material supplied free of payment
(c) By way of gift of goods accompanied by a declaration by the exporter that they are of six lakh rupees in
value
(d) Unaccompanied personal effects of travellers
4. An exporter receiving advance payment against exports from the foreign buyer is required to make the
shipment of the goods within ---------- of receiving advance payment, if export agreement does not mention
anything to the contrary regarding time period:
(a) 6 months
(b) 9 months
(c) One year
(d) One and a half years
5. An Indian citizen resident outside India is permitted to transfer his agricultural property in India to:
(a) any person resident in India
(b) any person resident outside India if he is a citizen of India or a person of Indian origin.
(c) Neither (a) nor (b)
(d) both (a) and (b)
6. Foreign exchange purchased from an authorised dealer by a resident individual, if remains unspent, needs
to be surrendered to the authorised dealer within ------------ of purchase or date of his return to India:
(a) 60 days
4

© The Institute of Chartered Accountants of India


(b) 90 days
(c) 120 days
(d) 180 days
7. In case of goods valuing up to Rs. 5,00,000 as declared by the exporter and sent by way of gift to an importer
in a foreign country:
(a) an export declaration need to be furnished
(b) an export declaration need not be furnished
(c) furnishing of export declaration depends upon the discretion of the authorised dealer who handles
export documents
(d) furnishing of export declaration depends upon the discretion of the Custom authorities
8. The term 'Moratorium' in the Insolvency and Bankruptcy Code, means-
(a) A temporary prohibition on an activity by the competent authority.
(b) A period declared by the NCLT, during which no action can be taken against the Company or the
assets of the Company.
(c) Suspension order of the Board on the debtor's operations.
(d) Order issued by the NCLT prohibiting an action against the creditor.
9. Is it possible for a non-resident Indian to acquire immovable property outside India:
(a) No, it is not possible
(b) Yes, it is possible
(c) Yes, it is possible but subject to the permission of RBI
(d) None of the above
10. As per the Insolvency and Bankruptcy Code, 2016, an Interim Resolution professional approved by the
committee of Creditors:
(a) Can never be replaced until the conclusion of the resolution process
(b) Has a fixed term of 180 days
(c) Can be replaced with 75% voting in favour of the decision and approval of the Board
(d) Can be replaced with 75% voting in favour of the decision.

II Descriptive Questions (10 marks each)


1. Analyse the following situations under the Foreign Exchange Management Act, 1999:
(i) Forex Dealers Ltd. is an Authorised Person within the meaning of Foreign Exchange Management Act,
1999. Reserve Bank of India issued certain directions to the said Authorised Person to file certain returns,
which it failed to file. You are required to state the penal provisions to which the said Authorised Person has
exposed itself.
(ii) Mr. Shekhar resided for a period of 150 days in India during the Financial year 2016-2017 and thereafter
went abroad. He came back to India on 1 st April, 2017 as an employee of a business organization. What
would be his residential status during the financial year 2017-2018?

© The Institute of Chartered Accountants of India


(iii) ‘Printex Computer’ is a Singapore based company having several business units all over the world. It has
a unit for manufacturing computer printers with its Headquarters in Pune. It has a Branch in Dubai which is
controlled by the Headquarters in Pune. What would be the residential status under the FEMA, 1999 of printer
units in Pune and that of Dubai branch?
2. Examine with reference to the provisions of the Foreign Exchange Management Act, 1999 whether there
are any restrictions in respect of the following:-
(i) Drawal of Foreign Exchange for payments due for depreciation of direct investment in the ordinary
course of business.
(ii) A person, who is resident of U.S.A. for several years, is planning to return to India permanently. Can
he continue to hold the investment made by him in the securities issued by the co mpanies in U.S.A.?
(iii) A person resident outside India proposes to invest in the shares of an Indian company engaged in
construction of farm houses.
(iv) A person, who is resident of Canada, is planning to acquire an immovable property in Mumbai.
3. Analyze the following situations under the Real Estate (Regulation and Development) Act, 2016:
(i) Mr. Ram booked a 4 BHK flat under the Gateways project. The project is under supervision of Mr.
Pankaj. Mr. Pankaj without telling the allottees reduced the number of rooms from 4 to 3 himself.
Whether this is allowed under the Act and what remedies does the Allottees have.
(ii) Mr. Vivaan booked a 4 BHK flat under the Flower Valley project for a total cost of Rs. 2 Crore. The
project is under supervision of Mr. Shyam. Mr. Shyam put a condition to pay Rs. 50 Lakhs as an
application fee before entering into a written agreement for sale with Mr. Vivaan. Decide whether the
contention of Mr. Shyam is valid?

© The Institute of Chartered Accountants of India


ELECTIVE PAPER 6D: ECONOMIC LAWS

SOLUTION

Case Study 5

ANSWER 1
I ANSWERS TO OBJECTIVE TYPE QUESTIONS

1. (c) [Hints: Refer Regulation 2 of Schedule III of Foreign Exchange Management (Current
Account Transactions) Rules, 2000]
2. (c) [Hint: Refer Section 2 (9) of the Prohibition of Benami Property Transactions Act, 1988.
It is not a benami transaction because all statutory dues have been paid and his father
knew about the transaction. Therefore, it falls under exempted category. The amount so
contributed can be a loan or gift to the son.]
3. (c) [Hint: Refer Regulation 4 of the Foreign Exchange Management (Export of Goods and
Services) Regulations, 2015]
4. (c) [Hint: Refer Regulation 15 of the FEM (Export of Goods and Services), Regulations, 2000]
5. (a) [Hint: Refer Regulation 3 of FEM (Acquisition and transfer of immovable property in India)
Regulations, 2000]
6. (d) [Hint: Refer Regulation 7 of FEM (Realisation, Repatriation and surrende r of Foreign
Exchange) Regulations, 2015]
7. (b) [Hint Refer Regulation 4 the FEM (Export of Goods and Services), Regulations, 2015
which has exempted such export transaction from furnishing of export declaration]
8. (b) [Hint: Section 14 of the Insolvency and Bankruptcy Code, 2016, describes moratorium. It
is an order passed by the adjudicating authority (NCLT) declaring a moratorium on the
debtor's operations for the period of the Insolvency Resolution Process, during which no
action can be taken against the Company or the assets of the Company. This operates
as a 'calm period' during which no judicial proceedings for recovery, enforcement of
security interest, sale or transfer of assets, or termination of essential contracts can take
place against the debtor.]
9. (b) [Hint: FEMA does not impose any restriction on acquisition of immovable property outside
India by a non-resident Indian. Further, when at a future date the person concerned
becomes a person resident in India, Section 6(4) even permits him to hold, own or transfer
immovable property situated outside India since such property was acquired by him when
he was resident outside India]
10. (c) [Hint: As per section 22 of the Insolvency and Bankruptcy Code, 2016, an Interim
Resolution professional approved by the Committee of Creditors can be replaced with
75% voting in favour of the decision and approval of the Board.
II. ANSWERS TO DESCRIPTIVE QUESTIONS
1. (i) Section 11(3) of the Foreign Exchange Management Act, 1999 states that where any
Authorised person contravenes any direction given by the Reserve Bank of India under the
said Act or fails to file any return as directed by the Reserve Bank of India, the Reserve Bank
of India may, after giving reasonable opportunity of being heard, impose on Autho rised
Person, a penalty which may extend to ten thousand rupees and in the case of continuing

© The Institute of Chartered Accountants of India


contraventions with an additional penalty which may extend to two thousand rupees for every
day during which such contravention continues.
Since as per the facts given in the question, the Authorised person, namely, Forex Dealers
Ltd., has failed to file the returns as directed by the Reserve Bank of India. According to the
above provisions, it has exposed itself to a penalty which may extend to ten thousand rupees
and in the case of continuing contraventions in the nature of failure to file the returns, with an
additional penalty which may extend to two thousand rupees for every day during which such
contravention continues.
(ii) According to the provisions of section 2(v) of the Foreign Exchange Management Act, 1999,
a person in order to qualify for the purpose of being treated as a "Person Resident in India"
in any financial year, must reside in India for a period of more than 182 days duri ng the
preceding financial year. In the given case, Mr. Shekhar has resided in India for a period of
only 150 days, i.e., less than 182 days, during the financial year 2016-2017. Hence, he cannot
be considered as a "Person Resident in India" during the financial year 2017-2018 irrespective
of the purpose or duration of his stay.
(iii) Printex Computer being a Singapore based company would be person resident outside India
[(Section 2(w)]. Section 2 (u) defines ‘person’ under clause (vii) thereof, as person would
include any agency, office or branch owned or controlled by such person. The term such
person appears to refer to a person who is included in clause (i) to (vii). Accordingly printex
unit in Pune, being a branch of a company would be a ‘person’.
Section 2(v) defines a person resident in India. Under clause (iii) thereof person resident in
India would include an office, branch or agency in India owned or controlled by a person
resident outside India. Printex unit in Pune is owned or controlled by a person resident outside
India, and hence it, would be a ‘person resident in India.’
However, Dubai Branch though not owned is controlled by Printex unit in Pune which is a
person resident in India. Hence prima facie, it may be possible to hold a view that t he Dubai
Branch is a person resident in India.
2. Capital Account Transactions: All the transactions referred to in the question are capital account
transactions.
Section 6(2) of FEMA, 1999 provides that the Reserve Bank may in consultation with the Centra l
Government specify the permissible capital account transactions and the limit upto which foreign
exchange will be allowed for such transactions.
(i) Depreciation of direct investments: According to proviso to section 6(2), the Reserve bank
shall not impose any restriction on the drawal of foreign exchange for certain transactions.
One such transaction is drawal of foreign exchange for payment due for depreciation of direct
investment in the ordinary course of business. Hence this transaction is permissi ble without
any restrictions.
(ii) Person resident in USA returning permanently to India: When the person returns to India
permanently, he becomes a resident in India. Section 6(4) provides that a person resident in
India may hold, own, transfer or invest in foreign currency, foreign security, etc. if such
currency, security or property was acquired, held or owned by such person when he was
resident outside India or inherited from a person who was resident outside India. In view of
this, the person who returned to India permanently can continue to hold the foreign security
acquired by him when he was resident in U.S.A.
(iii) Investment in shares of Indian company by non-resident: Reserve Bank issued Foreign
Exchange Management (Permissible Capital Account Transactions) Regulations, 2000.
Regulation 4(6) of the said Regulations prohibits a person resident outside India from making
investment in India, in any form, in any Company or partnership firm or proprietary concern
or any entity, whether incorporated or not, which is engaged or proposes to engage in

© The Institute of Chartered Accountants of India


construction of farm houses. Hence it is not possible for a person resident outside India to
invest in the shares of a company engaged in construction of farm houses as such investment
is prohibited.
(iv) Acquisition of immovable property by person resident outside India: Reserve Bank
issued Foreign Exchange Management (Permissible Capital Account Transactions)
Regulations, 2000. The regulations specify the classes of capital account transactions of
persons resident outside India in Schedule II. Under this schedule, acquisition and transfer of
immovable property in India by a person resident outside India is permissible. Hence, the
person resident of Canada can acquire the immovable property in Mumbai.
3. (i) Adherence to sanctioned plans and project specifications by the promoter (Section 14)
The proposed project shall be developed and completed by the promoter in accordance with
the sanctioned plans, layout plans and specifications as approved by the competent
authorities.
Notwithstanding anything contained in any law, contract or agreement, after the sanctioned
plans, layout plans and specifications and the nature of the fixtures, fittings, amenities and
common areas, of the apartment, plot or building, as the case may be, as approved by the
competent authority, are disclosed or furnished to the person who agree to take one or more
of the said apartment, plot or building, as the case may be, the promoter shall not make —
(1) any additions and alterations in the sanctioned plans, layout plans and specifications
and the nature of fixtures, fittings and amenities described therein in respect of the
apartment, plot or building, as the case may be, which are agreed to be taken, without
the previous consent of that person.
Provided that the promoter may make such minor additions or alterations as may be
required by the allottee, or such minor changes or alterations as may be necessary due
to architectural and structural reasons duly recommended and verified by an authorised
Architect or Engineer after proper declaration and intimation to the allottee.
Explanation.—For the purpose of this clause, "minor additions or alterations" excludes
structural change including an addition to the area or change in height, or the removal
of part of a building, or any change to the structure, such as the construction or removal
or cutting into of any wall or a part of a wall, partition, column, beam, joist, floor including
a mezzanine floor or other support, or a change to or closing of any required means of
access ingress or egress or a change to the fixtures or equipment, etc.
(2) any other alterations or additions in the sanctioned plans, layout plans and specifications
of the buildings or the common areas within the project without the previous written
consent of at least two-thirds of the allottees, other than the promoter, who have agreed
to take apartments in such building.
Explanation.—For the purpose of this clause, the allottees, irrespective of the number of
apartments or plots, as the case may be, booked by him or booked in the name of his
family, or in the case of other persons such as companies or firms or any association of
individuals, etc., by whatever name called, booked in its name or booked in the name of
its associated entities or related enterprises, shall be considered as one allottee only.
In case any structural defect or any other defect in workmanship, quality or provision of
services or any other obligations of the promoter as per the agreement for sale relating to
such development is brought to the notice of the promoter within a period of five years by the
allottee from the date of handing over possession, it shall be the duty of the promoter to rectify
such defects without further charge, within thirty days, and in the event of promoter's failure
to rectify such defects within such time, the aggrieved allottees shall be entitled to re ceive
appropriate compensation in the manner as provided under this Act.

© The Institute of Chartered Accountants of India


Hence, in the instant case, reducing the number of rooms does not come under minor
additions or alterations. The promoter i.e. Mr. Pankaj Gupta shall not make any additions and
alterations in the sanctioned plans, layout plans and specifications within the project without
the previous written consent of at least two-thirds of the allottees, other than the promoter,
who have agreed to take apartments in such buildings.
(ii) No deposit or advance to be taken by promoter without first entering into agreement
for sale
According to section 13 of the said Act, a promoter shall not accept a sum more than ten per
cent of the cost of the apartment, plot, or building as the case may be, as an advance payment
or an application fee, from a person without first entering into a written agreement for sale
with such person and register the said agreement for sale, under any law for the time being
in force.
In the instant case, the cost of the flat is Rs. 2 crore and Mr. Shyam put a condition to pay
Rs. 50 Lakhs as an application fee before entering into a written agreement for sale with Mr.
Vivaan. This is invalid as a promoter can accept only Rs.20 Lakhs (10% of Rs. 2 Crore) as an
advance or an application fee without first entering into a written agreement for sale.

© The Institute of Chartered Accountants of India


ELECTIVE PAPER 6D: ECONOMIC LAWS

Case Study 6
Ronit Chawla was a Fellow Chartered Accountant (FCA) practicing in the field of corporate and economic laws. He
represented his clients before Company Law Board (CLB) and thereafter in National Company Law Tribunal
(NCLT). After coming into force of Insolvency and Bankruptcy Code, 2016 w.e.f. 28 May, 2016, he learnt about the
Limited Insolvency Examination (LIE) for becoming Insolvency Professional (IP). Since he had about eleven years
of experience as practicing CA, he attempted the very first examination of LIE conducted by Insolvency and
Bankruptcy Board of India (IBBI) in December 2016 and successfully cleared it. He then enrolled himself with a
reputed Insolvency Professional Agency (IPA) and got registered with IBBI by fulfilling the requisite formalities
including payment of non-refundable application fee of Rs. ten thousand.
His father Roopesh Chawla, a resident of Green Park, New Delhi, was recently posted as Chief Manager in Bank
of India, Delhi which was a full-fledged Foreign Exchange (FX) branch though Roopesh, being unable to get a
chance to work in a FX branch, had very little knowledge of rules relating to Foreign Exchange. Therefore, he used
to consult his son Ronit in the matters of foreign exchange from time to time. His mother Rukmani Chawla was a
senior teacher in Kendriya Vidyalaya, New Delhi, taking commerce classes.
Rajnish Sinha, a close friend of Roopesh, was heading a Delhi branch of Punjab National Bank (PNB) and knew
that Roopesh’s son Ronit besides being a Chartered Accountant was also an Insolvency Professional. Rajnish, on
behalf of PNB, wanted to initiate corporate insolvency resolution process (CIRP) before NCLT in the case of its
customer Manohar Masale Pvt. Ltd. (MMPL) of Delhi which had defaulted in repaying the dues of the bank totaling
approximately Rs. 23.00 lacs. Accordingly, PNB being financial creditor, while making an insolvency resolution
application to NCLT proposed the name of Ronit as Interim Resolution Professional (IRP). MMPL was sanctioned
cash credit limit of Rs. 10.00 lacs against hypothecation of stock of raw material and finished goods and another
bill discounting limit of Rs. 5.00 lacs against actionable claims. MMPL was registered with an authorised capital of
Rs. 25.00 lacs but its paid up capital was to the tune of Rs. 10.00 lacs.
Initially started as a registered partnership concern (Manohar Masale & Co.) by two brothers, namely, Ram
Manohar and Shyam Manohar, it did profitable business and keeping an eye on future business growth, it was
converted into a private limited company with Ram, Shyam and Shyam’s elder son Shivam as directors. Shyam’s
younger son Dwapam, an alumnus of IIFT, Delhi and also a law graduate, did not have any interest in the family
business and was more inclined to continue with his current employment in a German MNC having its office in
Gurugram.
MMPL’s factory in Okhla Industrial Area was located on the one-fourth portion of the plot which was co-owned by
the brothers. However, the bank had created an equitable mortgage on the plot as well as factory building while
sanctioning the working capital limits to the company. The elder brother Ram Manohar was the anchoring person
who steered the company to newer heights due to his sheer business acumen and inherent managerial skills but
one day, all of a sudden, he had a massive heart attack resulting in his untimely death. Since he was not married,
the business of ‘masale making’ was now run by Shyam and his son Shivam.
However, the father-son duo could not manage the business properly because of the lack of foresight, faulty inter-
personal relations and poor organisational skills. Their authoritative style of leadership resulted in demotivation of
workers which led to labour unrest and all sort of other conflicts. The paternalistic approach towards them which
Ram always displayed was missing altogether. Needless to say, the output started declining and wastage of raw
material turned north. Since there was no vigourous follow-up as well, the debtors to the tune of around Rs. 12.00
lacs were long overdue. Consequently, the company started suffering losses and also defaulted on dues from the
bank.

© The Institute of Chartered Accountants of India


When PNB, even after repeated reminders to MMPL, could not realise its dues and the liability touched the height
of around Rs. 23.00 lacs (including normal and overdue interest), Rajnish Sinha, on behalf of PNB, decided to file
corporate insolvency resolution application duly supported by ledger extracts and other specified evidences
(services of Information Utility could not be used as by the time application was filed there was no IU registered
with IBBI) with Adjudicating Authority i.e. NCLT, New Delhi for initiating CIRP against MMPL.
NCLT considered the corporate insolvency resolution application along with the proposed name of Ronit as Interim
Resolution Professional (IRP). Within next 10 days of receipt of application (which was lesser than the statutory
period of 14 days) NCLT ascertained that there existed default because the defaulted amount was much more
than the minimum required of Rs. one lac. Since the CIRP application was complete in all respects, NCLT admitted
it and within the statutory period of next seven days after admission, it conveyed its order of commencement of
CIRP to the financial creditor (i.e. PNB) and the corporate debtor (i.e. MMPL).
The order of NCLT confirmed the proposed appointment of Ronit as IRP for 30 days, for Ronit had a clean record
without any disciplinary proceedings pending against him. It was also stated in the order that a moratorium period
of 180 days had become applicable during which all suits and legal proceedings, etc. against MMPL (i.e. corporate
debtor) were to be held in abeyance so as to give time to the ailing company to resolve its status. MMPL was also
barred from transferring or disposing of any of its assets or any legal rights therein. However, the supply of specified
essential goods and services to the MMPL as mentioned in the order, were not to be interrupted during moratorium
period.
In the meantime, Ronit’s father Roopesh faced a peculiar problem related to the foreign exchange matter at his
branch. His FX officer brought to his knowledge that one of their exporter customers who had received an advance
of USD 75,000 from an importer based at California, USA against export of ready-made jeans had not shipped the
requisite items worth USD 2,00,000 by utilizing the advance so received. The exporter, not willing to ship the goods,
wanted to refund the advance to the importer along with interest for which permission of Roopesh was required.
Roopesh did not allow the refund immediately and in turn, advised the FX officer to gather more knowledge about
FX provisions whether refund along with interest was permissible. At the same time he also discussed the matter
with his son Ronit who advised him to refer FEM (Export of Goods and Services) Regulations, 2015. A scrutiny of
the relevant banking records revealed that 14 months had already expired since advance of USD 75,000 was
received. Further, he came to know that if goods were not shipped within one year of receipt of advance, such
advance could not be refunded without the permission of the RBI. Accordingly, he advised the customer to seek
permission of RBI through his branch.
After his appointment as IRP, Ronit assumed full control of the affairs of MMPL. Since powers of the board of
directors stood suspended he was empowered to exercise such powers. Accordingly, he took immediate custody
and control of all the assets of the MMPL including its business records.
Following the orders of NCLT, Ronit took steps to make a public announcement within three days from the date of
his appointment regarding the initiation of CIRP against MMPL.
Public announcement, included the following aspects:
• Name and address of the corporate debtor (i.e. MMPL) and its registration/incorporating authority.
• His details as IRP and the fact that he would be vested with the management of the corporate debtor and
be responsible for receiving claims.
• Penalties for false or misleading claims.
• The last date for the submission of the claims.
• The date on which the CIRP would end.

© The Institute of Chartered Accountants of India


After the expiry of last date for submission of claims, a Committee of Creditors was constituted which included
PNB and five trade creditors who had cumulative dues of Rs. 3.00 lacs. Within seven days of its constitution, the
first meeting of the committee was called. In the meantime, Ronit electronically submitted an Information
Memorandum to the creditors after they had given an undertaking regarding maintaining of confidentiality. This
Information Memorandum contained details of assets and liabilities of the MMPL with their estimated values,
audited financial statements for the last two financial years and provisional financial statements for the current
financial year made just eight days earlier from the date of the application, a list of creditors and the amounts
claimed by them which were duly admitted and other prescribed information.
In the meeting of the Committee of Creditors it was resolved to let Ronit continue as full-fledged Resolution
Professional (RP) since he was eligible to be appointed as an independent director and was not a related party of
the MMPL and such decision was conveyed to the NCLT as well as MMPL. As RP, Ronit assumed all those powers
which were conferred on him as IRP. He was required to manage the operations of the MMPL during the CIRP
period.
Based on the Information Memorandum, Rajnish on behalf of PNB as resolution applicant undertook to prepare a
resolution plan as per the provisions of the Code for onward submission to Ronit. Before finalizing the resolution
plan, he along with his two officers took up the matter with Shyam and his son Shivam regarding the revival of
MMPL and repayment of long outstanding dues or face liquidation if they were not inclined to revive the company.
The fear psychosis of liquidation made them think frantically to save their company from imminent death. Having
woken up from their slumber they started exploring ways to bring in short term finance and also to rope in some
professional who would help the company in its revival.
Shyam saw a ray of hope in his younger son Dwapam and persuaded him to participate in the management of the
affairs of the company at least for the first three months to which he ultimately agreed. In the meantime Shyam,
with a view to raise short term finance, consulted his elder sister Rama Devi to lend at least Rs. 5.00 lacs for a
short period of about one year and also convinced his daughter Ria, her husband Dushyant as well as Dwapam to
invest at least Rs. 3.00 lacs each in the share capital of the company. Shivam who had invested funds in the share
market agreed to sell his securities to raise Rs. 3.00 lacs against which he was to be allotted shares in the MMPL.
As per the advice of the bankers, Shyam also started inter-acting with long overdue debtors for recovery who
eventually agreed to pay 50% of Rs. 12.00 lacs in the current month and remaining amount in the next month. Out
of the raised amount, the operational creditors were to be paid fully while dues of PNB were to be satisfied to the
extent of Rs. 12.00 lacs. Further, Rs. 2.50 lacs were to be allocated towards insolvency resolution process costs
including fee of RP and remaining amount was to be utilized as working capital. Since both the directors of MMPL
had consented to repay Rs. 12.00 lacs in one lump sum, Rajnish on behalf of PNB assured them that he would
take up the matter of waiving of overdue interest up to Rs. 2.00 lacs with his Dy. General Manager and would also
seek permission to revive MMPL’s limits which were currently frozen.
Based on the experience he gathered while working with two MNCs, Dwapam assumed the role of a leader to set
the company on rails. He took note of the prevailing situation from which the ailing MMPL was passing through.
He observed that the current as well as liquid ratios were much far away from the standard norms of 2:1 and 1:1
respectively. The turnover ratios were also unhealthy and at the same time the operating ratio was very high - not
a good sign for any business. An investment of about Rs. 5.00 lacs was tied up in raw material like whole red
chillies, coriander seeds, turmeric, black pepper, dry mango, etc.
Since currently the business of spices was run in a traditional manner, Dwapam decided to take the following short,
medium and long term measures:
Short term measures:
• to understand the needs and wants of customers in the target market;

© The Institute of Chartered Accountants of India


• to apply the principles of scientific management;
• to set standards for raw material, wastage, working conditions, etc.;
• to conduct time and motion studies;
• to provide financial incentives and to adopt social security plans for the workers;
• to secure registration with FSSAI immediately;
• to appoint an Administrative Officer and, if need be, to appoint another one in future;
• to devise competitive pricing strategy;
• to create a corporate brand identity by assigning the product a brand name ‘Manohar Uttam Masale’ which
would help in building a brand image;
• to design an attractive package and label by using a graphic design of spices combining green, yellow
and red colours for different varieties of masale;
• to promote the masale by advertising initially in leading newspapers and depending upon income
generation in future, to advertise on FM radio, TV as well as cinema halls;
• to adopt sales promotion measures like free gift offers, contests, free sample distribution, etc.
• to select the similar channels of distribution as used by the competitors;
• to conduct SWOT analysis of MMPL and important competitors;
• to create an effective Website of the company;
• to take decisions regarding various activities under physical distribution of masale like order processing,
transportation, warehousing and inventory control;
• to adopt strict credit policy by reducing debtors’ days with a regular follow-up;
• to use an accounting software;
• to submit various Government Returns within the prescribed time limits so that avoidable hefty penalties
are not levied.
Medium and Long Term Measures:
• to stop heavy expenditure on repairs and maintenance by installing new machines and grinders;
• to establish direct contacts with the cultivators for obtaining raw material which would help in avoiding
middlemen and their high commissions;
• to develop the remaining three-fourth portion of the plot and rent out some of the developed portion to a
commercial establishment;
• to renovate the factory building.
• to manufacture more types of different spices like Rajma Masala, Pindi Chana Masala, Shahi Paneer
Masala, Dal Makhni Masala, Mushroom Matar Masala, etc;
• to diversify MMPL’s operations by manufacturing Jams and Ketchups;
• To explore offshore markets.
Rajnish prepared a resolution plan containing the above strategies and submitted it to Ronit for his consideration.
Later on, a meeting of committee of creditors was called by Ronit and the resolution plan was presented for its
approval. The plan was duly approved by full majority. Thereafter, Ronit submitted the approved resolution plan to
the NCLT for its approval.
Since the resolution plan was approved by the committee of creditors much before the statutory period of 180 days
and also met the prescribed requirements, NCLT approved it and passed an order to this effect. Now the plan was
binding on the MMPL and its employees, members, PNB and operational creditors as well as other stakeholders
involved in the resolution plan.

© The Institute of Chartered Accountants of India


I. Objective Type Questions (2 marks each)
Select the most appropriate answer from the options given for each question:
1. “Default” under the IBC is said to be occurred on the fulfillment of condition/s-
(a) Debts becoming due and payable
(b) Non- payment of the debt
(c) Liability /obligation in respect of a claim which is due
(d) Both (a) & (b)
2. In the case study PNB initiated Corporate Insolvency Resolution Process against MMPL for the default in the
capacity of-
(a) Corporate debtor
(b) Operational debtor
(c) Financial creditor
(d) Resolution applicant
3. If the goods against which an advance payment is received from a foreign buyer are not shipped within one
year and there exists no agreement regarding timing of shipment, the advance payment:
(a) shall be refunded within reasonable time without prior approval of Reserve Bank.
(b) Shall be refunded within one year from the date of receipt of advance payment without the prior approval
of Reserve Bank
(c) Shall be refunded within one year from the date of receipt of advance payment with the prior approval of
Reserve Bank
(d) Shall be refunded after one year from the date of receipt of advance payment on the basis of reasonable
cause.
4. PNB through an assignment agreement, assigned here the debt to the X trust. X trust filed the petition for
initiation of corporate Insolvency resolution process (CIRP) against MMPL. State the correct statement with
respect to the competency of the X trust in the filing of the petition in the above situation-
(a) X Trust is not a competent applicant as per section 6 of the IBC
(b) X Trust is being authorized by the PNB to file an application
(c) X Trust in the capacity of financial creditor can file a valid petition.
(d) None of the above
5. As per the Insolvency & Bankruptcy Code, 2016, resolution plan is prepared by ----------- is submitted to ------
----------- for examination and submission to ------------- for approval.
(a) Committee of Creditors, Adjudicating Authority, Resolution Professional
(b) Resolution applicant, committee of creditors, Adjudicating Authority
(c) Resolution applicant, Resolution Professional, Committee of Creditors
(d) Committee of Creditors, Resolution Professional, Adjudicating Authority
6. The maximum duration during which the appointment of Interim Resolution Professional (IRP) is valid shall
not exceed -------- days.
(a) 10
(b) 20
(c) 30
5

© The Institute of Chartered Accountants of India


(d) 40
7. In the case study, the expenses of public announcement shall be borne by the-
(a) MMPL
(b) Ronit
(c) Roopesh
(d) PNB
8. In the case study, committee of creditors of MMPL was constituted on 17.3.2018. Time limit, within which the
first meeting of committee of creditors should be held, is ----------------.
(a) 20.3.2018
(b) 22.3.2018
(c) 24.3.2018
(d) 31.3.2018
9. Ronit, being an Insolvency Professional can be appointed as Resolution Professional, if:
(a) he is eligible to be appointed as an independent director under section 149 of the Companies Act, 2013
(b) he is not a related party of the corporate debtor
(c) only (a)
(d) Both (a) and (b)
10. MMPL finds material irregularity in exercise of the powers of the Ronit during the corporate insolvency
resolution period. Remedy available to MMPL-
(a) File a complaint to the adjudicating authority
(b) Complain to the committee of creditor’s
(c) Complaint filed before the IBBI
(d) File an appeal against the order of adjudicating authority against the approval of resolution plan.
II. Descriptive Questions (10 marks each)
1. Suppose the resolution plan prepared by Rajneesh was delayed in approval by committee of creditors. Ronit,
further presented the said resolution plan, before NCLT after 180 days of insolvency commencement date.
Answer the following-
(i) What step shall be taken by NCLT on such presented resolution plan.
(ii) What, if MMPL contravened the resolution plan which effected its employees and stake holders.
(iii) What consequences be there where liquidator continued the business of MMPL during liquidation
process.
2. Ronit in an examination of sale of property of MMPL finds that a transaction was made by the MMPL to Rama
devi (the elder sister of Shyam) in 6 months preceding the Insolvency Commencement date, was undervalued.
Give the following answers in reference to the above situation-
(i) State the validity of the conduct of such transaction by MMPL to Ramadevi.
(ii) What will be the consequences when resolution professional determines such transactions undervalue
and fails to report that same to NCLT?
(iii) What order NCLT shall pass when MMPL entered into an undervalued transaction?

© The Institute of Chartered Accountants of India


3. (a) Discuss the legal position and liability of Mr. Shyam in the following given situations-
(i) Where Mr. Shyam fraudulently transferred his holding of shares in favour of his sister of Rs.1 lakh
within 1 year immediately preceding the insolvency commencement date.
(ii) Mr. Shyam makes false entry in the books of account of MMPL to defraud creditors on insolvency
commencement date.
(iii) Shyam permitted Shivam to provide information for initiation of CIRP which is false in material
particular and omits material fact related to a books of accounts of a specified period in the
application.
(b) What course of action can be taken by NCLT against the directors of the MMPL for transactions
defrauding creditors?

© The Institute of Chartered Accountants of India


ELECTIVE PAPER 6D: ECONOMIC LAWS

SOLUTION

Case Study 6

I. ANSWERS TO OBJECTIVE TYPE QUESTIONS


1. (d) [Hints: As per section 3(12), Default means non-payment of debt when whole or any part
or instalment of the amount of debt has become due and payable and is not repaid by the
debtor or the corporate debtor, as the case may be]
2. (c) [Hints: Financial creditor means any person to whom a financial debt is owed and includes
a person to whom such debt has been legally assigned or transferred to;{section 5(7)}]
3. (b) [Hint: Refer Regulation 15 of the FEM (Export of Goods and Services), Regulations, 2000]
4. (c) [Hint: Refer Section 5 (7) of the Code]
5. (c) [Hint: Refer Section 5 (25) read with section 28 of the Code]
6. (c) [Hint: Refer Section 16 of the Code]
7. (d) [Hint: Refer Section 15 of the Code]
8. (c) [Hint: Refer Section 22 (1)]
9. (d) [Hint: Refer Regulation 3 of the Insolvency and Bankruptcy (Insolvency Resolution
Process for Corporate Persons) Regulations, 2016]
10. (d) [Hint: section 61(3) of the IBC]

II ANSWERS TO OBJECTIVE TYPE QUESTIONS

1. (i) According to section 33 of the Insolvency and Bankruptcy Code, 2016, where the Adjudicating
Authority before the expiry of the insolvency resolution process period does not receive a
resolution plan as approved by the committee of creditors, it shall—
(a) pass an order requiring the corporate debtor to be liquidated as per the relevant
provisions
(b) issue a public announcement stating that the corporate debtor is in liquidation; and
(c ) require such order to be sent to the authority with which the corporate debtor is
registered.
According to section 12 of the Insolvency and Bankruptcy Code, 2016, the corporate
insolvency resolution process (CIRP) shall be completed within a period of one hundred and
eighty days from the date of admission of the application to initiate such process.
As per the facts, Ronit, presented the approved resolution plan, before NCLT after the
prescribed period for the completion of CIRP i..e, after 180 days of insolvency commence ment
date.
According to the above stated provisions, NCLT, shall pass an order requiring the corporate
debtor (MMPL) to be liquidated. It shall issue a public announcement of its liquidation and
send such order to the Registrar of companies.
(ii) As per Section 33(3) of the Insolvency and Bankruptcy Code, 2016, where the resolution plan
approved by the Adjudicating Authority is contravened by the concerned corporate debtor,
any person other than the corporate debtor, whose interests are prejudicially affec ted by such
contravention, may make an application to the Adjudicating Authority for a liquidation order

© The Institute of Chartered Accountants of India


as referred above. Accordingly, the employees and the stakeholders of MMPL, whose
interests are affected by contravention in compliances of the resolut ion plan, may make an
application to NCLT for initiation of liquidation. On receipt of an application, if the Adjudicating
Authority determines that the MMPL has contravened the provisions of the resolution plan, it
shall pass a liquidation order.
(iii) As per section 33(7) of the Insolvency and Bankruptcy Code, 2016, the order for liquidation
shall be deemed to be a notice of discharge to the officers, employees and workmen of the
corporate debtor.
However, where the business of the corporate debtor when continued during the liquidation
process by the liquidator, it shall not be deemed to be notice of discharge to the officers,
employees and workmen of the corporate debtor.
So the Conduct of business of MMPL during liquidation process by the liquidator i s tenable
and shall not be deemed to be notice of discharge to the officers, employees and workmen of
the MMPL.
2. (i) Validity of the conduct of undervalued transaction : As per the provisions given in section
45 of the Insolvency and Bankruptcy Code, 2016, Ronit, on an examination of the
transactions of the MMPL, determines that certain transactions were made by MMPL with a
related party (Rama devi) within the period of two years preceding the insolvency
commencement date (in 6 months preceding the Insolvency Commencement date), which
were undervalued. Ronit, shall make an application to the NCLT to declare such transactions
as void and reverse the effect of such undervalued transaction and requiring the person who
benefits from such transaction to pay back any gains he may have made as a result of such
transaction.
(ii) Failure to report to NCLT of undervalued transactions: As per the stated facts given in
the light of the provisions laid in Section 47 of the Insolvency and Bankruptcy Code, an
undervalued transaction has taken place and Ronit (Resolution Professional) has not reported
it to the NCLT, in such case, a creditor , member or a partner of a MMPL, as the case may
be, may make an application to the NCLT to declare such transactions void and reverse their
effect in accordance with the relevant provisions of this Code.
(iii) Order of NCLT: Where the NCLT, after examination of the application made above, is
satisfied that undervalued transactions had occurred; and Ronit (RP) after having sufficient
information or opportunity to avail information of such transactions did not report such
transaction, there it shall pass an order of —
(a) restoring the position as it existed before such transactions and reversing the effects
thereof in the manner as laid down in section 45 and section 48 of the Code. The order
of the Adjudicating Authority may provide for the following:—
(1) require any property transferred as part of the transaction, to be vested in the
corporate debtor(MMPL);
(2) release or discharge (in whole or in part) any security interest granted by the
corporate debtor (MMPL);
(3) require any person to pay such sums, in respect of benefits received by such
person, to the Ronit (RP), as the Adjudicating Authority may direct; or
(4) require the payment of such consideration for the transaction as may be determined
by an independent expert.
(b) requiring the Board(IBBI) to initiate disciplinary proceedings against Ronit.
3. (a) (i) As per the provisions given in section 68 of the Code, Mr. Shyam, Director (an officer in
default) has within the twelve months immediately preceding the insolvency
commencement date, fraudulently transferred his holding of shares in favour of his sister

© The Institute of Chartered Accountants of India


of Rs.1 lakh (which is more than value of ten thousand rupees ). So, Mr. shyam, shall
be punishable with imprisonment for a term which shall not be less than three years but
which may extend to five years, or with fine, which shall not be less than one lakh rupees,
but may extend to one crore rupees, or with both: However, he shall not be liable to any
punishment under this section if he proves that he had no intent to defraud or to conceal
the state of affairs of the corporate debtor.
(ii) According to section 71 of the Code, on and after the insolvency commencement date,
Mr. Shyam, makes a false entry in the books of account of MMPL with an intent to
defraud or deceive any person, he shall be punishable with imprisonment for a term
which shall not be less than three years, but which may extend to five years, or with fine
which shall not be less than one lakh rupees, but may extend to one crore rupees, or
with both.
(iii) As per Section 77 of the Code, as Shyam permitted Shivam to provide informations in
the application under section 10,which is false in material particular and omits material
fact related to a books of accounts of a specified period, so he shall be punishable
with imprisonment for a term which shall not be less than three years, but wh ich
may extend to five years or with fine which shall not be less than one lakh rupees,
but which may extend to one crore rupees, or with both.
(b) As per section 69 of the Code, on or after the insolvency commencement date, where the
directors of the MMPL—
(a) has made transfer of, or charge on, or has caused or connived in the execution of a
decree or order against, the property of the corporate debtor;
(b) has concealed or removed any part of the property of the corporate debtor within two
months before the date of any unsatisfied judgment, decree or order for payment of
money obtained against the corporate debtor,
such directors of MMPL, shall be punishable with imprisonment for a term which shall not be
less than one year, but which may extend to five years, or with fine, which shall not be less
than one lakh rupees, but may extend to one crore rupees, or with both.
However, directors of MMPL, shall not be punishable under this section if the acts mentioned
in clause (a) were committed more than five years before the insolvency commencement date;
or if he proves that, at the time of commission of those acts, he had no intent to defraud the
creditors of the corporate debtor.

© The Institute of Chartered Accountants of India


ELECTIVE PAPER 6D: ECONOMIC LAWS

Case Study 7

During March 2017, XMC Pvt. Ltd., a car manufacturing company, launched its TXI model of car with a lot of
advertisements and promotions in all types of media platforms, inter alia, highlighting the Ex-showroom price
of the said car model in Mumbai as Rs. 6.25 lacs.
Mr. Nazir, a prospective buyer of the said model, visited an authorised dealer of XMC Pvt. Ltd. i.e. M/s Ratan
Lal & Sons located at Bandra, Mumbai and after due consultation/ discussion with the representatives of M/s
Ratan Lal & Sons, booked a vehicle of the aforesaid model on 11 th May, 2017 on payment of Rs. 100,000/-.
M/s Ratan Lal & Sons in turn provided the money receipt for the aforesa id transaction with serial number
ABC/1010 as well as booking reference number 218/ 2017 to Mr. Nazir. He was assured by the
representatives of M/s Ratan Lal & Sons that the booked vehicle will be delivered within three months from
the date of booking i.e. by 10th August, 2017. However, the representative of M/s Ratan Lal & Sons have
stated to Mr. Nazir that as per XMC Pvt. Ltd.’s policy, five months’ time is given in writing so as to keep some
buffer for delays which may arise due to unforeseen exigencies or transportation of vehicle or other logistic
problems. Mr. Nazir, inter alia, noted the conditions in the booking document that “the vehicle would be
delivered within six months from the date of booking”. Believing the assurance given by the representative of
M/s Ratan Lal & Son, Mr. Nazir accepted the terms of the booking and thought that he will get the vehicle
within three months from the date of booking as assured by the representatives of M/s Ratan Lal & Son and
in worst scenario he will get delivery of the vehicle within six months from the date of booking as per the
terms and conditions of booking of the vehicle.
However, within three months of booking of the vehicle, M/s Ratan Lal & Son failed to deliver the vehicle to
Mr. Nazir despite repeated request and after 10 th August, 2017, Mr. Nazir contacted the representatives of
M/s Ratan Lal & Sons many times for delivery of the vehicle and they kept on giving assurances that the
delivery of the vehicle will be done within six months from the date of booking as per the conditions of booking.
After five months, on 15 th February 2018, Mr. Nazir written an e-mail to XMC Pvt. Ltd. highlighting the issue
of delay in delivery of the booked vehicle, but did not get any response. Then he wrote an email to the
President of XMC Pvt. Ltd. and got the reply that his grievances will be looked into by the sales team of the
Company and the concerned dealer.
Despite the assurance of the president of XMC Pvt. Ltd., the booked vehicle was not delivered to Mr. Nazir.
Rather, through M/s Ratan Lal & Sons, he was informed that due to delay in production of the said model,
the Company is not able to deliver the same and he was asked to wait for some more time. Subsequently, he
received a letter from XMC Pvt. Ltd wherein, inter alia, it was informed that due to unprecedented number of
bookings for the said model the delivery of the car will be delayed for two months. Through the said letter, it
was also informed that the price of the booked car will be revised and it will be eff ective from the date of
booking by dealer to the customer.
About the market and the state of competition
As per Mr. Nazir, XMC Pvt. Ltd. is a big player in the car manufacturing market. Its financial strength and
brand name is much more compared to other players in the market. Also, it commands largest market share
in terms of sales and revenue compared to its competitors and in the last financial year XMC Pvt. Ltd. acquired
a loss making car manufacturing company i.e. Trisha Ventures Pvt. Ltd. As per Mr. Nazir, XMC Pvt. Ltd. has
taken recourse to terms and conditions of the booking documents to enforce price hike and also not honouring
the commitment made for the delivery within the given time period despite repeated correspondence. XMC
Pvt. Ltd. and its dealer at Mumbai M/s Ratan Lal & Sons started the gimmick of non-delivery due to production
delay and started informing that there will be higher price of the vehicle. Mr. Nazir alleged that he and other
similarly situated consumers are being not given with delivery of the vehicle in due time and the delay tactics
done by XMC Pvt. Ltd. is to increase the price of the vehicle and to exploit the consumers by not giving the
benefit of initial launch price which is not fair in a competitive market.

© The Institute of Chartered Accountants of India


Concerns raised
As per Mr. Nazir, XMC Pvt. Ltd. has abused its powers to fix the price of the vehicle. It has initially priced
attractively and launched with heavy advertisements and promotions to lure the customers and take maximum
bookings by taking interest free amount of Rs. 100,000/- as booking amount. By doing this XMC Pvt. Ltd. has
been able to not only generate huge amounts of cash which is interest free but also create buzz in the market
because of publicity in the media regarding heavy bookings of the said vehicle. It is stated that XMC Pvt. Ltd.
has arbitrarily increased the price of the vehicle to encash on the market demand. Not only that, the Company
has also not passed on the benefit of recent GST reduction on the passenger cars by Government to the
consumers in the said car model. However, it has passed on the benefit of the GST reduction on its other car
models to the customers which are not in such demand. Most of the other car manufacturers have duly passed
on the GST reduction to the customers. As per Mr. Nazir, XMC Pvt. Ltd. has indulged in unfair practices in
connivance with its dealers by manipulating its delivery policy and price policy. After seeing huge response
because of attractive initial offer price, it not only delayed in giving delivery of t he booked car but also
increased the price which is nearly two times of the offer price at the time of booking. It has not honored the
commitment of delivery and price to the buyer who had booked on the very first day and first hour of the
launch. Mr. Nazir stated that it is not just an individual issue but it involves the larger interests of car buyers,
who do not have any recourse to effective mechanism against the abuse of dominant position by such auto
manufacturers for imposing anti-competitive terms on the buyers.
Based on the above submissions Mr. Nazir alleged that the aforesaid conduct of XMC Pvt. Ltd. is not in
tandem with the provisions of the Competition Act, 2002 and it has acted in a manner which can be termed
as anti-competitive.
I. Objective Type Questions (2 marks each)
Select the most appropriate answer from the options given for each question:
1. Which of the following is the appropriate authority for redressal of the grievances of Mr. Nazir?
(a) District Consumer Redressal Forum
(b) Competition Commission of India
(c) Car Manufacturers Association of India
(d) Both (a) and (b)
2. Under which provisions of the Competition Act, 2002, the grievances of Mr. Nazir can be examined?
(a) Prohibition of horizontal anti-competitive agreement under section 3(3) of the Competition Act, 2002
(b) Prohibition of abuse of dominant position under section 4 of the Competition Act, 2002
(c) Prohibition of vertical anti-competitive agreement under section 3(4) of the Competition Act, 2002
(d) Regulation of combination under section 6 of the Competition Act, 2002
3. Mr. Nazir stated that “it is not just an individual issue but it involves the larger interests of car buyers, who do
not have any recourse to effective mechanism against the abuse of dominant position by such auto
manufacturers for imposing anti-competitive terms on the buyers”. What would be his prime intention in stating
so?
(a) The car manufacturer’s conduct towards him is exploitative
(b) The car manufacturer is imposing anti-competitive terms on him.
(c) The conduct of car manufacturer is not conducive to the market as it affects larger consumers’ interest.
(d) All the above
2

© The Institute of Chartered Accountants of India


4. Let, Mr. Nazir approached the Competition Commission India for his grievances and you are the person in the
Commission to take a decision in the matter and according to you the matter pertains to abuse of dominance.
What would be your sequence of analysis of the matter?
(a) XMC Pvt. Ltd. is dominant or not
(b) Whether the alleged conduct is abusive under section 4 of the Competition Act, 2002
(c) Whether XMC Pvt. Ltd. falls under the definition of enterprise as defined under the Competition Act,
2002
(d) Define the relevant market where XMC Pvt. Ltd. is operating
5. Let Mr. Nazir approached the Competition Commission India for his grievances and you are the person in the
Commission to take a decision in the matter and according to you the matter pertains to vertical restraint under
section 3(4) of the Competition Act, 2002. What would be your sequence of analysis of the matter?
(a) Whether XMC Pvt. Ltd. and M/s Ratan Lal & Sons have entered into an agreement
(b) Whether XMC Pvt. Ltd. and M/s Ratan Lal & Sons are placed at vertical level.
(c) Whether there is any appreciable adverse effect on competition because of anti-competitive agreement
between XMC Pvt. Ltd. and M/s Ratan Lal & Sons.
(d) Whether XMC Pvt. Ltd. and M/s Ratan Lal & Sons have agreed on some issues which are anti-
competitive in terms of section 3(4) of the Competition Act, 2002.
6. If you think delineation of relevant market is necessary to examine the fact of the case, then what should be
the relevant product market in this case?
(a) Market for passenger car
(b) Market for dealership services for passenger car
(c) Market for motor vehicle
(d) Market for non-commercial passenger car
7. Mr. Nazir submitted that XMC Pvt. Ltd. is a dominant market player in the relevant market, if you agree with
his submission, what would be your reasoning?
(a) Market share of XMC Pvt. Ltd. is largest
(b) Competitors of XMC Pvt. Ltd. have lesser financial strength
(c) XMC Pvt. Ltd. is a known brand
(d) Consumers are dependent on XMC Pvt. Ltd.
8. Given the facts that XMC Pvt. Ltd. and M/s Ratan Lal & Sons, in connivance with each other, have delayed
the delivery of the booked passenger car to Mr. Nazir and revised the price of the said car, it cannot be a case
of cartelization. What would be the possible reason?
(a) The fact does not reveal any exclusive agreement between XMC Pvt. Ltd. and M/s Ratan Lal & Sons.
(b) The fact does not reveal any agreement of XMC Pvt. Ltd. with other car manufacture in fixing the price
(c) The fact does not reveal that M/s Ratan Lal & Sons is involved in price fixation of delay in giving delivery
of the car to Mr. Nazir
(d) None of the above

© The Institute of Chartered Accountants of India


9. In case the Competition Commission of India ordered that Mr. Nazir should approach in the appropriate forum,
what would be your reaction?
(a) The Competition Commission of India is rightly ordered so because the allegations of Mr. Nazir do not
raise any competition concerns in any market.
(b) The order of the Competition Commission of India should be challenged in National Company Law
Appellate Tribunal as it failed to address the concerns of Mr. Nazir in terms of the provisions of
Competition Act, 2002.
(c) Since it is grievance of an individual consumer, Consumer Redressal Forum is the appropriate authority
to deal this matter.
(d) None of the above
10. If you think that XMC Pvt. Ltd. has abused its dominant position, then which of the following conduct of XMC
Pvt. Ltd. is abusive in terms of Section 4 of the Competition Act, 2002?
(a) not giving delivery of the booked car within the assured time
(b) The President of XMC Pvt. Ltd. vide its mail to Mr. Nazir informed that the price of the booked vehicle
will revised and it will be applicable on the date of invoice by dealer to the customer
(c) XMC Pvt. Ltd has not passed the benefit of tax deduction to the consumers
(d) None of the above.
II. Descriptive Questions (10 marks each)
1. Do you think that the concerns raised by Mr. Nazir can be examined through the provisions of the
Competition Act, 2002? If yes, explain the steps through which the matter can be examined.
2. Do you think that not giving delivery of the booked car within the assured time without enabling
provisions in the booking form is tantamount to imposition of unfair conditions and revision of price of
the vehicle with effect from the date booking tantamount to imposition of unfair price on Mr. Nazir?
Examine the given situations in terms of the provisions of the Competition Act, 2002.
3. What is relevant market? State the provisions of the Competition Act, 2002 to delineate the relevant
market. Delineate the relevant market in the instant case.

© The Institute of Chartered Accountants of India


ELECTIVE PAPER 6D: ECONOMIC LAWS

SOLUTION

Case Study 7

I. ANSWERS TO OBJECTIVE TYPE QUESTIONS


1. (a) [Hint: The matter relates to concerns of an individual consumer regarding non-delivery of
booked vehicle in the given time]
2. (b) [Hint: The allegations essentially relate to abuse of dominance by a car manufacturing
company, directly or through its authorized dealer]
3. (c) [Hint: The tried to portray his issue as an issue of consumer exploitation to draw the attention
of the competition authority]
4. c, d, a, b [Hint: To examine a case under section 4 of the Competition Act, 2002, it is to be seen
first whether the alleged entity is an enterprise or not before defining the relevant market,
assessment of its position of dominance in the relevant market and examination of its conduct]
5. b, a, d, c [Hint: To examine a case under section 3 (4) of the Competition Act, 2002, first it is to be
seen whether the alleged two entities are in a vertical chain and whether they have entered into
any agreement as defined under the Competition Act, 2002. Then it is to seen whether such
agreement is anti-competitive and it has appreciable adverse effect on competition]
6. (a) [Hint: Essentially, the allegations relate to passenger car market]
7. (d) [Hint: All are the factors prescribed under section 19 (6) the Competition Act, 2002 to assess
dominance of an enterprise in a relevant market]
8. (b) [Hint: Cartelization requires agreement amongst players placed at horizontal level ]
9. (a) [Hint: No competition concerns raised in the matter as delay in giving delivery to a consumer
or not passing the benefit of tax reduction to consumer or increasing the price cannot said to
be anti-competitive]
10. (d) [ Hint: Refer section 4 of the Competition Act, 2002]
II. ANSWERS TO DESCRIPTIVE QUESTIONS
1. Even though the concerns raised by Mr. Nazir cannot be redressed by the competition authority
as it essentially relates to grievances of an individual consumer of a passenger car manufactured
by XMC Pvt. Ltd, however if the matter is placed before the competition authority it will be examined
in terms of section 4 of the Competition Act, 2002. It is so because the allegations of Mr. Nazir
essentially relate to abuse of dominance by XMC Pvt. Ltd, directly or through its authorized dealer
M/s Ratan Lal & Sons.
To examine the matter under section 4 of the Competition Act, 2002, it is to be seen first whet her
the alleged entity is an enterprise or not before defining the relevant market, assessment of its
position of dominance in the relevant market and examination of its conduct.
Enterprise: Yes, XMC Pvt. Ltd. is an enterprise in terms of Section 2 (h) of the Act.
Relevant Product Market: The market for passenger car [section 2 (t)]
Relevant Geographic Market: whole of India [see section 2 (s)]
Relevant Market: the market for passenger car in India [section 2 (r)]
Assessment of Dominance of XMC Pvt. Ltd.: Appear to be dominant in the market for
passenger car in India as it has highest market share and financial strength besides its brand name
and dependence of the consumer on it.

© The Institute of Chartered Accountants of India


Assessment of the alleged conduct of XMC Pvt. Ltd.: Not appear to be abusive. Delay in giving
delivery of a product to a consumer or not passing the benefit of tax reduction to consumer or
increasing the price cannot said to be anti-competitive in terms of section 4 of the Competition Act,
2002.
2. To examine the matter under section 4 of the Competition Act, 2002, it is to be seen first whether
the alleged entity is an enterprise or not before defining the relevant market, assessment of its
position of dominance in the relevant market and examination of its conduct. (all the steps of
answer no. 1 above to be followed)
Delay in giving delivery of a product to a consumer without enabling provisions in the booing
document may be an issue of breach of contract between two parties. It may not be a case of
imposition of unfair condition in term of the provisions of section 4 of the Competition Act, 2002.
Further, increasing price is a commercial decision of an enterprise which is taken considering the
market demand conditions of the product. If market is competitive then excess price, if any, can be
wiped out in the long run, no intervention of the competition authority is required. However, if the
company raised the price after negotiation with the consumer, it can be challenged in other
appropriate forum.
3. As per section 2(r) of the Act, ‘relevant market’ means the market which may be determined by the
Commission with reference to the relevant product market or the relevant geographic market or
with reference to both the markets. Further, the term ‘relevant product market’ has been defined in
section 2(t) of the Act as a market comprising all those products or services which are regarded as
interchangeable or substitutable by the consumer, by reason of characteristics of the products or
services, their prices and intended use. And, the term ‘relevant geographic market’ has been
defined in section 2(s) of the Act to mean a market comprising the area in which the conditions of
competition for supply of goods or provision of services or demand of goods or services are
distinctly homogenous and can be distinguished from the conditions prevailing in the neighbouring
areas.
In order to determine the ‘relevant product market’, the Commission, in terms of the factors
contained in section 19(7) of the Act, is required to have due regard to all or any of the following
factors viz. physical characteristics or end- use of goods, price of goods or service, consumer
preferences, exclusion of in-house production, existence of specialized producers and
classification of industrial products. Similarly in order to determine the ‘relevant geographic market’,
the Commission, in terms of the factors contained in section 19(6) of the Act, is required to have
due regard to all or any of the following factors viz., regulatory trade barriers, local speci fication
requirements, national procurement policies, adequate distribution facilities, transport costs,
language, consumer preferences and need for secure or regular supplies or rapid after - sales
services.
As stated above, as per the provisions of the Competition Act, 2002 the relevant market comprises
of the relevant product market and relevant geographic market. In the instant matter, the relevant
product market may be considered as the ‘market for passenger car’. It may be noted that the
allegations of Mr. Nazir pertains to purchase and after sale service of a passenger car which cannot
be substitutable with other type of vehicle in terms of price, end use, characteristics, etc. The
relevant geographic market in this matter may be considered as ‘India’ because the condition of
competition in passenger car market in India is homogeneous throughout India. A consumer can
buy a passenger car from any part of India with similar competitive condition. Thus, the market for
passenger car in India may be considered as the relevant market in this case.

© The Institute of Chartered Accountants of India

Vous aimerez peut-être aussi